You are on page 1of 63

CONSTI I - CHAPTER 5

CASE DIGESTS
CHAPTER 5 - PRINCIPLES AND STATE POLICIES (Article II, Sections 1 - 28)

ARTICLE II
Declaration of Principles and State Policies
Section 1

SECTION 1. The Philippines is a democratic and republican State. Sovereignty resides in the people and
all government authority emanates from them.

Case no. 060


Villavicencio v. Lukban
G.R. No. L-14639, Mar. 25, 1919
JING
FACTS:

Justo Lukban – then Mayor of the City of Manila


Zacarias Villavicencio, et. al. – relatives and friends of the deportees

One hundred and seventy women, who had lived in the segregated district for women of ill repute in the city of Manila,
were by orders of the Mayor of the city of Manila and the chief of police of that city isolated from society and then at
night, without their consent and without any opportunity to consult with friends or to defend their rights, were forcibly
hustled on board steamers for transportation to Davao. No law, order, or regulation authorized the Mayor of the city of
Manila or the chief of the police of that city to force citizens of the Philippine Islands to change their domicile from
Manila to another locality.
The women were received as laborers in a banana plantation. Some of the women were able to escape and return to
Manila. The attorney for the relatives and friends of a considerable number of the deportees presented an application
for habeas corpus to the Supreme Court

ISSUE:
Whether or not Mayor Lukban has the right to deport women with ill repute.

HELD:
That the writ of habeas corpus was properly granted, and that the Mayor of the city of Manila who was primarily
responsible for the deportation, is in contempt of court for his failure to comply with the order of the court. The
respondents had no authority to deport the women. No official, no matter how high, is above the law. The courts are
the forum which function to safeguard liberty and to punish official transgressors. The essential object and purpose of
the writ of habeas corpus are to inquire into all manner of involuntary restraint and to relieve a person therefrom if
such restraint is illegal. If the mayor and the chief of police could deport the women, they must have the means to
return them from Davao to Manila. The respondents may not be permitted to restrain a fellow citizen of her liberty by
forcing her to change her domicile and to avow the act with impunity in the courts. The great writ of liberty may not be
easily evaded. No one of the defense offered constituted a legitimate bar to the granting of the writ of habeas corpus.
We are clearly a government of laws. Lukban committed a grave abuse of discretion by deporting the prostitutes to a
new domicile against their will. There is no law expressly authorizing his action. On the contrary, there is a law
punishing public officials, not expressly authorized by law or regulation, who compels any person to change his
residence.
Furthermore, the prostitutes are still, as citizens of the Philippines, entitled to the same rights, as stipulated in the Bill
of Rights, as every other citizen. Their choice of profession should not be a cause for discrimination. It may make
some, like Lukban, quite uncomfortable but it does not authorize anyone to compel said prostitutes to isolate
themselves from the rest of the human race. These women have been deprived of their liberty by being exiled to
Davao without even being given the opportunity to collect their belongings or, worse,without even consenting to being
transported to Mindanao. For this, Lukban etal must be severely punished.
CONSTI I - CHAPTER 5
CASE DIGESTS

Section 2

SECTION 2. The Philippines renounces war as an instrument of national policy, adopts the generally
accepted principles of international law as part of the law of the land and adheres to the policy of peace, equality,
justice, freedom, cooperation, and amity with all nations.

Case no. 061


Kuroda s. Jalandoni
G.R. No. L-2662, Mar. 26, 1949
JING

FACTS:
Sheginori Kuroda - was the former formerly a Lieutenant-General of the Japanese Army and commanding general of
the Japanese forces during the occupation (WWII) in the Philippines. He was tried before the Philippine Military
Commission for War Crimes and other atrocities committed against military and civilians. The military commission was
establish under Executive Order 68.

Rafael Jalandoni – Major General of the Military Commission


Executive Order No. 68 - establishing a National War Crimes Office and prescribing rules and regulations
governing the trial of accused war criminals, was issued by the President of the Philippines on the 29th day of July,
1947. This Court holds that this order is valid and constitutional.
Article 2, Section 3 of our Constitution provides that: "The Philippines renounces war as an instrument of national
policy, and adopts the generally accepted principles of international law as part of the law of the nation."
Petitioner argues that respondent Military Commission has no jurisdiction to try petitioner for acts committed in
violation of the Hague Convention and the Geneva Convention because the Philippines is not a signatory to the first
and signed the second only in 1947.

ISSUE:
Whether or not E.O. No. 68 is constitutional and the military tribunal jurisdiction is valid.

HELD:
E.O. No. 68 is constitutional hence the tribunal has jurisdiction to try Kuroda. It cannot be denied that the
rules and regulations of the Hague and Geneva conventions form part of and are wholly based on the generally
accepted principles of international law. In fact, these rules and principles were accepted by the two belligerent
nations, the United States and Japan, who were signatories to the two Conventions. Such rules and principles,
therefore, form part of the law of our nation even if the Philippines was not a signatory to the conventions
embodying them, for our Constitution has been deliberately general and extensive in its scope and is not confined to
the recognition of rules and principles of international law as contained in treaties to which our government may
have been or shall be a signatory.
Executive Order No. 68 establishes a National War Crimes Office, and the power to establish government
office is essentially legislative. The order provides that persons accused as war criminals shall be tried by military
commissions. Whether such a provision is substantive or adjective, it is clearly legislative in nature. It confers upon
military commissions jurisdiction to try all persons charged with war crimes. The power to define and allocate
jurisdiction for the prosecution of persons accused of any crime is exclusively vested by the Constitution in
Congress.
Furthermore, when the crimes charged against petitioner were allegedly committed, the Philippines was under the
sovereignty of the United States, and thus we were equally bound together with the United States and with Japan,
to the rights and obligations contained in the treaties between the belligerent countries. These rights and obligations
CONSTI I - CHAPTER 5
CASE DIGESTS
were not erased by our assumption of full sovereignty. If at all, our emergence as a free state entitles us to enforce
the right, on our own, of trying and punishing those who committed crimes against our people.

Case no. 062


Mijares v. Ranada
G.R. No. 139325, April 12, 2005
JING

FACTS:
Priscilla C. Mijares - judge of the Regional Trial Court of Pasay
Hon. Santiago Javier Ranada - Presiding Judge of Branch 137, Regional Trial Court, Makati City
On 9 May 1991 - a complaint was filed with the United States District Court (US District Court), District of Hawaii,
against the Estate of former Philippine President Ferdinand E. Marcos (Marcos Estate). The action was brought
forth by ten Filipino citizens who each alleged having suffered human rights abuses such as arbitrary detention,
torture and rape in the hands of police or military forces during the Marcos regime.
The Alien Tort Act was invoked as basis for the US District Court's jurisdiction over the complaint, as it involved a
suit by aliens for tortious violations of international law. These plaintiffs brought the action on their own behalf and
on behalf of a class of similarly situated individuals, particularly consisting of all current civilian citizens of the
Philippines, their heirs and beneficiaries, who between 1972 and 1987 were tortured, summarily executed or had
disappeared while in the custody of military or paramilitary groups. Plaintiffs alleged that the class consisted of
approximately ten thousand (10,000) members.
The US District Court certified the case as a class action and created three (3) sub-classes of torture, summary
execution and disappearance victims. Trial ensued, and subsequently a jury rendered a verdict and an award of
compensatory and exemplary damages in favor of the plaintiff class.
The Final Judgment was eventually affirmed by the US Court of Appeals for the Ninth Circuit, in a decision rendered
on 17 December 1996.
On 20 May 1997 - the present petitioners filed Complaint with the Regional Trial Court, City of Makati (Makati RTC)
for the enforcement of the Final Judgment. They alleged that they are members of the plaintiff class in whose favor
the US District Court awarded damages. They argued that since the Marcos Estate failed to file a petition for
certiorari with the US Supreme Court after the Ninth Circuit Court of Appeals had affirmed the Final Judgment, the
decision of the US District Court had become final and executory, and hence should be recognized and enforced in
the Philippines.
On 5 February 1998 - Marcos Estate filed a motion to dismiss, raising, among others, the non-payment of the
correct filing fees. It alleged that petitioners had only paid Four Hundred Ten Pesos (P410.00) as docket and filing
fees, notwithstanding the fact that they sought to enforce a monetary amount of damages in the amount of over Two
and a Quarter Billion US Dollars (US$2.25 Billion). The Marcos Estate cited Supreme Court Circular No. 7,
pertaining to the proper computation and payment of docket fees. In response, the petitioners claimed that an action
for the enforcement of a foreign judgment is not capable of pecuniary estimation.
Petitioners submit that their action is incapable of pecuniary estimation as the subject matter of the suit is the
enforcement of a foreign judgment, and not an action for the collection of a sum of money or recovery of damages.

ISSUE:
Whether or not the enforcement of a foreign judgment is incapable of pecuniary estimation

HELD:
Pettiion GRANTED.
Petitioners' complaint may have been lodged against an estate, but it is clearly based on a judgment, the Final
Judgment of the US District Court. The provision does not make any distinction between a local judgment and a
foreign judgment, and where the law does not distinguish, we shall not distinguish.
CONSTI I - CHAPTER 5
CASE DIGESTS

Section 48, Rule 39 of the Rules of Civil Procedure states:


SEC. 48. Effect of foreign judgments. — The effect of a judgment of a tribunal of a foreign country, having
jurisdiction to pronounce the judgment is as follows:

(a) In case of a judgment upon a specific thing, the judgment is conclusive upon the title to the thing;

(b) In case of a judgment against a person, the judgment is presumptive evidence of a right as between the parties
and their successors in interest by a subsequent title;

There is an evident distinction between a foreign judgment in an action in rem and one in personam. For an action in
rem, the foreign judgment is deemed conclusive upon the title to the thing, while in an action in personam, the foreign
judgment is presumptive, and not conclusive, of a right as between the parties and their successors in interest by a
subsequent title. However, in both cases, the foreign judgment is susceptible to impeachment in our local courts on
the grounds of want of jurisdiction or notice to the party, collusion, fraud, or clear mistake of law or fact. Thus, the
party aggrieved by the foreign judgment is entitled to defend against the enforcement of such decision in the local
forum. It is essential that there should be an opportunity to challenge the foreign judgment, in order for the court in this
jurisdiction to properly determine its efficacy.

There is no obligatory rule derived from treaties or conventions that requires the Philippines to recognize foreign
judgments, or allow a procedure for the enforcement thereof. However, generally accepted principles of international
law, by virtue of the incorporation clause of the Constitution, form part of the laws of the land even if they do not derive
from treaty obligations.

The classical formulation in international law sees those customary rules accepted as binding result from the
combination two elements: the established, widespread, and consistent practice on the part of States; and a
psychological element known as the opinion juris sive necessitates (opinion as to law or necessity). Implicit in the
latter element is a belief that the practice in question is rendered obligatory by the existence

Case no. 063


Poe-Llamanzares v. Comelec
G.R. No. 221697, Mar. 8, 2016
JING

FACTS:
Petitioner Mary Grace Natividad S. Poe-Llamanzares was found abandoned as a newborn infant in the Parish Church
of Jaro, Iloilo on Sept. 3, 1968. After passing the parental care and custody over petitioner by Edgardo Militar to
Emiliano Militar and his wife, she has been reported and registered as a foundling and issued a Foundling Certificate
and Certificate of Live Birth, thus was given the name, Mary Grace Natividad Contreras Militar.
When the petitioner reached the age of five (5), celebrity spouses Ronal Allan Kelley (aka Fernando Poe, Jr) and
Jesusa Sonora Poe (aka Susan Roces) filed a petition foe her adoption. The trial court granted their petition and
ordered that her name be changed to Mary Grace Natividad Sonora Poe.

Petitioner registered as a voter in San Juan City at the age of 18 in 1986; in 1988, she applied and was issued
Philippine Passport by the DFA; in 1993 and 1998, she renewed her passport.
She left for the United States (U.S.) in 1988 to continue her studies after enrolling and pursuing a degree in
Development Studies at the University of the Philippines
She married Teodoro Misael Daniel V. Llamanzares, a citizen of both the Philippines and the U.S., in San Juan City
and decided to flew back to the U.S. after their wedding. She gave birth to her eldest child while in the U.S.; and her
two daughters in the Philippines.

She became a naturalized American citizen in 2001. She came back to the Philippines to support her father’s
candidacy for president in the May 2004 elections and gave birth to her youngest daughter. They then returned to the
U.S. in 2004 but after few months, she rushed back to the Philippines to attend to her ailing father.
After her father’s death, the petitioner and her husband decided to move and reside permanently in the Philippines in
2005.
CONSTI I - CHAPTER 5
CASE DIGESTS

In 2006, She took her Oath of Allegiance to the Republic of the Philippines pursuant to RA No. 9225 or the Citizenship
retention and Re-acquisition Act of 2003; she filed a sworn petition to reacquire Philippine citizenship together with
petitions for derivative citizenship on behalf of her three children which was granted. She registered as a voter;
secured Philippine passport; appointed and took her oath as Chairperson of the MTRCB after executing an affidavit of
Renunciation of American citizenship before the Vice Consul of the USA and was issued a Certificate of Loss of
Nationality of the USA in 2011.

In 2012, she filed with the COMELEC her Certificate of Candidacy (COC) for Senator for the 2013 Elections wherein
she answered “6 years and 6 months” to the question “Period of residence in the Philippines before May 13, 2013.”
Petitioner obtained the highest number of votes and was proclaimed Senator on 16 May 2013.
On 15 October 2015, petitioner filed her COC for the Presidency for the May 2016 Elections. In her COC, the
petitioner declared that she is a natural-born citizen and that her residence in the Philippines up to the day before 9
May 2016 would be ten (10) years and eleven (11) months counted from 24 May 2005. The petitioner attached to her
COC an “Affidavit Affirming Renunciation of U.S.A. Citizenship” subscribed and sworn to before a notary public in
Quezon City on 14 October 2015.

Petitions were filed before the COMELEC to deny or cancel her candidacy on the ground particularly, among others,
that she cannot be considered a natural-born Filipino citizen since she cannot prove that her biological parents or
either of them were Filipinos. The COMELEC en banc cancelled her candidacy on the ground that she was in want of
citizenship and residence requirements, and that she committed material misrepresentations in her COC.

ISSUE:
Whether or not Mary Grace Natividad S. Poe-Llamanzares is a natural-born Filipino citizen.

HELD:
Mary Grace Natividad S. Poe-Llamanzares may be considered a natural-born Filipino.
It ruled that a foundling is a natural-born citizen of the Philippines as there is no restrictive language which would
definitely exclude foundlings as they are already impliedly so recognized.

The principles found in two conventions, while yet unratified by the Philippines, are generally accepted principles of
international law.
The first is Article 14 of the 1930 Hague Convention on Certain Questions Relating to the Conflict of Nationality Laws
under which a foundling is presumed to have the “nationality of the country of birth,” to wit:
Article 14
A child whose parents are both unknown shall have the nationality of the country of birth.If the child's parentage is
established, its nationality shall be determined by the rules applicable in cases where the parentage is known.
A foundling is, until the contrary is proved, presumed to have been born on the territory of the State in which it was
found. (Underlining supplied)
The second is the principle that a foundling is presumed born of citizens of the country where he is found, contained in
Article 2 of the 1961 United Nations Convention on the Reduction of Statelessness:
Article 2
A foundling found in the territory of a Contracting State shall, in the absence of proof to the contrary, be considered to
have been born within the territory of parents possessing the nationality of that State.
In sum, all of the international law conventions and instruments on the matter of nationality of foundlings were
designed to address the plight of a defenseless class which suffers from a misfortune not of their own making. The
COMELEC, by its own admission, disregarded the evidence that petitioner actually and physically returned here on 24
May 2005 not because it was false, but only because COMELEC took the position that domicile could be established
only from petitioner's repatriation under R.A. No. 9225 in July 2006. However, it does not take away the fact that in
reality, petitioner had returned from the U.S. and was here to stay permanently, on 24 May 2005. When she claimed
to have been a resident for ten (10) years and eleven (11) months, she could do so in good faith.

Case no. 064


Ichong v. Hernandez
G.R. No. L-7995, May 31, 1957
CONSTI I - CHAPTER 5
CASE DIGESTS
JING

FACTS:

Jaime Hernandez – Secretary of Finance


Lao H. Ichong - is a Chinese businessman in the markets of Pasay City who seeks to declare the nullification of RA
1180 for it violates the international and treaty obligations of the Republic of the Philippines. The said Act is
unconstitutional, and to enjoin the Secretary of Finance and all other persons acting under him, particularly city and
municipal treasurers, from enforcing its provisions. He contends that RA 1180 denies to alien residents the equal
protection of the laws and deprives of their liberty and property without due process of law
Republic Act No. 1180 known as, “An Act to Regulate Retail Business” was passed by the Congress. The said
RA nationalizes the retail trade business by prohibiting against persons not citizens of the Philippines, as well as
associations, partnerships or corporations the capital of which are not wholly owned by citizens of the Philippines,
from engaging directly or indirectly in the retail trade with the exception of U.S. citizens and juridical entities. Aliens are
required to present registration to the proper authorities a verified statement concerning their businesses.

ISSUE:
Whether or not Republic Act No. 1180 violate international laws and treaties

HELD:
Petition DENIED.
R.A. No. 1180 does not violate international treaties and obligations. The United Nations Charter imposes no strict or
legal obligations regarding the rights and freedom of their subjects, and the Declaration of Human Rights contains
nothing more than a mere recommendation, or a common standard of achievement for all peoples and all nations.
The Treaty of Amity between the Republic of the Philippines and the Republic of China of April 18, 1947 guarantees
equality of treatment to the Chinese nationals "upon the same terms as the nationals of any other country". But the
nationals of China are not discriminated against because nationals of all other countries, except those of the United
States, who are granted special rights by the Constitution, are all Prohibited from engaging in the retail trade. But even
supposing that the law infringes upon the said treaty, the treaty is always subject to qualification or amendment by a
subsequent law, and the same may never curtail or restrict the scope of the police power of the State.

The Court resumed holding that the disputed law was enacted to remedy a real actual threat and danger to
national economy posed by alien dominance and control of the retail business and free citizens and country
from dominance and control. The enactment clearly falls within the scope of the police power of the State, thru
which and by which it protects its own personality and insures its security and future. The law does not violate the
equal protection clause of the Constitution because sufficient grounds exist for the distinction between alien and
citizen in the exercise of the occupation regulated, nor the due process of law clause, because the law is prospective
in operation and recognizes the privilege of aliens already engaged in the occupation and reasonably protects their
privilege.
Case no. 065
Sec. of Justice v. Lantion
G.R. No. 139465, Jan. 18, 2000
JING

FACTS:

Hon. Ralph C. Lantion - Presiding Judge, Regional Trial Court of Manila, Branch 25
Secretary of Justice - Secretary of Justice Franklin M. Drilon, representing the Government of the Republic of the
Philippines, signed in Manila the "Extradition Treaty Between the Government of the Republic of the Philippines
and the Government of the United States of America" (hereinafter referred to as the RP-US Extradition Treaty).
The Senate, by way of Resolution No. 11, expressed its concurrence in the ratification of said treaty. It also expressed
its concurrence in the Diplomatic Notes correcting Paragraph (5)(a), Article 7 thereof (on the admissibility of the
documents accompanying an extradition request upon certification by the principal diplomatic or consular officer of the
requested state resident in the Requesting State).
CONSTI I - CHAPTER 5
CASE DIGESTS
Department of Justice (DOJ) received from the Department of Foreign Affairs U.S. a request for the extradition of
private respondent Mark Jimenez to the U.S. for violation of Conspiracy to Commit Offense, Attempt to Evade Tax,
Fraud by Wire, Radio, or Television, False Statement, and Election Contribution in Name of Another.

During the evaluation process of the extradition, the private respondent, requested the petitioner, Secretary of Justice,
to furnish him copies of the extradition request from the U.S. government, that he be given ample time to comment
regarding the extradition request against him after he shall have received copies of the requested papers, and to
suspend the proceeding in the meantime.

The petitioner, Secretary of Justice denied the request in consistent with Art. 7 of the RP – US Extradition
Treaty which provides that the Philippine Government must represent the interests of the U.S. in any
proceedings arising from an extradition request.

The private respondent filed with the RTC against the petitioner Hon. Ralph Lantion (presiding judge RTC Manila
Branch 25) a mandamus, a certiorari, and a prohibition to enjoin the petitioner, the Secretary of DFA, and NBI from
performing any acts directed to the extradition of the respondent, for it will be a deprivation of his rights to due process
of notice and hearing.

ISSUE:
Whether or not the respondent Mark Jimenez is entitled to the basic rights of due process over the government’s
duties under a treaty?

HELD:
Petition DENIED. Petitioner is ordered to furnish private respondent copies of the extradition request and its
supporting papers, and to grant him a reasonable period within which to file his comment with supporting
evidence.
The human rights of person, Filipino or foreigner, and the rights of the accused guaranteed in our
Constitution should take precedence over treaty rights claimed by a contracting state. The duties of the
government to the individual deserve preferential consideration when they collide with its treaty obligations to the
government of another state. This is so although werecognize treaties as a source of binding obligations under
generally accepted principles of international law incorporated in our Constitution as part of the law of the land.
Due process is comprised of two components — substantive due process which requires the intrinsic validity of the
law in interfering with the rights of the person to his life, liberty, or property, and procedural due process which
consists of the two basic rights of notice and hearing, as well as the guarantee of being heard by an impartial and
competent tribunal.

The doctrine of incorporation is applied whenever municipal tribunals (or local courts) are confronted with situations
in which there appears to be a conflict between a rule of international law and the provisions of the constitution or
statute of the local state. Efforts should first be exerted to harmonize them, so as to give effect to both since it is to be
presumed that municipal law was enacted with proper regard for the generally accepted principles of international law
in observance of the Incorporation Clause in the above-cited constitutional provision|.
The rule of pacta sunt servanda, one of the oldest and most fundamental maxims of international law, requires the
parties to a treaty to keep their agreement therein in good faith. The observance of our country's legal duties under a
treaty is also compelled by Section 2, Article II of the Constitution which provides that "[t]he Philippines renounces war
as an instrument of national policy, adopts the generally accepted principles of international law as part of the law of
the land, and adheres to the policy of peace, equality, justice, freedom, cooperation and amity with all nations.”

The fact that the international law has been made part of the law of the land does not imply the primacy of
international law over national or municipal law in the municipal sphere. Rules of international law are given an equal
standing with, but not superior to, the national legislative enactment. The principle of “Lex Posterior Derogat Priori”
clarifies that a treaty may repeal a statute and a statute may repeal a treaty. And the Republic of the Philippines
considers its Constitution as the highest law of the land, therefore, both statutes and treaty may be invalidated if they
are conflict with the constitution.

Case no. 066


CONSTI I - CHAPTER 5
CASE DIGESTS
In Re Garcia
August 15, 1961
JING

In Re: Petition of Arturo Efren Garcia for Admission to the Philippine Bar Without Taking the Examination.
Arturo Efren Garcia - Petitioner.

FACTS:
Arturo E. Garcia has applied for admission to the practice of law in the Philippines without submitting to the required
bar examinations. In his verified petition, he avers, among others, that he is a Filipino citizen born in Bacolod City,
Province of Negros Occidental, of Filipino parentage; that he had taken and finished in Spain, the course of
"Bachillerato Superior"; that he was approved, selected and qualified by the "Instituto de Cervantes" for admission to
the Central University of Madrid where he studied and finished the law course graduating there as "Licenciado En
Derecho"; that thereafter he was allowed to practice the law profession in Spain; and that under the provisions of the
Treaty on Academic Degrees and the Exercise of Professions between the Republic of the Philippines and the
Spanish state, he is entitled to practice the law profession in the Philippines without submitting to the required bar
examinations.

ISSUE:
Whether or not the petitioner is entitled to practice the law profession in the Philippines without submitting to the
required bar examination as per Treaty on Academic Degrees

HELD:
The Court resolved to deny the petition on the following ground:
(1) The provisions of the Treaty on Academic Degrees and the Exercise of Professions between the Republic of the
Philippines and the Spanish State can not be invoked by applicant. Under Article 11 thereof;
"The Nationals of each of the two countries who shall have obtained recognition of the validity of their academic
degrees by virtue of the stipulations of this Treaty, can practice their professions within the territory of the Other”
from which it could clearly be discerned that said Treaty was intended to govern Filipino citizens desiring to practice
their profession in Spain, and the citizens of Spain desiring to practice their professions in the Philippines. Applicant is
a Filipino citizen desiring to practice the legal profession in the Philippines. He is therefore subject to the laws of his
own country and is not entitled to the privileges extended to Spanish nationals desiring to practice in the Philippines.

Section 3

SECTION 3. Civilian authority is, at all times, supreme over the military. The Armed Forces of the
Philippines is the protector of the people and the State. Its goal is to secure the sovereignty of the State and the
integrity of the national territory.

Case no. 067


Alih v. Castro
G.R. No. L-69401, June 23, 1987
JING

FACTS:

Rizal Alih, et. al. – petitioners were suspected of the Climaco killing
Major General Delfin C. Castro - Commander Southcom and Regional Unified Command, Region IX, Zamboanga
City

On November 25, 1984, a contingent of more than two hundred Philippine marines and elements of the home
defense forces raided the compound occupied by the petitioners at Gov. Alvarez street, Zamboanga City, in search of
loose firearms, ammunition and other explosives.
CONSTI I - CHAPTER 5
CASE DIGESTS
The military operation was commonly known and dreaded as a "zona," which was not unlike the feared practice of the
kempeitai during the Japanese Occupation of rounding up the people in a locality, arresting the persons fingered by a
hooded informer, and executing them outright

Respondents who were members of the Philippine marine and defense forces raided the compound occupied by
petitioner in search of loose firearms, ammunitions and explosives. A shoot-out ensued after petitioners resisted the
intrusion by the respondents, killing a number of men. The following morning, the petitioners were arrested and
subjected to finger –printing, paraffin testing and photographing despite their objection. Several kinds of rifle,
grenades and ammunitions were also confiscated.

On December 21, 1984, the petitioners came to this Court in a petition for prohibition and mandamus with preliminary
injunction and restraining order. Their purpose was to recover the articles seized from them, to prevent these from
being used as evidence against them, and to challenge their finger-printing, photographing and paraffin-testing as
violative of their right against self-incrimination.

The petitioners demand the return of the arms and ammunition on the ground that they were taken without a
search warrant as required by the Bill of Rights. This is confirmed by the said report and in fact admitted by the
respondents, "but with avoidance."

The record does not disclose that the petitioners were wanted criminals or fugitives from justice. At the time of the
"zona," they were merely suspected of the mayor's slaying and had not in fact even been investigated for it. As mere
suspects, they were presumed innocent and not guilty as summarily pronounced by the military.

The Court, treating the petition as an injunction suit with a prayer for the return of the articles alleged to have been
illegally seized, referred it for hearing to Judge Omar U. Amin of the regional trial court, Zamboanga City. After
receiving the testimonial and documentary evidence of the parties, he submitted the report and recommendations on
which this opinion is based.

ISSUE:
Whether or not the finger-printing, photographing and paraffin-test is protected by the constitutional right against self-
incrimination.

HELD:
WHEREFORE, the search of the petitioners' premises on November 25, 1984, is hereby declared ILLEGAL and all
the articles seized as a result thereof are inadmissible in evidence against the petitioners in any proceedings.
However, the said articles shall remain in custodia legis pending the outcome of the criminal cases that have been or
may later be filed against the petitioners.

Superior orders cannot countermand the Constitution. There is no excuse for the constitutional shortcuts done by the
military. Also, the aggravation of peace and order problem in their place does not excuse the non-observance of the
constitutional guaranty against unreasonable searches and seizure (Art III Sec. 2, 1973 Philippine Constitution).
The arrest does not fall also under the warrantless arrest provided for by Rule 113 Sec. 5 of the Rules of Court.
Therefore, all the firearms and ammunition taken from the raided compound are inadmissible in evidence in any
proceedings against petitioners. With respect to the finger-printing, photographing and paraffin-testing, the acts are
not covered by the protection against self-incrimination, for it only applies to testimonial compulsion.

Case no. 068


IBP v. Zamora
G.R. No. 141284, Aug. 15, 2000
JING

FACTS:
The President of the Philippines, Joseph Ejercito Estrada, in a verbal directive, ordered the PNP and the Marines to
conduct joint visibility patrols for the purpose of crime prevention and suppression. In compliance with the presidential
mandate, the PNP Chief, through Police Chief Superintendent Edgar B. Aglipay, formulated Letter of Instruction
02/2000 (the "LOI") which detailed the manner by which the joint visibility patrols, called Task Force Tulungan, would
be conducted.
CONSTI I - CHAPTER 5
CASE DIGESTS
Task Force Tulungan was placed under the leadership of the Police Chief of Metro Manila. Invoking his powers as
Commander-in-Chief under Section 18, Article VII of the Constitution, the President directed the AFP Chief of Staff
and PNP Chief to coordinate with each other for the proper deployment and utilization of the Marines to assist the
PNP in preventing or suppressing criminal or lawless violence. The President also declared that the services of the
Marines in the anti-crime campaign are merely temporary in nature and for a reasonable period only, until such time
when the situation shall have improved.

The Integrated Bar of the Philippines (the "IBP") filed the instant petition to annul LOI 02/2000 and to declare
the deployment of the Philippine Marines null and void and unconstitutional, arguing that the deployment of marines in
Metro Manila is violative of the Constitution because no emergency situation obtains in Metro Manila as would justify,
even only remotely, the deployment of soldiers for law enforcement work; hence, said deployment in derogation of
Article II, Section 3 of the Constitution.

ISSUE:

Whether or not the calling of the armed forces to assist the PNP in joint visibility patrols violates the constitutional
provisions on civilian supremacy over the military and the civilian character of the PNP

HELD:

The Supreme Court found no merit in the petition. When the President calls the armed forces to prevent or suppress
lawless violence, invasion or rebellion, he necessarily exercises a discretionary power solely vested in his wisdom.
This is clear from the intent of the framers and from the text of the Constitution itself.

The Court, thus, cannot be called upon to overrule the President's wisdom or substitute its own. It does not, however,
prevent an examination of whether such power was exercised within permissible constitutional limits or whether it was
exercised in a manner constituting grave abuse of discretion. In view of the constitutional intent to give the President
full discretionary power to determine the necessity of calling out the armed forces, it is incumbent upon the petitioner
to show that the President's decision is totally bereft of factual basis.

The petition failed to discharge such heavy burden as there was no evidence to support the assertion that there exists
no justification for calling out the armed forces nor was grave abuse committed because the power to call was
exercised in such a manner as to violate the constitutional provision on civilian supremacy over the military. In the
performance of the Court's duty of "purposeful hesitation" before declaring an act of another branch as
unconstitutional, only where such grave abuse of discretion is clearly shown shall the Court interfere with the
President's judgment and to doubt is to sustain.

The Court also ruled that the calling of the Marines in this case constitutes permissible use of military assets for
civilian law enforcement. The participation of the Marines in the conduct of joint visibility patrols is appropriately
circumscribed. The limited participation of the Marines is evident in the provisions of the LOI itself, which sufficiently
provides the metes and bounds of the Marines' authority. It is noteworthy that the local police forces are the ones in
charge of the visibility patrols at all times, the real authority belonging to the PNP.

Under the LOI, the police forces are tasked to brief or orient the soldiers on police patrol procedures. It is their
responsibility to direct and manage the deployment of the Marines. It is, likewise, their duty to provide the necessary
equipment to the Marines and render logistical support to these soldiers. It cannot be properly argued then that
military authority is supreme over civilian authority. Moreover, the deployment of the Marines to assist the PNP does
not unmake the civilian character of the police force. Neither does it amount to an "insidious incursion" of the military
in the task of law enforcement in violation of Section 5(4), Article XVI of the Constitution.

Case no. 069


Olaguer vs. Military Commission No. 34
G.R. No. L-54558 May 22, 1987
Lt. Col. Atty. Dana Ruedas, MNSA., MBA., MA., Phd., LLM., JSD.
Note: it is quite long but try to appreciate it since it is a nice case :)
FACTS:
Criminal Case No. MC-34-1 – the petitioners (all civilians) were charged for subversion upon the
recommendation of respondent Judge Advocate General and approval of the respondent Minister of National
Defense.
CONSTI I - CHAPTER 5
CASE DIGESTS
On June 13, 1980, the respondent Chief of Staff of the Armed Forces of the Philippines 3 created the
respondent Military Commission No. 34 to try the criminal case filed against the petitioners. 4 On July 30, 1980, an
amended charge sheet was filed for seven (7) offenses:
1) unlawful possession of explosives and incendiary devices; (2) conspiracy to assassinate President and Mrs. Marcos; (3) conspiracy to
assassinate cabinet members Juan Ponce Enrile, Francisco Tatad and Vicente Paterno; (4) conspiracy to assassinate Messrs. Arturo
Tangco, Jose Roño and Onofre Corpus; (5) arson of nine buildings; (6) attempted murder of Messrs. Leonardo Perez, Teodoro Valencia
and Generals Romeo Espino and Fabian Ver; and (7) conspiracy and proposal to commit rebellion, and inciting to rebellion. 5 Sometime
thereafter, trial ensued.

First Petition:
Petition for prohibition and habeas corpus to enjoin the respondent Military Commission No. 34 from proceeding with
the trial of their release from detention by way of a writ of habeas corpus. The thrust of their arguments is that military
commissions have no jurisdiction to try civilians for offenses alleged to have been committed during the period of
martial law. They also maintain that the proceedings before the respondent Military Commission No. 34 are in gross
violation of their constitutional right to due process of law.

On December 4, 1984, pending the resolution of the Petition, the respondent Military Commission No. 34
passed sentence convicting the petitioners and imposed upon them the penalty of death by electrocution.

Second Petition:
Thus, on February 14, 1985, petitioners Olaguer, Maclang and Othoniel and Ester Jimenez went to this Court
and filed the other instant Petition, this time for habeas corpus, certiorari, prohibition and mandamus. They also
sought the issuance of a writ of preliminary injunction. They seek to enjoin the said respondents from taking any
further action on the case against the petitioners, and from implementing the judgment of conviction rendered by the
respondent Military Commission No. 34 for the reason that the same is null and void. The petitioners also seek the
return of all property taken from them by the respondents concerned.

On September 12, 1985, this Court issued a temporary restraining order enjoining the respondents from
executing the Decision of the respondent Military Commission No. 34.

SUPERVENING EVENTS WHICH HAVE OCCURED HITHERTO THE TWO PETITIONS:


1. Proclamation No. 2045 - officially lifting Martial Law in the Philippines. Revoked General Order No. 8 (created
the military tribunals) and directed that all military tribunals created pursuant thereto are dissolved upon final
determination of cases pending which may not be transferred to the civil courts without irreparable prejudice
to the state in view of the rules on double jeopardy, or other circumstances which render prosecution of the
cases difficult, if not impossible."
2. Petitioner Ester Misa-Jimenez was granted provisional liberty in January, 1981. On the other hand, petitioners
Eduardo Olaguer and Othoniel Jimenez obtained provisional liberty on January 23, 1986. 16 The rest of the
petitioners have been released sometime before or after President Corazon C. Aquino assumed office in
February, 1986.

ISSUES: Whether or not a military tribunal has the jurisdiction to try civilians while the civil courts are open and
functioning.

HELD:
1. Issue on Habeas Corpus – moot and academic because petitioners have been released from their
confinement in military detention centers.
2. Military Commissions or Tribunals Jurisdiction on civilians – the issue on their jurisdiction of military
commission or tribunals to try civilians for offenses allegedly committed before, and more particularly during a
period of martial law, as well as the other issues raised by the petitioners.

As held in Senator Benigno Aquino, Jr. v. Military Commission No. 2,


● The military commission has been lawfully constituted and validly vested with jurisdiction to hear cases against civilians,
including petitioner.
● Court has declared the proclamation of Martial Law valid and constitutional and its continuance is justified by the danger posed
to the public safety.
● The President of the Philippines (Marcos) necessarily possesses broad authority compatible with the imperative requirements
of emergency to preserve the safety of the nation in times of national peril.
CONSTI I - CHAPTER 5
CASE DIGESTS
○ General Order No. 8 - creating military tribunals to try and decide cases of military personnel and such other cases
as may be referred to them.
○ General Order No. 12 - military tribunals were vested with jurisdiction “exclusive of the civil courts”, among others,
over crimes against public order, violations of the Anti-Subversion Act, violations of the laws on firearms, and
other crimes which, in the face of the emergency, are directly related to the quelling of the rebellion and preservation
of the safety and security of the Republic.
● Aquino insists that he, being a civilian, in his trial by the military commission deprives him of his right to due process, since due
process granted by the Constitution to persons accused of ordinary crimes means judicial process.
○ This argument ignores the reality of rebellion and the existence of martial law. The martial law administrator must
have ample and sufficient means to quell rebellion and restore civil order. Prompt and effective trial and punishment
are necessary.
● The guarantee of due process is not a guarantee of any particular form of tribunal in criminal cases. A military tribunal of
competent jurisdiction, accusation in due form, notice and opportunity to defend and trial before an impartial tribunal,
adequately inset the due process requirement. Due process of law does not necessarily mean a judicial proceeding in the
regular courts.

The petitioners pray that the ruling in Aquino, Jr. be appraised anew and abandoned or modified accordingly,
therefore:
● Due process of law demands that in all criminal prosecutions (where the accused stands to lose either his life
or his liberty), the accused shall be entitled to, among others, a trial. The trial contemplated by the due
process clause of the Constitution, in relation to the Charter as a whole, is a trial by judicial process, not by
executive or military process. Military commissions or tribunals, by whatever name they are called, are not
courts within the Philippine judicial system.
● Military tribunals pertain to the Executive Department and are simply instrumentalities of the executive power.
In the principle of separation of powers, the power and duty of interpreting of laws is primarily a function of the
judiciary. NOT and CANNOT be the function of the executive department through the military authorities. As
long as the civil courts in the land remain open and are regularly functioning, as they do so today and as they
did during the period of martial law in the country, military tribunals cannot try and exercise jurisdiction over
civilians for offenses committed by them and which are properly cognizable by the civil courts. To have it
otherwise would be a violation of the constitutional right to due process of the civilian concerned.
● Military commissions or tribunals have no jurisdiction to try civilians for alleged offenses when the civil courts
are open and functioning. As long as the civil courts in the land are open and functioning, military tribunals
cannot try and exercise jurisdiction over civilians for offenses committed by them. Whether or not martial law
has been proclaimed throughout the country or over a part thereof is of no moment. The exclusive judicial
power of civil courts may no longer be appropriated by any military body or tribunal.
Section 18, Article VII of the 1987 Constitution,
"A state of martial law does not suspend the operation of the Constitution, nor supplant the
functioning of the civil courts or legislative assemblies, nor authorize the conferment of jurisdiction on
military courts and agencies over civilians where civil courts are able to function, nor automatically
suspend the privilege of the writ.”
● Proclamation No. 2045 (dated January 17, 1981) officially lifting martial law in the Philippines and abolishing
all military tribunals created pursuant to the national emergency effectively divests the respondent Military
Commission No. 34 (and all military tribunals for that matter) of its supposed authority to try civilians, including
the herein petitioners.
○ All cases against civilians pending therein should eventually be transferred to civil courts.
○ The main opinion in Aquino, Jr. is premised on the theory that military tribunals have the jurisdiction to try civilians as long as
the period of national emergency (brought about by public disorder and similar causes) lasts. Undoubtedly, Proclamation No.
2045 is an acknowledgment on the part of the Executive Department of the Government that the national emergency no longer
exists. Thereafter, following the theory relied upon in the main opinion, all military tribunals should henceforth be considered
functus officio in their relationship with civilians.
● The rule of stare decisis is entitled to respect because stability in jurisprudence is desirable. Nonetheless,
reverence for precedent, simply as precedent, cannot prevail when constitutionalism and the public interest
demand otherwise. Thus, a doctrine which should be abandoned or modified should be abandoned or
modified accordingly. After all, more important than anything else is that this Court should be right.
○ Decision by Military Commission No. 34 is null and void for lack of jurisdiction.

Section 4
SECTION 4. The prime duty of the Government is to serve and protect the people. The Government may
CONSTI I - CHAPTER 5
CASE DIGESTS
call upon the people to defend the State and, in the fulfillment thereof, all citizens may be required, under conditions
provided by law, to render personal military or civil service.

Case no. 070


People vs. Lagman
G.R. No. 45892 July 13, 1938
Dana

FACTS:
The appellants Tranquilino Lagman and Primitivo de Sosa are charged with a violation of section 60 of
Commonwealth Act No. 1, known as the National Defense Law. It is alleged that these two appellants, being
Filipinos and having reached the age of twenty years in 1936, willfully and unlawfully refused to register in the
military service between the 1st and 7th of April of said year, notwithstanding the fact that they had been required to
do so. The evidence shows that these two appellants were duly notified by the corresponding authorities to appear
before the Acceptance Board in order to register for military service in accordance with law, and that the said
appellants, in spite of these notices, had not registered up to the date of filing of the information. Both appellants do
not deny the facts.

They allege that:


Primitivo de Sosa is fatherless and has a mother and a brother eight years old to support.
Tranquilino Lagman also has a father to support, has no military leanings, and does not wish to kill or be killed.

Each of them were sentenced by the Court of First Instance to one month and one day of imprisonment and they
impugn the constitutionality of the National Defense Law.

ISSUE: Whether or not the National Defense Law is unconstitutional.

HELD:
At this time, 1935 Constitution was still in force hence the provision is:
"SEC. 2. The defense of the State is a prime duty of government, and in the fulfillment of this duty all citizens
may be required by law to render personal military or civil service."
● The National Defense Law is not unconstitutional. It is, in faithful compliance of the provision.
● The duty of the Government to defend the State cannot be performed except through an army. To leave the
organization of an army to the will of the citizens would be to make this duty of the Government excusable
should there be no sufficient men who volunteer to enlist therein.
● US Courts held:
○ Compulsory military service does not violate the Constitution because the power to establish it is
derived from that granted to Congress to declare war and to organize and maintain an army.
○ The right of the Government to require compulsory military service is a consequence of its duty to
defend the State and is reciprocal with its duty to defend the life, liberty, and property of the citizen.
● The circumstance that the appellants have dependent families to support does not excuse them from their
duty to present themselves before the Acceptance Board because, if such circumstance exists, they can ask
for deferment in complying with their duty and, at all events, they can obtain the proper pecuniary allowance
to attend to these family responsibilities (secs. 65 and 69 of Commonwealth Act No. 1).

Section 6
SECTION 6. The separation of Church and State shall be inviolable.

Case no. 071


Aglipay vs. Ruiz
G.R. No. L-45459 May 13, 1937
Dana
FACTS:
CONSTI I - CHAPTER 5
CASE DIGESTS
The petitioner, Mons. Gregorio Aglipay, Supreme Head of the Philippine Independent Church, seeks the
issuance from this court of a writ of prohibition to prevent the respondent Director of Posts from issuing and selling
postage stamps commemorative of the Thirty-third International Eucharistic Congress.
● The 33rd Intl. Eucharistic Congress is organized by the Roman Catholic Church.
● The Solicitor General contends that a writ of prohibition is not the proper legal remedy although he admits that
the writ may properly restrain ministerial functions.
○ While, generally, prohibition as an extraordinary legal writ will not issue to restrain or control the performance of other than
judicial or quasi-judicial functions (50 C. J., 658), its issuance and enforcement are regulated by statute and in this jurisdiction
may issue to ". . . inferior tribunals, corporations, boards, or persons, whether exercising functions judicial or ministerial, which
are without or in excess of the jurisdiction of such tribunal, corporation, board, or person . . .."
● The terms "judicial" and "ministerial" used with reference to "functions" in the statute are undoubtedly
comprehensive and include the challenged act of the respondent Director of Posts in the present case, which
act because alleged to be violative of the Constitution is a fortiori "without or in excess of . . . jurisdiction."
● The statutory rule, therefore, in this jurisdiction is that the writ of prohibition is not confined exclusively to
courts or tribunals to keep them within the limits of their own jurisdiction and to prevent them from
encroaching upon the jurisdiction of other tribunals but will issue, in appropriate cases, to an other or person
whose acts are without or in excess of his authority. Not infrequently, "the writ is granted, where it is
necessary for the orderly administration of justice, or to prevent the use of the strong arm of the law in an
oppressive or vindictive manner, or a multiplicity of actions"

ISSUES: Whether or not the issuing and selling postage stamps commemorative of the 33rd International Eucharistic
Congress is violative of Section 13, Article VI, of the Constitution of the Philippines: "No public money or property shall
ever be appropriated, applied, or used, directly or indirectly, for the use, benefit, or support of any sect, church,
denomination, sectarian institution, or system of religion, or for the use, benefit, or support of any priest, preacher,
minister, or other religious teacher or dignitary as such, except when such priest, preacher, minister, or dignitary is
assigned to the armed forces or to any penal institution, orphanage, or leprosarium."

HELD:
● The prohibition herein is a direct corollary to the principle of separation of church and state.
○ Without the necessity of adverting to the historical background of this principle in our country, it is sufficient to say that our
history, not to speak of the history of mankind, has taught us that the union of church and state is prejudicial to both, for
occasions might arise when the state will use the church, and the church the state, as a weapon in the furtherance of their
respective ends and aims.
● In the case at bar, it appears that the respondent Director of Posts issued the postage stamps in question
under the provisions of Act. No. 4052 of the Philippine Legislature.
○ Act No. 4052. — AN ACT APPROPRIATING THE SUM OF SIXTY THOUSAND PESOS AND MAKING THE SAME
AVAILABLE OUT OF ANY FUNDS IN THE INSULAR TREASURY NOT OTHERWISE APPROPRIATED FOR THE COST OF
PLATES AND PRINTING OF POSTAGE STAMPS WITH NEW DESIGNS, AND FOR OTHER PURPOSES.
● The Act appropriate the sum of sixty thousand pesos for the cost of plates and printing of postage stamps with
new designs and other expenses incident thereto, and authorizes the Director of Posts, with the approval of
the Secretary of Public Works and Communications, to dispose of the amount appropriated in the manner
indicated and "as often as may be deemed advantageous to the Government". The printing and issuance of
the postage stamps in question appears to have been approved by authority of the President of the
Philippines in a letter dated September 1, 1936, made part of the respondent's memorandum as Exhibit A.
The respondent alleges that the Government of the Philippines would suffer losses if the writ prayed for is
granted. He estimates the revenue to be derived from the sale of the postage stamps in question at
P1,618,179.10 and states that there still remain to be sold stamps worth P1,402,279.02.
● Act No. 4052 contemplates no religious purpose in view. What it gives the Director of Posts is the
discretionary power to determine when the issuance of special postage stamps would be "advantageous to
the Government." Of course, the phrase "advantageous to the Government" does not authorize the violation
of the Constitution. It does not authorize the appropriation, use or application of public money or property for
the use, benefit or support of a particular sect or church.
○ In the present case, however, the issuance of the postage stamps in question by the Director of Posts
and the Secretary of Public Works and Communications was not inspired by any sectarian feeling to
favor a particular church or religious denominations. The stamps were not issued and sold for the
benefit of the Roman Catholic Church. Nor were money derived from the sale of the stamps given to
that church.
CONSTI I - CHAPTER 5
CASE DIGESTS
■ It was to "to advertise the Philippines and attract more tourists to this country." and "to give
publicity to the Philippines and its people"
● It is significant to note that the stamps as actually designed and printed, instead of showing a Catholic Church
chalice as originally planned, contains a map of the Philippines and the location of the City of Manila, and an
inscription as follows: "Seat XXXIII International Eucharistic Congress, Feb. 3-7, 1937." What is emphasized
is not the Eucharistic Congress itself but Manila, the capital of the Philippines, as the seat of that congress.
● We are much impressed with the vehement appeal of counsel for the petitioner to maintain inviolate the
complete separation of church and state and curb any attempt to infringe by indirection a constitutional
inhibition. Indeed, in the Philippines, once the scene of religious intolerance and persecution, care should be
taken that at this stage of our political development nothing is done by the Government or its officials that may
lead to the belief that the Government is taking sides or favoring a particular religious sect or institution.
● The examination of Act No. 4052, and scrutiny of the attending circumstances, we have come to the
conclusion that there has been no constitutional infraction in the case at bar. Act. No. 4052 grants the Director
of Posts, with the approval of the Secretary of Public Works and Communications, discretion to issue postage
stamps with new designs "as often as may be deemed advantageous to the Government. "Even if we were to
assume that these oHcials made use of a poor judgment in issuing and selling the postage stamps in question
still, the case of the petitioner would fail to take in weight. Between the exercise of a poor judgment and the
unconstitutionality of the step taken, a gap exists which is yet to be filled to justify the court in setting aside the
oHcial act assailed as coming within a constitutional inhibition.

Case no. 072


Garces vs. Estenzo
G.R. No. L-53487 May 25, 1981
Dana

SYNOPSIS:
A wooden image of San Vicente Ferrer was acquired by the barangay council with funds raised by means of solicitations and cash donations
pursuant to Resolution No. 5 of said council, duly ratified by the barangay assembly in a plebiscite, reviving the traditional socioreligious celebration
of the feast day of the saint. The image was brought to the Catholic parish church during the saint's feast day as per Resolution No. 6 which also
designated the hermano mayor as the custodian of the image. After the fiesta, however, petitioner parish priest refused to return custody of the
image to the council until after the latter, by resolution, filed a replevin case against the priest and posted the required bond. The parish priest and
his co-petitioners thereafter filed an action for annulment of the council's resolutions relating to the subject image contending that when they were
adopted, the barangay council was not duly constituted because the chairman of the Kabataang Barangay was not allowed to participate; and that
they contravened the constitutional provisions on separation of church and state. freedom of religion and the use of public money to favor any sect
or church. The lower court dismissed the complaint and upheld the validity of the resolution. On petition for review, the Supreme Court held, that the
absence of the Kabataang Barangay chairman, despite due notice from the sessions of the barangay council, did not render the resolutions then
adopted void since there was a quorum; and that the questioned resolutions did not contravene any constitutional provision since the image was
purchased with private funds, not with tax money, and in connection with a socioreligious affair, the celebration of which is an ingrained tradition in
rural communities. Judgment of the lower court affirmed.

FACTS:
● Resolution No. 5 - the barangay council adopted this “reviving the traditional socio-religious celebration” every
fifth day of April "of the feast day of Señor San Vicente Ferrer, the patron saint of Valencia.”
○ 9 committees who would take charge of the 1976 festivities
○ It provided for (1) the acquisition of the image of San Vicente Ferrer and (2) the construction of a
waiting shed as the barangay's projects. Funds for the two projects would be obtained through the
"selling of tickets and cash donations"
● Resolution No. 6 - in accordance with the practice in Eastern Leyte, Councilman Tomas Cabatingan, the
chairman or hermano mayor of the fiesta, would be the caretaker of the image of San Vicente Ferrer and that
the image would remain in his residence for one year and until the election of his successor as chairman of
the next feast day. The image would be made available to the Catholic parish church during the celebration of
the saint's feast day.
● Funds were raised by means of solicitations and cash donations of the barangay residents and those of the
neighboring places of Valencia. With those funds, the waiting shed was constructed and the wooden image of
San Vicente Ferrer was acquired in Cebu City by the barangay council for four hundred pesos.
CONSTI I - CHAPTER 5
CASE DIGESTS
● April 5, 1976 - the image was temporarily placed in the altar of the Catholic Church of Barangay Valencia so
that the devotees could worship the saint during the mass for the fiesta.
○ After the mass, Father Sergio Marilao Osmeña refused to return the image to the barangay council on
the pretext that it was the property of the church because church funds were used for its acquisition.
● The image of San Vicente Ferrer remained in the Catholic Church of Valencia because Father Osmenña did
not accede to the request of Cabatingan to have custody of the image and “maliciously ignored” Resolution
No. 6.
● The replevin case was filed in the city court of Ormoc City against Father Osmeña and Bishop Cipriano Urgel.
After the barangay council had posted a cash bond of eight hundred pesos, Father Osmeña turned over the
image to the council. In his answer to the complaint for replevin, he assailed the constitutionality of the said
resolutions.
● Later, he and three other persons, Andres Garces, a member of the Aglipayan Church, and two Catholic
laymen, Jesus Edullantes and Nicetas Dagar, filed against the barangay council and its members (excluding
two members) a complaint in the Court of First Instance at Ormoc City, praying for the annulment of the said
resolutions (Civil Case No. 1680-0).
○ The lower court dismissed the complaint. It upheld the validity of the resolutions. The petitioners
appealed under Republic Act No. 5440.
■ The petitioners contend that the barangay council was not duly constituted because Isidoro
M. Mañago, Jr., the chairman of the kabataang barangay, was not allowed to participate in its
sessions.
● Mañago was duly notified of the sessions but was not able to attend because he was
working with a construction company.
○ His absence did not render the resolutions void. There was a quorum when
the said resolutions were passed.
■ Petitioners contention that the resolutions contravene the constitutional provisions of
separation of church and state: Devoid of merit.
ISSUE: Whether or not the constitutional provisions on separation of church and state were violated.

HELD:
● Resolution No. 5 does not directly or indirectly establish any religion, nor abridge religious liberty, nor
appropriate money for the benefit of any sect, priest or clergyman. The image was purchased with private
funds, not with tax money. The construction of the waiting shed is entirely a secular matter.
● The wooden image was purchased in connection with the celebration of the barrio fiesta honoring the patron
saint, San Vicente Ferrer, and not for the purpose of favoring any religion nor interfering with religious beliefs
of the barrio residents. One of the highlights of the fiesta was the mass. Consequently, the image of the
patron saint had to be placed in the church when the mass was celebrated.
● If there is nothing unconstitutional or illegal in holding a fiesta and having a patron saint for the barrio, then
any activity intended to facilitate the worship of the patron saint (such as the acquisition and display of his
image) cannot be branded as illegal. As noted in the resolution, the barrio fiesta is a socio-religious affair. Its
celebration is an ingrained tradition in rural communities. The fiesta relieves the monotony and drudgery of
the lives of the masses.
● Resolution No. 5 and 6 does not involve at all, even remotely or indirectly, the momentous issues of
separation of church and state, freedom of religion and the use of public money to favor any sect or church,
contrary to the contradictory positions of the petitioners — petitioner Garces swearing that the said resolutions
favored the Catholic Church, and petitioners Dagar and Edullantes swearing that the resolutions prejudiced
the Catholics because they could see the image in the church only once a year during the fiesta. There can
be no question that the image in question belongs to the barangay council.
● Father Osmeña's claim that it belongs to the church is wrong. The barangay council, as owner of the image,
has the right to determine who should have custody thereof. The barangay council designated a layman as
the custodian of the wooden image in order to forestall any suspicion that it is favoring the Catholic church. A
more practical reason for that arrangement would be that the image, if placed in a layman's custody, could
easily be made available to any family desiring to borrow the image in connection with prayers and novenas.
● If the council chooses to change its mind and decides to give the image to the Catholic church, that action
would not violate the Constitution because the image was acquired with private funds and is its private
property.
CONSTI I - CHAPTER 5
CASE DIGESTS
● The barangay council of Valencia has the right to take measures to recover possession of the image of San
Vicente Ferrer, which is its private property, from the parish priest of Valenzuela by enacting Resolution No.
10, authorizing the hiring of a lawyer to file a replevin case against Father Osmeña for the recovery of the
image, as well as Resolution No. 12, appointing Veloso as its representative in the replevin case.
● Not every governmental activity which involves the expenditure of public funds and which has some religious
tint is violative of the constitutional provisions regarding separation of church and state, freedom of worship
and banning the use of public money or property. (Sec Aglipay vs. Ruiz, 64 Phil. 201)

Case no. 073


Estrada vs. Escritor
A.M. P-02-1651 June 22, 2006
Dana
Huhu super hirap ng case na to. It is super long pero kasi I have to discuss the past

FACTS:
In a sworn-letter complaint dated July 27, 2000, complainant Alejandro Estrada requested Judge Jose F.
Caoibes, Jr., presiding judge of Branch 253, Regional Trial Court of Las Piñas City, for an investigation of respondent
Soledad Escritor, court interpreter in said court, for living with a man not her husband, and having borne a child within
this live-in arrangement. Estrada believes that Escritor is committing an immoral act that tarnishes the image of the
court, thus she should not be allowed to remain employed therein as it might appear that the court condones her
act.Consequently, respondent was charged with committing disgraceful and immoral conduct under Book V, Title I,
Chapter VI, Sec. 46(b)(5) of the Revised Administrative Code.

Respondent Escritor testified that when she entered the judiciary in 1999, she was already a widow, her
husband having died in 1998. She admitted that she started living with Luciano Quilapio, Jr. without the benefit of
marriage more than twenty years ago when her husband was still alive but living with another woman. She also
admitted that she and Quilapio have a son. But as a member of the religious sect known as the Jehovah's Witnesses
and the Watchtower and Bible Tract Society, respondent asserted that their conjugal arrangement is in conformity
with their religious beliefs and has the approval of her congregation. In fact, after ten years of living together, she
executed on July 28, 1991, a Declaration of Pledging Faithfulness.

For Jehovah’s Witnesses, the Declaration allows members of the congregation who have been abandoned
by their spouses to enter into marital relations. The Declaration thus makes the resulting union moral and binding
within the congregation all over the world except in countries where divorce is allowed. As laid out by the tenets of
their faith, the Jehovah’s congregation requires that at the time the declarations are executed, the couple cannot
secure the civil authorities approval of the marital relationship because of legal impediments. Escritor and Quilapios
declarations were executed in the usual and approved form prescribed by the Jehovah’s Witnesses, approved by
elders of the congregation where the declarations were executed, and recorded in the Watchtower Central Office.

Moreover, the Jehovah’s congregation believes that once all legal impediments for the couple are lifted, the
validity of the declarations ceases, and the couple should legalize their union. In Escritors case, although she was
widowed in 1998, thereby lifting the legal impediment to marry on her part, her mate was still not capacitated to
remarry. Thus, their declarations remained valid. In sum, therefore, insofar as the congregation is concerned, there is
nothing immoral about the conjugal arrangement between Escritor and Quilapio and they remain members in good
standing in the congregation.

By invoking the religious beliefs, practices and moral standards of her congregation, in asserting that her
conjugal arrangement does not constitute disgraceful and immoral conduct for which she should be held
administratively liable, the Court had to determine the contours of religious freedom under Article III, Section 5 of the
Constitution, which provides, viz:

Sec. 5. No law shall be made respecting an establishment of religion, or prohibiting the free exercise thereof.
The free exercise and enjoyment of religious profession and worship, without discrimination or preference,
shall forever be allowed. No religious test shall be required for the exercise of civil or political rights.
CONSTI I - CHAPTER 5
CASE DIGESTS
PAST RULING:
In our decision dated August 4, 2003, after a long and arduous scrutiny into the origins and development of
the religion clauses in the United States (U.S.) and the Philippines, we held that in resolving claims involving religious
freedom (1) benevolent neutrality or accommodation, whether mandatory or permissive, is the spirit, intent and
framework underlying the religion clauses in our Constitution; and (2) in deciding respondents plea of exemption
based on the Free Exercise Clause (from the law with which she is administratively charged), it is the compelling
state interest test, the strictest test, which must be applied.

ISSUES: Whether or not respondent should be found guilty of the administrative charge of gross and immoral
conduct. To resolve this issue, it is necessary to determine the sub-issue of whether or not respondents right to
religious freedom should carve out an exception from the prevailing jurisprudence on illicit relations for which
government employees are held administratively liable.

HELD:

U.S. history has produced two identifiably different, even opposing, strains of jurisprudence on the religion clauses. First is the standard of
separation, which may take the form of either (a) strict separation or (b) the tamer version of strict neutrality or separation, or what Mr. Justice
Carpio refers to as the second theory of governmental neutrality. Although the latter form is not as hostile to religion as the former, both are
anchored on the Jeffersonian premise that a wall of separation must exist between the state and the Church to protect the state from the church.
Both protect the principle of church-state separation with a rigid reading of the principle. On the other hand, the second standard, the benevolent
neutrality or accommodation, is buttressed by the view that the wall of separation is meant to protect the church from the state
1. The Strict Separationist believes that the Establishment Clause was meant to protect the state from the church, and the states hostility
towards religion allows no interaction between the two. Religious institutions could not receive aid, whether direct or indirect, from the
state. Nor could the state adjust its secular programs to alleviate burdens the programs placed on believers. Only the complete
separation of religion from politics would eliminate the formal influence of religious institutions and provide for a free choice among
political views, thus a strict wall of separation is necessary.
2. The tamer version of the strict separationist view, the strict neutrality or separationist view, (or, the governmental neutrality theory)
believes that the wall of separation does not require the state to be their adversary. Rather, the state must be neutral in its relations with
groups of religious believers and non-believers. State power is no more to be used so as to handicap religions than it is to favor them.The
strict neutrality approach is not hostile to religion, but it is strict in holding that religion may not be used as a basis for classification for
purposes of governmental action, whether the action confers rights or privileges or imposes duties or obligations. Only secular criteria
may be the basis of government action. It does not permit, much less require, accommodation of secular programs to religious belief.
3. The benevolent neutrality theory believes that with respect to these governmental actions, accommodation of religion may be
allowed, not to promote the governments favored form of religion, but to allow individuals and groups to exercise their religion
without hindrance. The purpose of accommodations is to remove a burden on, or facilitate the exercise of, a persons or
institutions religion. As Justice Brennan explained, the government [may] take religion into account to exempt, when possible,
from generally applicable governmental regulation individuals whose religious beliefs and practices would otherwise thereby
be infringed, or to create without state involvement an atmosphere in which voluntary religious exercise may flourish.
Accomodation under Religious Clauses:
A free exercise claim could result to three kinds of accommodation:
(a) those which are found to be constitutionally compelled, i.e., required by the Free Exercise Clause;
(b) those which are discretionary or legislative, i.e., not required by the Free Exercise Clause but nonetheless permitted by the Establishment
Clause; and
(c) those which the religion clauses prohibit.

1. Mandatory accommodation results when the Court finds that accommodation is required by the Free Exercise Clause, i.e, when the
Court itself carves out an exemption. This accommodation occurs when all three conditions of the compelling interest test are met.
2. In permissive accommodation, the Court finds that the State may, but is not required to, accommodate religious interests.
3. When the Court finds no basis for a mandatory accommodation, or it determines that the legislative accommodation runs afoul of the
establishment or the free exercise clause, it results to a prohibited accommodation. In this case, the Court finds that establishment
concerns prevail over potential accommodation interests.To say that there are valid exemptions buttressed by the Free Exercise Clause
does not mean that all claims for free exercise exemptions are valid.

Unlike in the U.S. where legislative exemptions of religion had to be upheld by the U.S. Supreme Court as constituting permissive accommodations,
similar exemptions for religion are mandatory accommodations under our own constitutions. Thus, our 1935, 1973 and 1987 Constitutions contain
provisions on tax exemption of church property, salary of religious officers in government institutions, and optional religious instruction. Our own
preamble also invokes the aid of a divine being. These constitutional provisions are wholly ours and have no counterpart in the U.S. Constitution or
its amendments. They all reveal without doubt that the Filipino people, in adopting these constitutions, manifested their adherence to the benevolent
neutrality approach that requires accommodations in interpreting the religion clauses.

There is no ambiguity with regard to the Philippine Constitutions departure from the U.S. Constitution, insofar as religious accommodations are
concerned. It is indubitable that benevolent neutrality-accommodation, whether mandatory or permissive, is the spirit, intent and framework
underlying the Philippine Constitution.

Our own Constitutions have made significant changes to accommodate and exempt religion. Philippine jurisprudence shows that the Court has
allowed exemptions from a law of general application, in effect, interpreting our religion clauses to cover both mandatory and permissive
accommodations.
CONSTI I - CHAPTER 5
CASE DIGESTS

Having established that benevolent neutrality-accommodation is the framework by which free exercise cases must be decided, the next question
then turned to the test that should be used in ascertaining the limits of the exercise of religious freedom. In our Decision dated August 4, 2003, we
reviewed our jurisprudence, and ruled that in cases involving purely conduct based on religious belief, as in the case at bar, the compelling state
interest test, is proper.

The Compelling State Test:


1. First, [H]as the statute or government action created a burden on the free exercise of religion? The courts often look into the sincerity of
the religious belief, but without inquiring into the truth of the belief because the Free Exercise Clause prohibits inquiring about its truth as
held in Ballard and Cantwell. The sincerity of the claimants belief is ascertained to avoid the mere claim of religious beliefs to escape a
mandatory regulation.
2. Second, the court asks: [I]s there a sufficiently compelling state interest to justify this infringement of religious liberty? In this step, the
government has to establish that its purposes are legitimate for the state and that they are compelling. Government must do more than
assert the objectives at risk if exemption is given; it must precisely show how and to what extent those objectives will be undermined if
exemptions are granted. xxx
3. Third, the court asks: [H]as the state in achieving its legitimate purposes used the least intrusive means possible so that the free exercise
is not infringed any more than necessary to achieve the legitimate goal of the state? The analysis requires the state to show that the
means in which it is achieving its legitimate state objective is the least intrusive means, i.e., it has chosen a way to achieve its legitimate
state end that imposes as little as possible on religious liberties

Again, the application of the compelling state interest test could result to three situations of accommodation: First, mandatory accommodation would
result if the Court finds that accommodation is required by the Free Exercise Clause. Second, if the Court finds that the State may, but is not
required to, accommodate religious interests, permissive accommodation results. Finally, if the Court finds that that establishment concerns prevail
over potential accommodation interests, then it must rule that the accommodation is prohibited.

First, in relation to criminal statutes, only the question of mandatory accommodation is uncertain, for Philippine law and jurisprudence have, in fact,
allowed legislative accommodation. Second, the power of the Courts to grant exemptions in general (i.e., finding that the Free Exercise Clause
required the accommodation, or mandatory accommodations) has already been decided, not just once, but twice by the Court. Thus, the crux of the
matter is whether this Court can make exemptions as in Ebralinag and the American Bible Society, in cases involving criminal laws of general
application.

On the final task of subjecting this case to the careful application of the compelling state interest test, i.e.,
determining whether respondent is entitled to exemption, an issue which is essentially factual or evidentiary
in nature.
● A look at the evidence that the OSG has presented fails to demonstrate the gravest abuses, endangering
paramount interests which could limit or override respondents fundamental right to religious freedom. Neither
did the government exert any effort to show that the means it seeks to achieve its legitimate state objective is
the least intrusive means.
● In its Memorandum-In-Intervention, the OSG contends that the State has a compelling interest to override
respondents claimed religious belief and practice, in order to protect marriage and the family as basic social
institutions. The Solicitor General, quoting the Constitution and the Family Code, argues that marriage and the
family are so crucial to the stability and peace of the nation that the conjugal arrangement embraced in the
Declaration of Pledging Faithfulness should not be recognized or given effect, as it is utterly destructive of the
avowed institutions of marriage and the family for it reduces to a mockery these legally exalted and socially
significant institutions which in their purity demand respect and dignity
● Be that as it may, the free exercise of religion is specifically articulated as one of the fundamental rights in our
Constitution. It is a fundamental right that enjoys a preferred position in the hierarchy of rights the most
inalienable and sacred of human rights, in the words of Jefferson. Hence, it is not enough to contend that the
states interest is important, because our Constitution itself holds the right to religious freedom sacred. The
State must articulate in specific terms the state interest involved in preventing the exemption, which must be
compelling, for only the gravest abuses, endangering paramount interests can limit the fundamental right to
religious freedom. To rule otherwise would be to emasculate the Free Exercise Clause as a source of right by
itself.
● Thus, it is not the States broad interest in protecting the institutions of marriage and the family, or even in the
sound administration of justice that must be weighed against respondents claim, but the States narrow
interest in refusing to make an exception for the cohabitation which respondents faith finds moral. In other
words, the government must do more than assert the objectives at risk if exemption is given; it must precisely
show how and to what extent those objectives will be undermined if exemptions are granted. This, the
Solicitor General failed to do.
● As previously discussed, our Constitution adheres to the benevolent neutrality approach that gives room for
accommodation of religious exercises as required by the Free Exercise Clause. Thus, in arguing that
respondent should be held administratively liable as the arrangement she had was illegal per se because, by
CONSTI I - CHAPTER 5
CASE DIGESTS
universally recognized standards, it is inherently or by its very nature bad, improper, immoral and contrary to
good conscience, the Solicitor General failed to appreciate that benevolent neutrality could allow for
accommodation of morality based on religion, provided it does not offend compelling state interests.
● Finally, even assuming that the OSG has proved a compelling state interest, it has to further demonstrate that
the state has used the least intrusive means possible so that the free exercise is not infringed any more than
necessary to achieve the legitimate goal of the state, i.e., it has chosen a way to achieve its legitimate state
end that imposes as little as possible on religious liberties. Again, the Solicitor General utterly failed to prove
this element of the test. Other than the two documents offered as cited above which established the sincerity
of respondents religious belief and the fact that the agreement was an internal arrangement within
respondents congregation, no iota of evidence was offered. In fact, the records are bereft of even a feeble
attempt to procure any such evidence to show that the means the state adopted in pursuing this compelling
interest is the least restrictive to respondents religious freedom.
● Thus, we find that in this particular case and under these distinct circumstances, respondent Escritors
conjugal arrangement cannot be penalized as she has made out a case for exemption from the law based on
her fundamental right to freedom of religion. The Court recognizes that state interests must be upheld in order
that freedoms - including religious freedom - may be enjoyed. In the area of religious exercise as a preferred
freedom, however, man stands accountable to an authority higher than the state, and so the state interest
sought to be upheld must be so compelling that its violation will erode the very fabric of the state that will also
protect the freedom. In the absence of a showing that such state interest exists, man must be allowed to
subscribe to the Infinite.

IN VIEW WHEREOF, the instant administrative complaint is DISMISSED.

ISAGANI CRUZ:
The court, in providing for the dismissal of charges of immorality against the respondent based on her having
cohabited with a person other than her husband upon finding that said arrangement was sanctioned under her
religion, invoked, among other principles, the so-called benevolent neutrality approach, which “gives room for
accommodation of religious exercises as required by Free Exercise Clause” and “for accomodation of morality based
on religion, provided it does not offend compelling state interests,” declaring that her “conjugal arrangement cannot be
penalized as she has made out a case for exemption from the law based on her fundamental right to freedom of
religion.

Case no. 074


Taruc vs. Bishop Dela Cruz
G.R. No. 044801 March 10, 2005
Dana
FACTS:
This is an appeal under Rule 45 of the Revised Rules of Court of the decision of the Court of Appeals in CA-G.R. SP No. 45480 which
reversed and set aside the decision of the Regional Trial Court of Surigao City, Branch 32 in Civil Case No. 4907 and ordered said case dismissed
for lack of jurisdiction. Petitioners were lay members of the Philippine Independent Church (PIC) in Socorro, Surigao del Norte. Respondents
Porfirio de la Cruz and Rustom Florano were the bishop and parish priest, respectively, of the same church in that locality.

Petitioners, led by Dominador Taruc, clamored for the transfer of Fr. Florano to another parish but Bishop de
la Cruz denied their request. It appears from the records that the family of Fr. Florano's wife belonged to a political
party opposed to petitioner Taruc's, thus the animosity between the two factions with Fr. Florano being identified with
his wife's political camp. Bishop de la Cruz, however, found this too flimsy a reason for transferring Fr. Florano to
another parish.
Meanwhile, Taruc tried to organize an open mass to be celebrated by a certain Fr. Renato Z. Ambong during
the town Aesta of Socorro. Bishop de la Cruz tried to stop Taruc’s plan but he still continued to carry out his plans. Fr.
Ambong was not a member of the clergy of the diocese of Surigao and his credentials (bishop de la cruz) as a parish
priest were in doubt. The hostility between members of PIC worsened because of this.

On June 28, 1993, Bishop de la Cruz declared petitioners expelled/excommunicated from the Philippine
Independent Church for reasons of:
(1) disobedience to duly constituted authority in the Church;
CONSTI I - CHAPTER 5
CASE DIGESTS
(2) inciting dissension, resulting in division in the Parish of Our Mother of Perpetual Help, Iglesia Filipina Independiente, Socorro, Surigao
del Norte when they celebrated an open Mass at the Plaza on June 19, 1996; and
(3) for threatening to forcibly occupy the Parish Church causing anxiety and fear among the general membership.

Because of the order of expulsion/excommunication, petitioners filed a complaint for damages with
preliminary injunction against Bishop de la Cruz before the Regional Trial Court of Surigao City, Branch 32. They
impleaded Fr. Florano and one Delfin T. Bordas on the theory that they conspired with the Bishop to have petitioners
expelled and excommunicated from the PIC. They contended that their expulsion was illegal because it was done
without trial thus violating their right to due process of law.

● Respondents filed a motion to dismiss the case before the lower court on the ground of lack of jurisdiction but
it was denied. Their motion for reconsideration was likewise denied so they elevated the case to the Court of
Appeals.
● The appellate court reversed and set aside the decision of the court a quo and ordered the dismissal of the
case without prejudice to its being refiled before the proper forum.
○ Acts being purely ecclesiastical matters which this Court considers to be outside the province of the
civil courts.
○ Civil Courts will not interfere in the internal affairs of a religious organization except for the protection
of civil or property rights
○ This case does not involve a violation and/or protection of a civil or property rights in order for the
court a quo to acquire jurisdiction in the instant case.
■ Petitioners appealed again but denied; Motion for reconsideration was filed, likewise denied.

ISSUE: Whether or not the courts have jurisdiction to hear a case involving the expulsion/excommunication of
members of a religious institution.

HELD:
● The Courts do not have jurisdiction.
● Section 5, Article III or the Bill of Rights of the 1987 Constitution specifically provides that:
Sec. 5. No law shall be made respecting an establishment of religion or prohibiting the free exercise thereof. The free
exercise and enjoyment of religious profession and worship, without discrimination or preference, shall forever be
allowed. No religious test shall be required for the exercise of civil or political rights.
● In our jurisdiction, we hold the Church and the State to be separate and distinct from each other. "Give to
Ceasar what is Ceasar's and to God what is God's."
● As early as 1928, the court has observed that:
In a form of government where the complete separation of civil and ecclesiastical authority is insisted upon, the civil
courts must not allow themselves to intrude unduly in matters of an ecclesiastical nature.
● The Court agrees with CA that, the expulsion/excommunication of members of a religious
institution/organization is a matter best left to the discretion of the officials, and the laws and canons, of said
institution/organization. It is not for the courts to exercise control over church authorities in the performance of
their discretionary and official functions.
● In the leading case of Fonacier v. Court of Appeals, we enunciated the doctrine that in disputes involving
religious institutions or organizations, there is one area which the Court should not touch: doctrinal and
disciplinary differences.
○ Thus, The amendments of the constitution, restatement of articles of religion and abandonment of faith or abjuration alleged by
appellant, having to do with faith, practice, doctrine, form of worship, ecclesiastical law, custom and rule of a church and having
reference to the power of excluding from the church those allegedly unworthy of membership, are unquestionably ecclesiastical
matters which are outside the province of the civil courts.
● We would, however, like to comment on petitioners' claim that they were not heard before they were expelled
from their church. The records show that Bishop de la Cruz pleaded with petitioners several times not to
commit acts inimical to the best interests of PIC. They were also warned of the consequences of their actions,
among them their expulsion/excommunication from PIC. Yet, these pleas and warnings fell on deaf ears and
petitioners went ahead with their plans to defy their Bishop and foment hostility and disunity among the
members of PIC in Socorro, Surigao del Norte. They should now take full responsibility for the chaos and
dissension they caused.

State Policies
CONSTI I - CHAPTER 5
CASE DIGESTS

Section 7

SECTION 7. The State shall pursue an independent foreign policy. In its relations with other states the
paramount consideration shall be national sovereignty, territorial integrity, national interest, and the right to self-
determination.

Important provisions:
ART. XVIII, Section 25.
SECTION 25. After the expiration in 1991 of the Agreement between the Republic of the Philippines and the
United States of America concerning Military Bases, foreign military bases, troops, or facilities shall not be
allowed in the Philippines except under a treaty duly concurred in by the Senate and, when the Congress so
requires, ratified by a majority of the votes cast by the people in a national referendum held for that purpose,
and recognized as a treaty by the other contracting State.

Case no. 075


Lim vs. Executive Secretary
G.R. No. 151445 April 11, 2002
Dana
FACTS:
● Petitioners and the intervenors challenged the constitutionality of the joint exercises, Balikatan 02-1.
Petitioners filed suit in their capacities as citizens, lawyers and taxpayers, while the intervenors claimed that
some of their members are residents of Zamboanga and Sulu where the exercises would be held, and, hence,
would be directly affected by the operations.
● Beginning January of this year 2002, personnel from the armed forces of the United States of America started
arriving in Mindanao to take part, in conjunction with the Philippine military, in "Balikatan 02-1." These so-
called "Balikatan" exercises are the largest combined training operations involving Filipino and American
troops. In theory, they are a simulation of joint military maneuvers pursuant to the Mutual Defense Treaty, a
bilateral defense agreement entered into by the Philippines and the United States in 1951.
● On February 1, 2002, petitioners Arthur D. Lim and Paulino P. Ersando filed this petition for certiorari and
prohibition, attacking the constitutionality of the joint exercise. They were joined subsequently by SANLAKAS
and PARTIDO NG MANGGAGAWA, both party-list organizations, who filed a petition-in-intervention on
February 11, 2002.
● Lim and Ersando fied suit in their capacities as citizens, lawyers and taxpayers. SANLAKAS and PARTIDO,
on the other hand, aver that certain members of their organization are residents of Zamboanga and Sulu, and
hence will be directly affected by the operations being conducted in Mindanao. They likewise pray for a
relaxation on the rules relative to locus standi citing the unprecedented importance of the issue involved.

The SolGen says that the petitioners do not have the locus standi to the case. That this case is a question of fact.
Being lawyers does not invest them with sufficient personality to initiate the case. That the petitioner failed to
demonstrate the requisite of direct personal injury. The SC agrees to this. The petitioners are indulging in speculation.
The Terms of Reference (which defines and limits the Balikatan exercise) are clear as to the extent and duration
of the exercises. The issues raised by the petitioners are premature, only based on fear of future violation of the
Terms of Reference. SolGen claims that there is no question to the constitutionality involved. True object is to obtain
an interpretation of VFA.

ISSUES: Whether or not “Balikatan 02-1” is covered by the VFA.

HELD:
The holding of "Balikatan 02-1" must be studied in the framework of the treaty antecedents to which the Philippines
bound itself. The first of these is the Mutual Defense Treaty (MDT, for brevity). The MDT has been described as the
"core" of the defense relationship between the Philippines and its traditional ally, the United States. Its aim is to
enhance the strategic and technological capabilities of our armed forces through joint training with its American
CONSTI I - CHAPTER 5
CASE DIGESTS
counterparts; the "Balikatan" is the largest such training exercise directly supporting the MDT's objectives. It is this
treaty to which the VFA adverts and the obligations thereunder which it seeks to reaffirm.

To determine whether or not the exercise is covered by the VFA, it is necessary to refer to the treaty itself.
● VFA permits US personnel to engage, on an impermanent basis, in “Activities”.
○ It is undefined to permit wide scope of undertakings subject only to the approval of the PH gov’t.
■ Deliberately made this way to give both parties leeway in negotiation.
○ THE SOLE ENCUMBRANCE placed on its definition is couched in the negative, in that United States
personnel must "abstain from any activity inconsistent with the spirit of this agreement, and in
particular, from any political activity." All other activities, in other words, are fair game.
● The cardinal rule of interpretation must involve an examination of the text, which is presumed to verbalize the
parties' intentions. The Convention likewise dictates what may be used as aids to deduce the meaning of
terms, which it refers to as the context of the treaty, as well as other elements may be taken into account
alongside the aforesaid context. (Vienna Convention on the Law of Treaties, Art. 31-32)
○ Treaty interpreted in good faith
● The Terms of Reference rightly fall within the context of VFA.
○ VFA gives legitimacy to the current Balikatan exercises.
■ Combat related activities, as opposed to combat itself, are indeed authorized.
○ US exercise participants may NOT engage in combat EXCEPT IN SELF DEFENSE.
■ The TOR are explicit enough about American troops engaging in combat in PH territory.
● The indirect violation is the petitioners’ worry, that in reality, "Balikatan 02-1" is actually a war principally
conducted by the United States government, and that the provision on self-defense serves only as
camouflage to conceal the true nature of the exercise.
● Neither the MDT nor the VFA allow foreign troops to engage in an offensive war on Philippine territory.
● Both the Mutual Defense Treaty and the Visiting Forces Agreement, as in all other treaties and international
agreements to which the Philippines is a party, must be read in the context of the 1987 Constitution. In
particular, the Mutual Defense Treaty was concluded way before the present Charter, though it nevertheless
remains in effect as a valid source of international obligation. The present Constitution contains key provisions
useful in determining the extent to which foreign military troops are allowed in Philippine territory.
○ See Sections 2, 7, 8 of ART II.
● The Constitution also regulates the foreign relations powers of the Chief Executive when it provides that "[n]o
treaty or international agreement shall be valid and effective unless concurred in by at least two-thirds of all
the members of the Senate.
● From the perspective of public international law, a treaty is favored over municipal law pursuant to the
principle of pacta sunt servanda. Hence, "[e]very treaty in force is binding upon the parties to it and must be
performed by them in good faith." Further, a party to a treaty is not allowed to "invoke the provisions of its
internal law as justi:cation for its failure to perform a treaty." Our Constitution espouses the opposing view.
Witness our jurisdiction as stated in Section 5 of Article VIII, In Ichong v. Hernandez, we ruled that the
provisions of a treaty are always subject to quali:cation or amendment by a subsequent law, or that it is
subject to the police power of the State. The foregoing premises leave us no doubt that US forces are
prohibited from engaging in an offensive war on Philippine territory.

Case no. 076


Saguisag vs. Executive Secretary
G.R. No. 212426 January 12, 2016
Dana
FACTS:
● 1987 CONST provides that the executive power is vested in the President.
● Prime duty of state is to serve and protect the people. See Section 4, Art II
○ Protection of the whole territory is included.
○ To carry this out, President is equipped with AFP, protector of people and state - to secure sovereignty of the state and the
integrity of the national territory.
● The Executive is constitutionally empowered to maintain peace and order; protect life, liberty, and property; and promote the general
welfare.
○ It is subject to limitations. Congress’ involvement in the proclamation of martial law and suspension of writ of habeas corpus.
● The President also carries the mandate of being the sole organ in the conduct of foreign relations. Since every state has the capacity to
interact with and engage in relations with other sovereign states, it is but logical that every state must vest in an agent the authority to
represent its interests to those other sovereign states.
CONSTI I - CHAPTER 5
CASE DIGESTS
● The power to defend the State and to act as its representative in the international sphere inheres in the
person of the President. This power, however, does not crystallize into absolute discretion to craft whatever
instrument the Chief Executive so desires. As previously mentioned, the Senate has a role in ensuring that
treaties or international agreements the President enters into, as contemplated in Section 21 of Article VII of
the Constitution, obtain the approval of two-thirds of its members.
● Previously, treaties under the 1973 Constitution required ratification by a majority of the Batasang Pambansa,
except in instances wherein the President "may enter into international treaties or agreements as the
national welfare and interest may require." This left a large margin of discretion that the President could
use to bypass the Legislature altogether. This was a departure from the 1935 Constitution, which explicitly
gave the President the power to enter into treaties only with the concurrence of two-thirds of all the Members
of the Senate. The 1987 Constitution returned the Senate's power and, with it, the legislative's traditional role
in foreign affairs. The responsibility of the President when it comes to treaties and international agreements
under the present Constitution is therefore shared with the Senate. This shared role, petitioners claim, is
bypassed by EDCA.
● EDCA authorizes the U.S. military forces to have access to and conduct activities within certain "Agreed
Locations" in the country. It was not transmitted to the Senate on the executive's understanding that to
do so was no longer necessary. Accordingly, in June 2014, the Department of Foreign Affairs (DFA) and
the U.S. Embassy exchanged diplomatic notes confirming the completion of all necessary internal
requirements for the agreement to enter into force in the two countries. According to the Philippine
government, the conclusion of EDCA was the result of intensive and comprehensive negotiations in the
course of almost two years. After eight rounds of negotiations, the Secretary of National Defense and the
U.S. Ambassador to the Philippines signed the agreement on 28 April 2014. President Benigno S. Aquino III
ratified EDCA on 6 June 2014. The OSG clarified during the oral arguments that the Philippine and the U.S.
governments had yet to agree formally on the specific sites of the Agreed Locations mentioned in the
agreement.
● Two petitions for certiorari were thereafter filed before us assailing the constitutionality of EDCA. They
primarily argue that it should have been in the form of a treaty concurred in by the Senate, not an
executive agreement. On 10 November 2015, months after the oral arguments were concluded and the
parties ordered to Ile their respective memoranda, the Senators adopted Senate Resolution No. (SR) 105.
The resolution expresses the "strong sense" of the Senators that for EDCA to become valid and effective, it
must first be transmitted to the Senate for deliberation and concurrence.

ISSUES: Whether the President may enter into an executive agreement on foreign military bases, troops, or facilities.
Whether the provisions under EDCA are consistent with the Constitution, as well as with existing laws and treaties

HELD:
1. The role of the President as the executor of the law includes the duty to defend the State, for which
purpose he may use that power in the conduct of foreign relations
● One of the principal functions of the supreme executive is the responsibility for the faithful execution of laws
as embodied by the oath of office.
○ These functions include the faithful execution of the law in autonomous regions; the right to prosecute crimes; the
implementation of transportation projects; the duty to ensure compliance with treaties, executive agreements and executive
orders; the authority to deport undesirable aliens; the conferment of national awards under the President's jurisdiction; and the
overall administration and control of the executive department.
● The court has long treated the power of entering into executive agreements as something the Courts must not
unduly restrict.
○ in the maintenance of our international relations, embarrassment — perhaps serious embarrassment — is to be avoided and
success for our aims achieved, congressional legislation which is to be made effective through negotiation and inquiry within
the international field must often accord to the President a degree of discretion and freedom from statutory restriction which
would not be admissible where domestic affairs alone involved.
○ The presidential role in foreign affairs is dominant and the President is traditionally accorded a wider degree of discretion in the
conduct of foreign affairs.
2. The plain meaning of the Constitution prohibits the entry of foreign military bases, troops or facilities,
except by way of a treaty concurred in by the Senate — a clear limitation on the President's dual role as
defender of the State and as sole authority in foreign relations.
● Despite the President's roles as defender of the State and sole authority in foreign relations, the 1987
Constitution expressly limits his ability in instances when it involves the entry of foreign military bases, troops
or facilities. The initial limitation is found in Section 21 of the provisions on the Executive Department: "No
CONSTI I - CHAPTER 5
CASE DIGESTS
treaty or international agreement shall be valid and effective unless concurred in by at least two-thirds of all
the Members of the Senate." The specific limitation is given by Section 25 of the Transitory Provisions, the full
text of which reads as follows: (SEC25, ART18)
SECTION 25. After the expiration in 1991 of the Agreement between the Republic of the Philippines
and the United States of America concerning Military Bases, foreign military bases, troops, or facilities
shall not be allowed in the Philippines except under a treaty duly concurred in by the Senate and,
when the Congress so requires, ratified by a majority of the votes cast by the people in a national
referendum held for that purpose, and recognized as a treaty by the other contracting State.
● It is quite plain that the Transitory Provisions of the 1987 Constitution intended to add to the basic
requirements of a treaty under Section 21 of Article VII. This means that both provisions must be read as
additional limitations to the President's overarching executive function in matters of defense and foreign
relations.
3. The President, however, may enter into an executive agreement on foreign military bases, troops, or
facilities, if (a) it is not the instrument that allows the presence of foreign military bases, troops, or facilities;
or (b) it merely aims to implement an existing law or treaty.
● It is evident that the constitutional restriction refers solely to the initial entry of the foreign military bases,
troops, or facilities. Once entry is authorized, the subsequent acts are thereafter subject only to the limitations
provided by the rest of the Constitution and Philippine law, and not to the Section 25 requirement of validity
through a treaty.
○ The VFA has already allowed the entry of troops in the Philippines. (Lim v. Exec. Sec.)
○ Moreover, the Court indicated that the Constitution continues to govern the conduct of foreign military
troops in the Philippines, readily implying the legality of their initial entry into the country.
○ The OSG emphasizes that EDCA can be in the form of an executive agreement, since it merely
involves "adjustments in detail" in the implementation of the MDT and the VFA.
● As applied, verba legis aids in construing the ordinary meaning of terms. In this case, the phrase being
construed is "shall not be allowed in the Philippines" and not the preceding one referring to "the expiration in
1991 of the Agreement between the Republic of the Philippines and the United States of America concerning
Military Bases, foreign military bases, troops, or facilities." It is explicit in the wording of the provision itself that
any interpretation goes beyond the text itself and into the discussion of the framers, the context of the
Constitutional Commission's time of drafting, and the history of the 1947 MBA. Without reference to these
factors, a reader would not understand those terms. However, for the phrase "shall not be allowed in the
Philippines," there is no need for such reference. The law is clear. No less than the Senate understood this
when it ratified the VFA.
4. The President may generally enter into executive agreements subject to limitations defined by the
Constitution and may be in furtherance of a treaty already concurred in by the Senate.
● In this section it is discussed why the President can enter into executive agreements.
● The power of the President to enter into binding executive agreements without Senate concurrence is already
well-established in this jurisdiction.
○ That power has been alluded to in our present and past Constitutions, in various statutes, in Supreme Court decisions, and
during the deliberations of the Constitutional Commission. They cover a wide array of subjects with varying scopes and
purposes, including those that involve the presence of foreign military forces in the country.
● As the sole organ of our foreign relations and the constitutionally assigned chief architect of our foreign policy,
the President is vested with the exclusive power to conduct and manage the country's interface with other
states and governments. Being the principal representative of the Philippines, the Chief Executive speaks and
listens for the nation; initiates, maintains, and develops diplomatic relations with other states and
governments; negotiates and enters into international agreements; promotes trade, investments, tourism and
other economic relations; and settles international disputes with other states.
● This exercise of power is subjected for a check (judicial review) on its exercise as provided in Sec. 5(2)(a),
Art. VIII of the CONST.
(2) Review, revise, reverse, modify, or affirm on appeal or certiorari, as the law or the Rules of Court may
provide, final judgments and orders of lower courts in: (a) All cases in which the constitutionality or
validity of any treaty, international or executive agreement, law, presidential decree, proclamation, order,
instruction, ordinance, or regulation is in question.
● Treaties are formal documents which require ratification with the approval of two-thirds of the Senate.
Executive agreements become binding through executive action without the need of a vote by the Senate or
by Congress.
CONSTI I - CHAPTER 5
CASE DIGESTS
○ The right of the Executive to enter into binding agreements without the necessity of subsequent
Congressional approval has been confirmed by long usage.
○ The validity of these has never been seriously questioned by our courts.

5. The President had the choice to enter into EDCA by way of an executive agreement or a treaty.
● No court can tell the President to desist from choosing an executive agreement over a treaty to embody an
international agreement, unless the case falls squarely within Article [X]VIII, Section 25.
● The Constitutional Commission were aware that legally binding international agreements were being entered
into by countries in forms other than a treaty. At the same time, it is clear that they were also keen to preserve
the concept of "executive agreements" and the right of the President to enter into such agreements.
● Indeed, in the field of external affairs, the President must be given a larger measure of authority and wider
discretion, subject only to the least amount of checks and restrictions under the Constitution.
● Section 9 of Executive Order No. 459, or the Guidelines in the Negotiation of International Agreements and its
Ratification, thus, correctly reflected the inherent powers of the President when it stated that the DFA "shall
determine whether an agreement is an executive agreement or a treaty."
● In the exercise of its power of judicial review, the Court does not look into whether an international agreement
should be in the form of a treaty or an executive agreement, save in cases in which the Constitution or a
statute requires otherwise. Rather, in view of the vast constitutional powers and prerogatives granted to the
President in the field of foreign affairs, the task of the Court is to determine whether the international
agreement is consistent with the applicable limitations.
● In order to keep the peace in its archipelago in this region of the world, and to sustain itself at the same time
against the destructive forces of nature, the Philippines will need friends. Who they are, and what form the
friendships will take, are for the President to decide. The only restriction is what the Constitution itself
expressly prohibits. It appears that this overarching concern for balancing constitutional requirements against
the dictates of necessity was what led to EDCA. As it is, EDCA is not constitutionally infirm. As an executive
agreement, it remains consistent with existing laws and treaties that it purports to implement.

Section 10

SECTION 10. The State shall promote social justice in all phases of national development.

Case no. 077


Calalang vs. Williams
G.R. No. 47800 December 2, 1940
CONSTI I - CHAPTER 5
CASE DIGESTS
BY: Wheiiy
FACTS:

Maximo Calalang - Petitioner, a private citizen in Manila filed a petition for a writ of prohibition against the
respondents.

National Traffic Commission - resolved to recommend on July 17, 1940 to the Director of the Public Works and to the
Secretary of Public Works and Communications that animal-drawn vehicles be prohibited from passing along Rosario
Street extending from Plaza Calderon de la Barca to Dasmariñas Street from 7:30 Am to 12:30 pm and from 1:30 pm
to 530 pm; and along Rizal Avenue extending from the railroad crossing at Antipolo Street to Echague Street from 7
am to 11pm for a period of one year from the date of the opening of the Colgante Bridge to traffic.

The Chairman of the NTC - recommended on July 18, 1940 to the Director of Public Works of the measure proposed
in the resolution aforementioned in pursuance of the provisions of the Commonwealth Act No. 548 which authorizes
the Director to promulgate rules and regulations to regulate and control the use of and traffic on national roads.

On August 2, 1940, the Director recommended to the Secretary the approval of the recommendations made by the
Chairman of the NTC with modifications and thereafter the Secretary of Public Works approved the recommendations.
Hence, the Mayor of Manila and the Acting Chief of Police of Manila have enforced the rules and regulations. As a
consequence, all animal-drawn vehicles are not allowed to pass and pick up passengers in the places above
mentioned to the detriment not only of their owners but of the riding public as well.

ISSUE: Whether or not CA No. 548 infringes upon the constitutional precept regarding the promotion of social justice
to insure the well-being and economic security of all the people

HELD: No. The promulgation of the Act aims to promote safe transit upon and avoid obstructions on national roads in
the interest and convenience of the public. Thus, SC denied the writ for prohibition. Social justice means the
promotion of the welfare of all the people, the adoption by the Government of measures calculated to insure economic
stability of all the competent elements of society, through the maintenance of a proper economic and social
equilibrium in the interrelations of the members of the community, constitutionally, through the adoption of measures
legally justifiable, or extra-constitutionally, through the exercise of powers underlying the existence of all governments
on the time-honored principles of salus populi est suprema lex. Social justice must be founded on the recognition of
the necessity of interdependence among divers and diverse units of a society and of the protection that should be
equally and evenly extended to all groups as a combined force in our social and economic life, consistent with the
fundamental and paramount objective of the state of promoting health, comfort and quiet of all persons, and of
bringing about “the greatest good to the greatest number.”

Case no. 078


Guido vs. Rural Progress Administration
G.R. No. L-2089 October 31, 1949
BY: Wheiiy

FACTS:

Justa Guido - Petitioner, owner of the land being expropriated by the Rural Progress Administration, filed a petition for
prohibition to prevent the RPA and Judge Oscar Castelo from proceeding with the expropriation. Petitioner alleged
that the land sought to be expropriated is commercial and therefore excluded within the sphere of the provisions of CA
No. 539.

Commonwealth Act No. 539 - authorized the President of the Philippines to acquire private lands or any interest
therein through purchaser or farms for resale at a reasonable price in which the National Assembly approved this
enactment on the authority of ..
CONSTI I - CHAPTER 5
CASE DIGESTS
Article XIII Sec 4 of the Constitution - provides that the Congress may authorize, upon payment of just compensation,
the expropriation of lands to be subdivided into small lots and conveyed at cost to individuals

ISSUE: Whether the expropriation of Guido’s land is in conformity to the principle of social justice.

HELD: ”the promotion of social justice to insure the well-being and economic security of all the people should be the
concern of the state," is a declaration, with which the former should be reconciled, that "the Philippines is a
Republican state" created to secure to the Filipino people "the blessings of independence under a regime of justice,
liberty and democracy.” The promotion of social justice ordained by the Constitution does not supply paramount basis
for untrammeled expropriation of private land by the Rural Progress Administration or any other government
instrumentality. Social justice does not champion division of property or equality of economic status; what it and the
Constitution do guaranty are equality of opportunity, equality of political rights, equality before the law, equality
between values given and received on the basis of efforts exerted in their production. As applied to metropolitan
centers, especially Manila, in relation to housing problems, it is a command to devise, among other social measures,
ways and means for the elimination of slums, shambles, shacks, and house that are dilapidated, overcrowded, without
ventilation. light and sanitation facilities, and for the construction in their place of decent dwellings for the poor and the
destitute. As will presently be shown, condemnation of blighted urban areas bears direct relation to public safety
health, and/or morals, and is legal.

Case no. 079


Almeda vs. CA
G.R. No. L-43800 July 29, 1977
BY: Wheiiy

FACTS:

Eulogio Gonzales - Respondent, an agricultural share tenant of Glicerio et.al Angeles on a 46,529 sqm land
containing sugar cane and coconuts located in Tanauan, Batangas

Leonila & Venancion Almeda - Petitioners-Spouses, whom the landowners sold their property to without notifying
Eulogio Gonzales of the writing of the sale. Moreover, the document of sale was registered in the Register of Deeds.

March 27, 1971 - Gonzales petitioned for the redemption of the land pursuant to the provisions of Sections 11 and 12
of the Code of Agrarian Reforms with the Court of Agrarian Relations at Lipa City

During the hearing, the parties waived their right to present evidence and agreed to file simultaneous memoranda
upon which the decision of the court would be based. Thereafter, the Agrarian Court rendered judgment authorizing,
Gonzales to redeem the tenanted land for P24,000.00, the said amount to be deposited by him with the Clerk of Court
within 15 days from receipt of the decision. The Appellate Court, however, affirmed the decision of the Agrarian
Court.Petitioners- spouses were denied of their motions for reconsideration, hence instituted the present petition for
review.

ISSUE: WON respondent Eulogio Gonzales have validly exercised his right of redemption over his tenanted
agricultural land.

HELD: The appealed decision of the Court of Appeals is hereby reversed and set aside. Respondent Eulogio
Gonzales is hereby held not to have validly exercised his right of redemption over his tenanted agricultural land. The
state may regulate property ownership in the promotion of social justice. Under the new Constitution, property
ownership is impressed with social function. Property use must not only be for the benefit of the owner but of society
as well. The State, in the promotion of social justice, may "regulate the acquisition, ownership, use, enjoyment and
disposition of private property, and equitably diffuse property . . . ownership and profits." Moreover, the Court of
Agrarian Relations has jurisdiction over suits for redemption, like the present case, of sugar and coconut lands.
Section 154 of the Agricultural Land Reform Code, as amended, states: "The Court of Agrarian Relations shall have
CONSTI I - CHAPTER 5
CASE DIGESTS
original and exclusive jurisdiction over (1) all cases or actions involving matters, controversies, disputes, or money
claims arising from agrarian relations ..." Since this case involves a matter, controversy or dispute "arising from
agrarian relations" — whether respondent-tenant on sugar and coconut lands has the right of redemption

Case no. 080


Ondoy vs. Ignacio
G.R. No. L-47178 May 16, 1980
BY: Wheiiy

FACTS:

Jose Ondoy - whom, according to the chief engineer, was aboard the ship of the Virgilio Ignacio’s enterprise
(Respondent) as part of the workforce. Jose was invited by his friends to a drinking spree and afterwards left the ship
but was thereafter found dead due to drowning.

Estrella Ondoy - Petitioner, mother of Jose Ondoy, an employee who worked under Virgilio Ignacio, asked for
compensation. However, the testimonies by the chief engineer were dismissed by the hearing officer due to lack of
merit. Thereafter, a motion for reconsideration was filed before the Secretary of Labor but was again denied due to
lack of merit.

ISSUE : Whether or not the compensation for Jose’s death is constitutional and whether social justice has a role in
this case.

RULING:

Art II Sec 10 of the Constitution of the Philippines - The State shall promote social justice in all phases of national
development

The principle of social justice applied in this case is a matter of protection, and not equality. The SC recognized the
right of petitioner to claim a compensation from the respondent, as Jose did drown while “in the actual performance of
his duty.” The SC granted the petition for review, granting Estrella Ondoy 6,000 pesos as compensation for Jose’s
death, 300 pesos for burial fees and 600 pesos as attorney’s fee with the costs against Virgilio Ignacio.

[“What was said in Victorias Milling Co., Inc. v. Workmen's Compensation Commission is not amiss: "There is need, it
seems, even at this late date, for [private respondent] and other employers to be reminded of the high estate accorded
the Workmen's Compensation Act in the constitutional scheme of social justice and protection to labor. Further: "No
other judicial attitude may be expected in the face of a clearly expressed legislative determination which antedated the
constitutionally avowed concern for social justice and protection to labor. It is easily understandable why the judiciary
frowns on resort to doctrines, which even if deceptively plausible, would result in frustrating such a national policy.
Lastly, to quote from the opinion therein rendered: "To be more specific, the principle of social justice is in this sphere
strengthened and vitalized. A realistic view is that expressed in Agustin v. Workmen's Compensation Commission: 'As
between a laborer, usually poor and unlettered, and the employer, who has resources to secure able legal advice, the
law has reason to demand from the latter stricter compliance. Social justice in these cases is not equality but
protection.' “ ]

Case no. 081


Salonga vs. Farrales
G.R. No. L-47088 July 10, 1981
BY: Wheiiy
CONSTI I - CHAPTER 5
CASE DIGESTS
FACTS:

Julia B. Farrales - Respondent, the titled owner of residential lot in Sta. Rita, Olongapo City.

Salonga Spouses - Petitioners, are lessees of the 156 sqm. land where they built a house and are paying rentals to
Farrales. Sometime before 1968, respondent filed an ejectment case for non-payment of rentals against the
petitioners. The lower court hereby rendered a decision in favor of Farrales and ordered the respondents and other
lessees to vacate the portion occupied by them and pay the necessary fees such as rentals, attorney’s fees, etc.
Thereafter, they offered to buy the occupied parcel of land instead of vacating however the respondent refused.

The plaintiff the filed for petition for relief and the case was elevated until the CA, praying for ordering the defendant
to sell her parcel of land where the house of the plaintiff was erected and invoked their right to be subjected to the
application social justice through Article II Section 6 of the new constitution

ISSUE: WON the contention of Salonga is correct in invoking the promotion of social justice

HELD:

The appeal is dismissed for lack of merit and the judgment appealed from is hereby affirmed.

Art II Sec 10 of the Constitution - The State shall promote social justice in all phases of national development. No,
social justice cannot be invoked by the petitioners just to gain relief and force the respondent to sell her land where
the petitioners’ house was erected.

Article II Section 6 of the Constitution - Social justice cannot be invoked to trample on the rights of property owners
who under our Constitution and laws are also entitled to protection. The social justice consecrated in our constitution
was not intended to take away rights from a person and give them to another who is not entitled thereto. Evidently, the
plea for social justice cannot nullify the law on obligations and contracts, and is, therefore, beyond the power of the
Court to grant.

Section 11
Case no. 082
Carino vs. CHR
G.R. No. 96681 December 2, 1991
BY: Wheiiy

FACTS:

September 17, 1990 - 800 public school teachers in Manila did not attend work and decided to stage rallies in order to
air their grievances. As a result, 8 teachers got suspended from work for 90 days.

Secretary Isidro Cariño - Petitioner; On December 17, 1990, he ordered the dismissal from the service of one teacher
and the suspension of three others.

Commission on Human Rights - the case was appealed to, and in the meantime, the Solicitor General filed an action
for certiorari regarding the case and prohibiting the CHR from continuing the case. Nevertheless, CHR continued trial
and issued a subpoena to petitioner Secretary Isidro Cariño.

ISSUE: Whether or not CHR has the power to try and decide and determine certain specific cases such as the alleged
human rights violation involving civil and political rights.

HELD:

No. The CHR is not competent to try such case because it has no judicial power. Hence, petition is granted.
CONSTI I - CHAPTER 5
CASE DIGESTS
Article XIII Section 18 - It can only “Investigate, on its own or on complaint by any party, all forms of human rights
violations involving civil and political rights;” but it cannot and should not try and decide on the merits and matters
involved therein. The CHR is hence then barred from proceeding with the trial.

[The CHR simply has no place in this scheme of things. It has no business intruding into the jurisdiction and functions
of the Education Secretary or the Civil Service Commission. It has no business going over the same ground traversed
by the latter and making its own judgment on the questions involved. This would accord success to what may well
have been the complaining teachers' strategy to abort, frustrate or negate the judgment of the Education Secretary in
the administrative cases against them which they anticipated would be adverse to them.The Order of December 29,
1990 is ANNULLED and SET ASIDE, and the respondent Commission on Human Rights and the Chairman and
Members thereof are prohibited "to hear and resolve the case (i.e., Striking Teachers HRC Case No. 90-775) on the
merits.]

Case no. 083


Sec. of National Defense vs. Manalo
G.R. No. 180906 October 7, 2008
BY: Wheiiy

FACTS:

Raymond & Reynaldo Manalo - Respondents, farmers from Bulacan who were suspected of being members of the
NPA and forcibly taken from their home, detained in different places, as well as tortured by CAFGU and other military
units.

General Jovito Palparan - together with his members of the armed forces, was recognized by the Manalo brothers as
their abductors. Moreover, the Petitioners also learned that they were being held in place for their brother who is a
suspected leader of the communist insurgents. While in captivity, they met other desaperacidos who were also
suspected of being communist insurgents and members of the NPA. After 18 months, the brothers were able to
escape and file a petition for the writ of amparo.

December 26, 2007 - CA granted the privilege of the writ of amparo and ordered the Secretary of National Defense
and the Chief of Staff of the AFP to furnish the Manalo Brothers and the court with all official and unofficial
investigation reports. The Secretary of National Defense and the Chief of Staff of the AFP appealed to the SC seeking
to reverse and set aside the decision promulgated by the CA.

ISSUE: WON the Manalo Brothers’ right to life and liberty as well as security were violated

HELD:

Article XIII Section 18 - The Commission on Human Rights shall have the following powers and functions:

(1) Investigate, on its own or on complaint by any party, all forms of human rights violations involving civil and political
rights;
(2) Adopt its operational guidelines and rules of procedure, and cite for contempt for violations thereof in accordance
with the Rules of Court;
(3) Provide appropriate legal measures for the protection of human rights of all persons within the Philippines, as well
as Filipinos residing abroad, and provide for preventive measures and legal aid services to the underprivileged whose
human rights have been violated or need protection;
(4) Exercise visitorial powers over jails, prisons, or detention facilities;
(5) Establish a continuing program of research, education, and information to enhance respect for the primacy of
human rights;
(6) Recommend to Congress effective measures to promote human rights and to provide for compensation to victims
of violations of human rights, or their families;
(7) Monitor the Philippine Government’s compliance with international treaty obligations on human rights;
CONSTI I - CHAPTER 5
CASE DIGESTS
(8) Grant immunity from prosecution to any person whose testimony or whose possession of documents or other
evidence is necessary or convenient to determine the truth in any investigation conducted by it or under its authority;
(9) Request the assistance of any department, bureau, office, or agency in the performance of its functions;
(10) Appoint its officers and employees in accordance with law; and
(11) Perform such other duties and functions as may be provided by law.

Article III Sec. 2. - The right of the people to be secure in their persons, houses, papers and effects against
unreasonable searches and seizures of whatever nature and for any purpose shall be inviolable, and no search
warrant or warrant of arrest shall issue except upon probable cause to be determined personally by the judge…

The SC characterized the right to security of person and how these were violated:
A) Freedom from fear/threat - while respondents were detained, they were threatened that if they escaped, their
families, including them, would be killed.
B) A guarantee of bodily and psychological integrity and security - guarantee of protection of one’s rights by the
government

The SC ruled that respondents’ right to security as freedom from threat was violated by the apparent threat to their
life, liberty and security of person. Their right to security as a guarantee of protection by the government was likewise
violated by the ineffective investigation and protection on the part of the military. Be that as it may, the Writ of Amparo
is the most potent remedy available to any person whose right to life, liberty, and security has been violated or is
threatened with violation by an unlawful act or omission by public officials or employees and by private individuals or
entities.

Section 12
Case no. 084
Meyer vs. Nebraska
262 U.S. 393
BY: Wheiiy

FACTS:

Meyer - Petitioner; a teacher, who taught German to a 10-year-old child. As a result, he was convicted of violating a
Nebraska state law forbidding, under penalty, the teaching in any private, denominational, parochial or public school,
of any modern language, other than English, to any child who has not attained and successfully passed the eighth
grade, invades the liberty guaranteed by the Fourteenth Amendment and exceeds the power of the State.

ISSUE: Does the statute as construed and applied unreasonably infringe on the liberty guaranteed by the Fourteenth
Amendment?

HELD:

Article II Section 13 of the Philippine Constitution - The State recognizes the vital role of the youth in nation- building
and shall promote and protect their physical moral, spiritual, intellectual, and social well-being. It shall inculcate in the
youth patriotism and nationalism, and encourage their involvement in public and civic affairs.

The State cannot prohibit the teaching of foreign language to children before they reach a certain age. Such restriction
does violence both to the letter and the spirit of the Constitution. The statute that prohibits the teaching of foreign
languages to students who have not yet passed the 8th grade is unconstitutional because it infringes on the liberty
interests of the plaintiff and fails to reasonably relate to any end within the competency of the state. The 14th
Amendment protects individuals from arbitrary or unreasonable state action impairing life, liberty, or property interests
and the right to practice the profession of the plaintiff is a right protected by the amendment.

The stated purpose of restriction on the right to teach such foreign languages was that children who know only English
CONSTI I - CHAPTER 5
CASE DIGESTS
through grade eight will be better citizens of the US. However, because there is no clear danger to the state that
stems from younger children studying foreign languages, the reason given for the restriction is unreasonable and
arbitrary, therefore, insufficient to support the limitation on the right to teach.

Case no. 085


Pierce vs. Society of Sisters
268 U.S. 510
BY: Wheiiy

FACTS:

Compulsory Education Act of 1922 - required parents or guardians to send children between the ages of eight and
sixteen to public school in the district where the children resided.

Walter Pierce - Governor of Oregon

The Society of Sisters of the holy Names of Jesus and Mary - Respondent, a corporation with the power to establish
and maintain academies or schools and Appellee Hill Military Academy, a private organization conducting an
elementary, college preparatory, and military training school, obtained preliminary restraining orders prohibiting
appellants from enforcing Oregon’s Compulsory Education Act wherefore the appellants appealed the granting of the
preliminary restraining orders.

ISSUE: Does the Act unreasonably interfere with the liberty of parents and guardians to direct the upbringing and
education of children under their control?

HELD:

Article II Section 13 of the Philippine Constitution - The State recognizes the vital role of the youth in nation- building
and shall promote and protect their physical moral, spiritual, intellectual, and social well-being. It shall inculcate in the
youth patriotism and nationalism, and encourage their involvement in public and civic affairs.

The State cannot require children to attend only public schools before they reach a certain age. The child is not a
mere creature of the State. Those who nurture him and direct his destiny have the right to recognize and prepare him.
The court therefore upheld the right of parents to make educational decisions on behalf of their children while
acknowledging the states’ right to regulate education, even in nonpublic schools. The 14th Amendment provides a
liberty interest in a parent’s or guardian’s right to decide the mode in which their children are educated. State’s may
not usurp this right when the questioned legislation does not reasonably relate to a viable state interest.

[“Associate Justice James Clark McReynolds wrote the opinion of the Court. He stated that children were not "the
mere creature[s] of the state" (268 U.S. 510, 535), and that, by its very nature, the traditional American understanding
of the term liberty prevented the state from forcing students to accept instruction only from public schools. He stated
that this responsibility belonged to the child's parents or guardians, and that the ability to make such a choice was a
"liberty" protected by the Fourteenth Amendment.”]

Case no. 086


DECS and Dir. of Educational Measurement vs. Roberto Rey San Diego and Judge Dizon-Capulong
G.R. No. 89572, December 21, 1989
Case Digest By: Atty. Tal
FACTS:
It is time indeed that the State took decisive steps to regulate and enrich our system of education by directing the student to the course for which he
is best suited as determined by initial tests and evaluations. Otherwise, we may be "swamped with mediocrity," in the words of Justice Holmes, not
because we are lacking in intelligence but because we are a nation of misfits.
● The private respondent is a graduate of the University of the East with a degree of Bachelor of Science in
Zoology. The petitioner claims that he took the NMAT three times and flunked it as many times. When he
applied to take it again, the petitioner rejected his application on the basis of the aforesaid rule.
● He then went to the Regional Trial Court of Valenzuela, Metro Manila, to compel his admission to the test.
CONSTI I - CHAPTER 5
CASE DIGESTS
● In his original petition for mandamus, he first invoked his constitutional rights to academic freedom and
quality education. By agreement of the parties, the private respondent was allowed to take the NMAT
scheduled on April 16, 1989, subject to the outcome of his petition.
● In an amended petition led with leave of court, he squarely challenged the constitutionality of MECS Order
No. 12, Series of 1972, containing the above-cited rule. The additional grounds raised were due process
and equal protection. After hearing, the respondent judge rendered a decision on July 4, 1989, declaring the
challenged order invalid and granting the petition.
Judge Teresita Dizon-Capulong
● held that the petitioner had been deprived of his right to pursue a medical education through an arbitrary
exercise of the police power. We cannot sustain the respondent judge. Her decision must be reversed.
● agreed with the petitioner that the said case was not applicable. Her reason was that it upheld only the
requirement for the admission test and said nothing about the so-called "three-flunk rule." We see no reason
why the rationale in the Tablarin case cannot apply to the case at bar. The issue raised in both cases is the
academic preparation of the applicant.
HELD:
1.CASE OF TABLARIN V. GUTIERREZ, UPHOLDING THE CONSTITUTIONALITY OF THE NMAT REITERATED.
In Tablarin v. Gutierrez, the Court upheld the constitutionality of the NMAT as a measure intended to limit the
admission to medical schools only to those who have initially proved their competence and preparation for a medical
education.
● We believe that the government is entitled to prescribe an admission test like the NMAT as a means of
achieving its stated objective of "upgrading the selection of applicants into our medical schools" and of
"improving the quality of medical education in the country." Given the widespread use today of such
admission, considering the failure or inability of the petitioners to even attempt to prove otherwise, we are
entitled to hold that the NMAT is reasonably related to the securing of the ultimate end of legislation and
regulation in this area.
2. POLICE POWER validly exercised if (a) the interests of the public generally, as distinguished from those of a
particular class, require the interference of the State (b) the means employed are reasonably necessary to the
attainment of the object sought to be accomplished & not unduly oppressive upon individuals.
● The subject of the challenged regulation is certainly within the ambit of the police power. It is the right and
indeed the responsibility of the State to insure that the medical profession is not infiltrated by incompetents to
whom patients may unwarily entrust their lives and health.
● The three-flunk rule is intended to insulate the medical schools and ultimately the medical profession from the
intrusion of those not qualified to be doctors. While every person is entitled to aspire to be a doctor, he does
not have a constitutional right to be a doctor. This is true of any other calling in which the public interest is
involved; and the closer the link, the longer the bridge to one's ambition.
● A person cannot insist on being a physician if he will be a menace to his patients. If one who wants to be a
lawyer may prove better as a plumber, he should be so advised and adviced.
3. ACADEMIC FREEDOM, NOT ABSOLUTE. — The right to quality education is not absolute. The Constitution also
provides that "every citizen has the right to choose a profession or course of study, subject to fair, reasonable and
equitable admission and academic requirements."
4. EQUAL PROTECTION CAUSE NOT VIOLATED IN CASE AT BAR. — The contention that the challenged rule
violates the equal protection clause is not well-taken. The medical profession directly affects the very lives of the
people, unlike other careers which, for this reason, do not require more vigilant regulation. There would be unequal
protection if some applicants who have passed the tests are admitted and others who have also qualfiied are denied
entrance. In other words, what the equal protection requires is equality among equals.
● The Court feels that it is not enough to simply invoke the right to quality education as a guarantee of the
Constitution: one must show that he is entitled to it because of his preparation and promise. The private
respondent has failed the NMAT 5 times. While his persistence is noteworthy, to say the least, it is certainly
misplaced, like a hopeless love.

Case no. 087


Francisco Virtuoso Jr. v. Municipal Judge of Mariveles, Bataan
G.R. No. L-47841. March 21, 1978.
Case Digest by: Atty. Tal
FACTS:
CONSTI I - CHAPTER 5
CASE DIGESTS
Francisco Virtouso, Jr- application for the writ of habeas corpus on February 23, 1978, premised his plea for liberty
primarily on the ground that the preliminary examination which led to the issuance of a warrant of arrest against him
was a useless formality
● Petitioner is a seventeen-year old minor entitled to the protection and benefits of the Child and Youth Welfare
Code, a youthful offender being deemed therein as "one who is over nine years but under eighteen years of
age at the time of the commission of the offense." As such, he could be provisionally released on
recognizance in the discretion of a court
Respondent Municipal Judge of Mariveles, Bataan
1. failed to meet the strict standard required by the Constitution to ascertain whether there was a probable cause.
2. alleged that aside from the constitutional infirmity that tainted the procedure followed in the preliminary examination,
the bail imposed was clearly excessive.
3 It was in the amount of P16,000.00, the alleged robbery of a TV set being imputed to petitioner. As prayed for, the
Court issued a writ of habeas corpus, returnable to it on Wednesday, March 15, 1978.
● Respondent Judge, in his return led on March 8, 1978, justified the issuance of the warrant of arrest, alleging
that there was no impropriety in the way the preliminary examination was conducted.
Excessive character of the bail- he asserted that it was fixed in accordance with the Revised Bail Bond Guide
issued by the Executive Judge of Bataan in 1977, he nevertheless reduced the amount to P8,000.00
HELD:
- Petition resolved, without the need of passing upon the issue of whether or not the procedure by respondent
Judge in ascertaining the existence of probable cause was constitutionally deficient. Nonetheless, it must ever
be kept in mind by occupants of the bench that they should always be on the alert lest by sloth or indifference
or due to the economic or social standing of the alleged offended party, as was intimated in this petition, the
rights of an accused, instead of being honored, are disregarded.
- There is much more importance attached to the immunities of an individual during a period of martial law,
which in itself is a creature of the Constitution as a mode of coping with grave emergency situations.
- It is equally pertinent to state that there should be fealty to the constitutional ban against excessive bail being
required. Where, however, the right to bail exists, it should not be rendered nugatory by requiring a sum that
is excessive. The sole permissible function of money bail is to assure the accused's presence at trial, and
declared that "bail set at a higher figure than an amount reasonably calculated to fulfill this purpose is
excessive.

Case no. 088


IMBONG vs OCHOA
G.R. No. 204819 April 8, 2014
Case Digest by: Atty. Tal
FACTS:
● Concerned citizens and the Catholic Church had petitioned for the constitutionality of the Reproductive Health Bill.
● This a Landmark Case not only citing the validity of the RH Bill but also the classic scenario where of the Catholic Church vs the
Government. It is no secret that the Catholic Church plays political power in the Philippines for many years. This displays ow the
separation of the State and the Church paves it
FACTS:
● The increase of the country’s population at an uncontrollable pace led to the executive and the legislative’s
decision that prior measures were still not adequate. Thus, Congress enacted R.A. No. 10354, otherwise
known as the Responsible Parenthood and Reproductive Health Act of 2012 (RH Law), to provide Filipinos,
especially the poor and the marginalized, access and information to the full range of modern family planning
methods, and to ensure that its objective to provide for the people's’ right to reproductive health be achieved.
Stated differently, the RH Law is an enhancement measure to fortify and make effective the current laws on
contraception, women’s health and population control.
● Petitioners question, among others, the constitutionality of the RH Law, claiming that it violates Section 26(1),
Article VI of the Constitution, prescribing the one subject-one title rule.
● According to them, being one for reproductive health with responsible parenthood, the assailed legislation
violates the constitutional standards of due process by concealing its true intent – to act as a population
control measure.
● On the other hand, respondents insist that the RH Law is not a birth or population control measure, and that
the concepts of “responsible parenthood” and “reproductive health” are both interrelated as they are
inseparable.
CONSTI I - CHAPTER 5
CASE DIGESTS
● Shortly after the President placed his imprimatur on the said law, challengers from various sectors of society
came knocking on the doors of the Court, beckoning it to wield the sword that strikes down constitutional
disobedience. Aware of the profound and lasting impact that its decision may produce, the Court now faces
the iuris controversy, as presented in 14 petitions and 2 petitions- in-intervention. A perusal of the foregoing
petitions shows that the petitioners are assailing the constitutionality of RH Law

SUBSTANTIAL: Whether the court is Unconstitutional. WON RA 10354/Reproductive Health (RH) Law is
unconstitutional for violating the:
1. Right to life
2. Right to health
3. Freedom of religion and right to free speech
4. Right to privacy (marital privacy and autonomy)
5. Freedom of expression and academic freedom
6. Due process clause
7. Equal protection clause
8. Prohibition against involuntary servitude

PROCEDURAL: Whether the Court may exercise its power of judicial review over the controversy.
1. Actual Case or Controversy
2. Facial Challenge
3. Locus Standi
4. Declaratory Relief
5. One Subject/One Title Rule
HELD:
SUBSTANTIAL
1. Majority of the Members of the Court believe that the question of when life begins is a scientific and medical issue
that should not be decided, at this stage, without proper hearing and evidence. However, they agreed that individual
Members could express their own views on this matter.
● Article II, Section 12 of the Constitution. In its plain and ordinary meaning, the traditional meaning of “conception” is that life
begins at fertilization. The framers of the Constitution also intended for (a) “conception” to refer to the moment of “fertilization” and (b)
the protection of the unborn child upon fertilization. Contraceptives that actually prevent the union of the male sperm and female ovum,
and those that similarly act before fertilization should be deemed non-abortive, and thus constitutionally permissible.
The RH Law is in line with this intent and actually prohibits abortion. By using the word “or” in defining abortifacient,
the RH Law prohibits not only drugs or devices that prevent implantation but also those that induce abortion and
induce the destruction of a fetus inside the mother’s womb. The RH Law recognizes that the fertilized ovum already
has life and that the State has a bounded duty to protect it.

2. The RH Law does not intend to do away with RA 4729 (1966). With RA 4729 in place, the Court believes adequate
safeguards exist to ensure that only safe contraceptives are made available to the public. In fulfilling its mandate
under Sec. 10 of the RH Law, the DOH must keep in mind the provisions of RA 4729: the contraceptives it will procure
shall be from a duly licensed drug store or pharmaceutical company and that the actual distribution of these
contraceptive drugs and devices will be done following a prescription of a qualified medical practitioner.

3. The Court cannot determine whether or not the use of contraceptives or participation in support of modern RH
measures (a) is moral from a religious standpoint; or, (b) right or wrong according to one’s dogma or belief. However,
the Court has the authority to determine whether or not the RH Law contravenes the Constitutional guarantee
of religious freedom.
● The State may pursue its legitimate secular objectives without being dictated upon the policies of any one religion. To allow
religious sects to dictate policy or restrict other groups would violate Article III, Section 5 of the Constitution or the Establishment Clause.
This would cause the State to adhere to a particular religion, and thus, establishes a state religion.

4. Section 23A (2)(i) of the RH Law, which permits RH procedures even with only the consent of the spouse
undergoing the provision, intrudes into martial privacy and autonomy and goes against the constitutional safeguards
for the family as the basic social institution. Section 3, Article XV of the Constitution mandates the State to defend: (a)
CONSTI I - CHAPTER 5
CASE DIGESTS
the right of spouses to found a family in accordance with their religious convictions and the demands of responsible
parenthood and (b) the right of families or family associations to participate in the planning and implementation of
policies and programs that affect them. The RH Law cannot infringe upon this mutual decision-making, and endanger
the institutions of marriage and the family.
A minor may receive information about family planning services. Parents are not deprived of parental guidance and
control over their minor child in this situation and may assist her in deciding whether to accept or reject the information
received.

5. The Court declined to rule on the constitutionality of Sec. 14 of the RH Law, which mandates the State to provide
Age-and Development-Appropriate Reproductive Health Education. Educators might raise their objection to their
participation in the RH education program, but the Court reserves its judgment should an actual case be filed before it.
● Any attack on its constitutionality is premature because the Department of Education has not yet formulated a curriculum on age-
appropriate reproductive health education.

6. The RH Law does not violate the due process clause of the Constitution as the definitions of several terms as
observed by the petitioners are not vague. The definition of “private health care service provider” must be seen in
relation to Section 4(n) of the RH Law which defines a “public health service provider”. The “private health care
institution” cited under Section 7 should be seen as synonymous to “private health care service provider.
· The terms “service” and “methods” are broad enough to include providing of information and rendering of medical procedures. Thus,
hospitals operated by religious groups are exempted from rendering RH service and modern family planning methods as well as
from giving RH information and procedures. The RH Law also defines “incorrect information”. Used together in relation to Section 23 (a)(1),
the terms “incorrect” and “knowingly” connote a sense of malice and ill motive to mislead or misrepresent the public as to the nature and effect
of programs and services on reproductive health.

7. To provide that the poor are to be given priority in the government’s RH program is not a violation of the equal
protection clause pursuant to Section 11, Article XIII of the Constitution. The RH Law does not only seek to target the
poor to reduce their number, since Section 7 of the RH Law prioritizes poor and marginalized couples who are
suffering from fertility issues and desire to have children. In addition, the RH Law does not prescribe the number of
children a couple may have and does not impose conditions on couples who intend to have children. The RH Law
only seeks to provide priority to the poor.

The exclusion of private educational institutions from the mandatory RH education program under Section 14 is valid.
There is a need to recognize the academic freedom of private educational institutions especially with respect to
religious instruction and to consider their sensitivity towards the teaching of reproductive health education

8. The requirement under Sec. 17 of the RH Law for private and non-government health care service providers to
render 48 hours of pro bono RH services does not amount to involuntary servitude, for two reasons. First, the practice
of medicine is undeniably imbued with public interest that it is both the power and a duty of the State to control and
regulate it in order to protect and promote the public welfare. Second, Section 17 only encourages private and non-
government RH service providers to render pro bono. Besides the PhilHealth accreditation, no penalty is imposed
should they do otherwise.
However, conscientious objectors are exempt from Sec. 17 as long as their religious beliefs do not allow them to
render RH service, pro bono or otherwise

PROCEDURAL:

1. In this case, the Court is of the view that an actual case or controversy exists and that the same is ripe for judicial
determination. Considering that the RH Law and its implementing rules have already taken effect and that
budgetary measures to carry out the law have already been passed, it is evident that the subject petitions
present a justiciable controversy. As stated earlier, when an action of the legislative branch is seriously alleged to
have infringed the Constitution, it not only becomes a right, but also a duty of the Judiciary to settle the dispute.
The petitioners have shown that the case is so because medical practitioners are in danger of being criminally
prosecuted under the RH Law for vague violations thereof, particularly public health officers who are threatened to be
dismissed from the service with forfeiture of retirement and other benefits.

2. This Court is mandated by the Fundamental Law not only to settle actual controversies involving rights which are
legally demandable and enforceable, but also to determine whether or not there has been a grave abuse of discretion
CONSTI I - CHAPTER 5
CASE DIGESTS
amounting to lack or excess of jurisdiction on the part of any branch or instrumentality of the Government. Verily, the
framers of Our Constitution envisioned a proactive Judiciary, ever vigilant with its duty to maintain the supremacy of
the Constitution.
Consequently, considering that the foregoing petitions have seriously alleged that the constitutional human rights to
life, speech and religion and other fundamental rights mentioned above have been violated by the assailed legislation,
the Court has authority to take cognizance of these kindred petitions and to determine if the RH Law can indeed pass
constitutional scrutiny.

3. The rule on locus standi is, after all, a procedural technicality which the Court has, on more than one occasion,
waived or relaxed, thus allowing non-traditional plaintiffs, such as concerned citizens, taxpayers, voters or legislators,
to sue in the public interest, albeit they may not have been directly injured by the operation of a law or any other
government act.
The present action cannot be properly treated as a petition for prohibition, the transcendental importance of the issues
involved in this case warrants that the Court set aside the technical defects and take primary jurisdiction over the
petition at bar. One cannot deny that the issues raised herein have potentially pervasive influence on the social and
moral well-being of this nation, especially the youth; hence, their proper and just determination is an imperative need.

4. Most of the petitions are praying for injunctive reliefs and so the Court would just consider them as petitions for
prohibition under Rule 65, over which it has original jurisdiction. Where the case has far-reaching implications and
prays for injunctive reliefs, the Court may consider them as petitions for prohibition under Rule 65.

5. The RH Law does not violate the one subject/one bill rule. In this case, a textual analysis of the various provisions
of the law shows that both “reproductive health” and “responsible parenthood” are interrelated and germane to the
overriding objective to control the population growth. As expressed in the first paragraph of Section 2 of the RH Law:
● SEC. 2. Declaration of Policy. – The State recognizes and guarantees the human rights of all persons including their right to equality and nondiscrimination of
these rights, the right to sustainable human development, the right to health which includes reproductive health, the right to education and information, and
the right to choose and make decisions for themselves in accordance with their religious convictions, ethics, cultural beliefs, and the demands of responsible
parenthood.
Considering the close intimacy between “reproductive health” and “responsible parenthood” which bears to the
attainment of the goal of achieving “sustainable human development” as stated under its terms, the Court finds no
reason to believe that Congress intentionally sought to deceive the public as to the contents of the assailed legislation.

Accordingly, the Court declares R.A. No. 10354 as NOT UNCONSTITUTIONAL except with respect to the following provisions which are declared
UNCONSTITUTIONAL:
1) Section 7 and the corresponding provision in the RH-IRR insofar as they: a) require private health facilities and non-maternity specialty hospitals and hospitals owned
and operated by a religious group to refer patients, not in an emergency or life-threatening case, as defined under Republic Act No. 8344, to another health facility which
is conveniently accessible; and b) allow minor-parents or minors who have suffered a miscarriage access to modem methods of family planning without written consent
from their parents or guardian/s;
2) Section 23(a)(l) and the corresponding provision in the RH-IRR, particularly Section 5 .24 thereof, insofar as they punish any healthcare service provider who fails and
or refuses to disseminate information regarding programs and services on reproductive health regardless of his or her religious beliefs.
3) Section 23(a)(2)(i) and the corresponding provision in the RH-IRR insofar as they allow a married individual, not in an emergency or life-threatening case, as defined
under Republic Act No. 8344, to undergo reproductive health procedures without the consent of the spouse;
4) Section 23(a)(2)(ii) and the corresponding provision in the RH-IRR insofar as they limit the requirement of parental consent only to elective surgical procedures.
5) Section 23(a)(3) and the corresponding provision in the RH-IRR, particularly Section 5.24 thereof, insofar as they punish any healthcare service provider who fails
and/or refuses to refer a patient not in an emergency or life-threatening case, as defined under Republic Act No. 8344, to another health care service provider within the
same facility or one which is conveniently accessible regardless of his or her religious beliefs;
6) Section 23(b) and the corresponding provision in the RH-IRR, particularly Section 5 .24 thereof, insofar as they punish any public officer who refuses to support
reproductive health programs or shall do any act that hinders the full implementation of a reproductive health program, regardless of his or her religious beliefs;
7) Section 17 and the corresponding prov1s10n in the RH-IRR regarding the rendering of pro bona reproductive health service in so far as they affect the conscientious
objector in securing PhilHealth accreditation; and
8) Section 3.0l(a) and Section 3.01 G) of the RH-IRR, which added the qualifier “primarily” in defining abortifacients and contraceptives, as they are ultra vires and,
therefore, null and void for contravening Section 4(a) of the RH Law and violating Section 12, Article II of the Constitution.

Case no. 089


Obergefell vs. Hodges
576 U.S. June 26, 2015
Case Digest by Atty. Tal

FACTS:
● Obergefell v. Hodges is a landmark civil rights case in which the SC of U.S ruled that the fundamental right to marry is guaranteed to
same-sex couples by both the Due Process Clause and the Equal Protection Clause of the 14th Amendment to the U.S. Constitution.
○ The 5–4 ruling requires all 50 states to perform and recognize the marriages of same-sex couples on the same terms &
conditions as marriages of opposite-sex couples, with all the accompanying rights and responsibilities
CONSTI I - CHAPTER 5
CASE DIGESTS
● John died three months later of amyotrophic lateral sclerosis but Ohio law prevented James being listed on John’s death certificate
as surviving spouse on the grounds that it didn’t recognize the marriage, Obergefell sued and won. Ohio appealed and won the second
round. By the time national lawyers recognized the case as a legal bombshell, several other couples with similar dilemmas had stepped
forward, joining Obergefell in appealing to the Supreme Court.
● Same-sex partners April DeBoer and Jayne Rowse have three adopted children; however, Michigan permits only opposite-sex
married couples or single persons to adopt, with the result that each child is treated as having only one parent, and if that partner
passed away, the other would have no legal right to the children.
● Ijpe DeKoe and Thomas Kostura married in New York, where same-sex marriage was legal, before Ijpe was deployed to Afghanistan.
Upon his return, they settled in Tennessee where their marriage is not recognised, with the result that their legal status in relation
to each other changes as they travel between states.
● The petitioners were 2 men whose same-sex partners had died and 14 same-sex couples who all brought
cases in their respective District Courts challenging either the denial of their right to marry or the right to have
their marriage performed elsewhere recognized in their own state. The cases were heard in Michigan,
Kentucky, Ohio and Tennessee, each of which defines marriage as between a woman and a man. In each
case, the relevant District Court found in favour of the petitioner. Each of the respondents, who were state
officials responsible for enforcing the relevant laws, appealed. The Court of Appeals for the Sixth Circuit
consolidated the respondents’ appeals and reversed the decisions, finding in favour of the respondents. The
petitioners then sought certiorari in the Supreme Court.
Justice Anthony Kennedy asserted that the right to marry is a fundamental right “inherent in the liberty of the
person” and is therefore protected by the due process clause, which prohibits the states from depriving any
person of “life, liberty, or property without due process of law.”
Section 1 of the Fourteenth Amendment to the United States Constitution: (…) nor shall any state deprive any
person of life, liberty, or property, without due process of law; nor deny to any person within its jurisdiction the equal
protection of the laws.
● Each petitioner sought certiorari arguing that the actions of the relevant respondent violated the 14 th
Amendment by either denying them their right to marry, or by denying the recognition of their marriage legally
performed in another state.
● The petitioners argued that rather than intending to devalue marriage, it was their respect for the institution of
marriage which meant that they sought it for themselves.
Respondents argued that the petitioners did not seek recognition of the right to marry, but sought recognition of a
new and non-existent “right to same-sex marriage”.
● They argued that marriage was by nature between a man and a woman and recognition of same-sex
marriage would demean the institution of marriage.
● Further, the respondents warned that there had not been sufficient democratic discourse to decide on an
issue as important as the definition of marriage.
● In addition, they argued that if same-sex couples are allowed to marry, fewer opposite sex couples would
marry because the connection between marriage and procreation would be severed.
ISSUES:
1. The constitutionality of same-sex marriage bans (the “marriage question”)
2. The constitutionality of bans on recognizing same-sex marriages (the “recognition” question)
HELD:
● The Court ruled by a majority of 5 to 4 in favor of the petitioners. The majority decision, delivered by Justice
Kennedy first considered the issue arising from the cases from Michigan and Kentucky, of whether states are
required to register same-sex marriages.
● The majority began by noting that the institution of marriage has evolved over time both legally and socially
and that the states were now divided on the issue of same-sex marriage, before turning to consider the Due
Process Clause nor shall any state deprive any person of life, liberty, or property, without due process of law”.
● The liberties protected by this Clause extend to choices that are central to a person’s dignity and autonomy,
including intimate choices about personal beliefs and identity.
● Injustice is not always recognized in our own times and when new insights reveal a conflict between
Constitutional provisions and legislation, the Court must consider a claim to liberty.
○ Applying these considerations, the Court has long recognized that the Constitution protected the right to marry, including in
Loving v Virginia 388 US 1, 12 (1967), in which the Court invalidated bans on interracial marriage.
Although these previous cases concerned opposite-sex marriages, they established more far-reaching constitutional
principles, including four essential principles relating to the right to marry:
1. The right to personal choice in relation to marriage as an inherent aspect of an individual’s autonomy
2. The importance of the union of marriage to the two individuals which was “unlike any other”
3. Marriage provides a safeguard for children and families
CONSTI I - CHAPTER 5
CASE DIGESTS
4. Marriage was central to social order, with states offering married couples rights, benefits and responsibilities.
● Each of these principles applies equally to same-sex marriages and while limiting marriage may have previously been seen as just and
natural, it is now manifest that limiting marriage to opposite-sex partners is inconsistent with the “central meaning of the right to marry”.
Such knowledge must lead to recognition that banning of same-sex marriage imposes “stigma and injury of the kind prohibited by our
basic charter.”
Equal Protection Clause. In interpreting this Clause, the Court has “recognized that new insights and societal
understandings can reveal unjustified inequality within our most fundamental institutions that once passed unnoticed
and unchallenged”.
● The marriage laws challenged by the petitioners are “in essence unequal”. They denied same-sex couples all
the benefits granted to opposite-sex couples and work as a “grave and continuing harm”, serving to disrespect
and subordinate gays and lesbians.
RESPONDENTS
● The issue is whether the Constitution protects the right to same-sex marriage and not whether same-sex
marriage currently has or lacks popular support.
● The respondents showed no foundation to conclude that recognizing same-sex marriage would harm the
institution of marriage.
Recognition of Same-Sex Marriages in Other States
If the current state of affairs is left in place (being married in one place but not another), the result would be instability
and uncertainty. It follows from the decision that same-sex couples may marry in all states, that there is no lawful
basis on which a state can refuse to recognize a same-sex marriage lawfully performed in another state because of its
same-sex character.

Section 13

SECTION 13. The State recognizes the vital role of the youth in nation-building and shall promote and
protect their physical, moral, spiritual, intellectual, and social well-being. It shall inculcate in the youth patriotism and
nationalism, and encourage their involvement in public and civic affairs.

Case no. 090

Francisco Virtuoso Jr. v. Municipal Judge of Mariveles, Bataan


G.R. No. L-47841. March 21, 1978.
Case Digest by: Atty. Tal
FACTS:
Francisco Virtouso, Jr- application for the writ of habeas corpus on February 23, 1978, premised his plea for liberty
primarily on the ground that the preliminary examination which led to the issuance of a warrant of arrest against him
was a useless formality
● Petitioner is a seventeen-year old minor entitled to the protection and benefits of the Child and Youth Welfare
Code, a youthful offender being deemed therein as "one who is over nine years but under eighteen years of
age at the time of the commission of the offense." As such, he could be provisionally released on
recognizance in the discretion of a court
Respondent Municipal Judge of Mariveles, Bataan
1. failed to meet the strict standard required by the Constitution to ascertain whether there was a probable cause.
2. alleged that aside from the constitutional infirmity that tainted the procedure followed in the preliminary examination,
the bail imposed was clearly excessive.
3 It was in the amount of P16,000.00, the alleged robbery of a TV set being imputed to petitioner. As prayed for, the
Court issued a writ of habeas corpus, returnable to it on Wednesday, March 15, 1978.
● Respondent Judge, in his return led on March 8, 1978, justified the issuance of the warrant of arrest, alleging
that there was no impropriety in the way the preliminary examination was conducted.
Excessive character of the bail- he asserted that it was fixed in accordance with the Revised Bail Bond Guide
issued by the Executive Judge of Bataan in 1977, he nevertheless reduced the amount to P8,000.00
HELD:
● Petition resolved, without the need of passing upon the issue of whether or not the procedure by respondent
Judge in ascertaining the existence of probable cause was constitutionally deficient. Nonetheless, it must ever
CONSTI I - CHAPTER 5
CASE DIGESTS
be kept in mind by occupants of the bench that they should always be on the alert lest by sloth or indifference
or due to the economic or social standing of the alleged offended party, as was intimated in this petition, the
rights of an accused, instead of being honored, are disregarded.
● There is much more importance attached to the immunities of an individual during a period of martial law,
which in itself is a creature of the Constitution as a mode of coping with grave emergency situations.
● It is equally pertinent to state that there should be fealty to the constitutional ban against excessive bail being
required. Where, however, the right to bail exists, it should not be rendered nugatory by requiring a sum that
is excessive. The sole permissible function of money bail is to assure the accused's presence at trial, and
declared that "bail set at a higher figure than an amount reasonably calculated to fulfill this purpose is
excessive.

Section 14

SECTION 14. The State recognizes the role of women in nation-building, and shall ensure the fundamental
equality before the law of women and men.

Case no. 091


PT&T Co. v. NLRC
GR No. 118978, May 23, 1997
Case Digest by: Atty. Tal
FACTS:
● Grace de Guzman was hired by PT&T as a Supernumerary Project Worker for a fixed period from November
21, 1990 until April 20, 1991 as reliever for C.F. Tenorio who went on maternity leave.
● On September 2, 1991, Grace was asked to join petitioner company as a probationary employee. In the job
application form furnished to Grace, she indicated in the civil status that she was single although she
had in fact contracted marriage on May 26, 1991.
● Petitioner dismissed Grace from the company after learning about Grace’s real civil status and being
unconvinced of Grace’s explanation for the discrepancy.
● Grace immediately filed a complaint for illegal dismissal coupled with a claim for non-payment of cost of living
allowances (COLA), before the Regional Arbitration Branch of the National Labor Relations Commission
(NLRC) in Baguio City.
● At the preliminary conference, Grace volunteered the information that she had failed to remit the amount of
P2,380.75 of her collections, and executed a promissory note for that amount
● The Labor Arbiter handed down a decision declaring that private respondent, who had already gained the
status of a regular employee, was illegally dismissed by petitioner and ordered her reinstatement plus
payment of the corresponding back wages and COLA.
● On appeal, the NLRC upheld the Labor Arbiter but modified the Labor Arbiter’s decision with the qualification
that Grace de Guzman deserved to be suspended for three months due to the dishonest nature of her acts
which should not be condoned.
HELD:
The Constitution, cognizant of the disparity in rights between men and women in almost all phases of social and
political life, provides a gamut of protective provisions. Section 14, Article II. It mandates that the State shall protect
working women through provisions for opportunities that would enable them to reach their full potential.
1. EMPLOYER'S POLICY OF NOT ACCEPTING FOR WORK ANY WOMAN WORKER WHO CONTRACTS
MARRIAGE, CONTRARY TO LAW, GOOD MORALS AND PUBLIC POLICY. Petitioner's policy is not only in
derogation of the provisions of Article 136 of the Labor Code on the right of a woman to be free from any kind of
stipulation against marriage in connection with her employment, but it likewise assaults good morals and public policy,
tending as it does to deprive a woman of the freedom to choose her status, a privilege that by all accounts inheres in
the individual as an intangible and inalienable right.
● Hence, while it is true that the parties to a contract may establish any agreements, terms, and conditions that
they may deem convenient the same should not be contrary to law, morals, good customs, public order, or
public policy.
CONSTI I - CHAPTER 5
CASE DIGESTS
2. DISMISSAL; LOSS OF CONFIDENCE, VALID GROUND. — While loss of confidence is a just cause of
termination of employment, it should not be simulated. It must rest on an actual breach of duty committed by the
employee and not on the employer's caprices.
3. CONCEALMENT OF FEMALE EMPLOYEE OF TRUE NATURE OF STATUS FOR FEAR OF BEING
DISQUALIFIED FROM WORK, NOT SUFFICIENT BASIS. Private respondent's act of concealing the true nature of
her status from PT & T could not be properly characterized as willful or in bad faith as she was moved to act the way
she did mainly because she wanted to retain a permanent job in a stable company. She was forced by that very same
illegal company policy into misrepresenting her civil status for fear of being disqualified from work.
4.FAILURE TO REMIT COMPANY FUNDS, NOT AN ADDITIONAL GROUND; CASE AT BAR. — Private
respondent admitted in the course of the proceedings that she failed to remit some of her collections, but that is an
altogether different story. The labor arbiter would thus consider petitioner's submissions on this a mere afterthought,
just too bolster its supposed dishonesty as case for dismissal, is a perceptive conclusion born of experience in labor
cases. (There was a promissory note)
5. EMPLOYEE ILLEGALLY DISMISSED A FEW DAYS BEFORE COMPLETION OF HER PROBATIONARY
EMPLOYMENT AND WHO WAS PREVIOUSLY HIRED RELIEVER FOR SEVERAL TIMES GAINED REGULAR
STATUS. — Her earlier stints with the company as reliever were undoubtedly those of a regular employee, even if the
same were for fixed periods, as she performed activities which were essential or necessary in the usual trade and
business of PT & T.
6. ENTITLED TO REINSTATEMENT WITHOUT LOSS OF SENIORITY RIGHTS AND OTHER PRIVILEGES. As an
employee who had therefore gained regular status, and as she had been dismissed without just cause, she is entitled
to reinstatement without loss of seniority rights and other privileges and to full back wages, inclusive of allowances
and other benefits or their monetary equivalent.
7. OBLIGATIONS AND CONTRACTS; CONTRACT OF LABOR; IMPRESSED WITH PUBLIC POLICY SHOULD
NOT CONTAIN OPPRESSIVE PROVISIONS NOR IMPAIR THE INTEREST OR CONVENIENCE OF THE PUBLIC;
POLICY AGAINST MARRIAGE, A PROHIBITED PROVISION.

Section 15

SECTION 15. The State shall protect and promote the right to health of the people and instill health
consciousness among them."

Section 16

SECTION 16. The State shall protect and advance the right of the people to a balanced and healthful
ecology in accord with the rhythm and harmony of nature.

Case no. 092


Oposa vs. Factoran
G.R. No. 101083, July 30, 1993
Case Digest By: Atty. Tal
FACTS:
The principal plaintiffs therein, are all minors duly represented and joined by their respective parents. Impleaded as an
additional plaintiff is the Philippine Ecological Network, Inc. (PENI), a domestic, non-stock and non-profit corporation
organized for the purpose of, inter alia, engaging in concerted action geared for the protection of our environment and
natural resources.
The original defendant was the Honorable Fulgencio S. Factoran, Jr., then Secretary of the Department of
Environment and Natural Resources (DENR). His substitution in this petition by the new Secretary, the Honorable
Angel C. Alcala, was subsequently ordered upon proper motion by the petitioners. The complaint was instituted as a
taxpayers' class suit and alleges that the plaintiffs "are all citizens of the Republic of the Philippines, taxpayers, and
entitled to the full benefit, use and enjoyment of the natural resource treasure that is the country's virgin tropical
rainforests."
CONSTI I - CHAPTER 5
CASE DIGESTS
● Plaintiffs prayed that judgement be rendered ordering the respondent, his agents, representatives and other
persons acting in his behalf to cancel all existing Timber License Agreement (TLA) in the country and to
cease and desist from receiving, accepting, processing, renewing or approving new TLAs.
● Defendant, on the other hand, filed a motion to dismiss on the ground that the complaint had no cause of
action against him and that it raises a political question.
● The RTC Judge sustained the motion to dismiss, further ruling that granting of the relief prayed for would
result in the impairment of contracts which is prohibited by the Constitution.
● Petitioners filed the instant special civil action for certiorari and asked the court to set aside the dismissal
order on the ground that the respondent RTC Judge gravely abused his discretion in dismissing the action.
HELD:
1. DECLARATION OF PRINCIPLES AND STATE POLICIES; RIGHT TO A BALANCED AND HEALTHFUL
ECOLOGY, CONSTRUED. — The complaint focuses on one specific fundamental legal right — the right to a
balanced and healthful ecology which, for the first time in our nation's constitutional history, is solemnly incorporated in
the fundamental law.
● Art 15 unites with Art 16 of Sec.2 of the 1987 Constitution. While the right to a balanced and healthful ecology
is to be found under the Declaration of Principles and State Policies and not under the Bill of Rights, it does
not follow that it is less important than any of the civil and political rights enumerated in the latter. Such a right
belongs to a different category of rights altogether for it concerns nothing less than self-preservation and self-
perpetuation - the advancement of which may even be said to predate all governments and constitutions.
○ It is because of the well-founded fear of its framers that unless the rights to a balanced and healthful
ecology and to health are mandated as state policies by the Constitution itself. The right to a balanced
and healthful ecology carries with it the correlative duty to refrain from impairing the environment.
2. TIMBER LICENSES; NON-IMPAIRMENT CLAUSE MAY NOT BE INVOKED- Licenses may thus be revoked or
rescinded by executive action. It is not a contract, property or a property right protected by the due process clause of
the Constitution.
● "A timber license is an instrument by which the State regulates the utilization and disposition of forest
resources to the end that public welfare is promoted. A timber license is not a contract within the purview of
the due process clause; it is only a license or privilege, which can be validly withdrawn whenever dictated by
public interest or public welfare as in this case.
● A license is merely a permit or privilege to do what otherwise would be unlawful, and is not a contract
between the authority, federal, state, or municipal, granting it and the person to whom it is granted; neither is it
property or a property right, nor does it create a vested right; nor is it taxation. Thus, this Court held that the
granting of license does not create irrevocable rights, neither is it property or property rights.
● "Since timber licenses are not contracts, the non-impairment clause, which reads: “SEC. 10. No law
impairing the obligation of contracts shall be passed." The instant case does not involve a law or even an
executive issuance declaring the cancellation or modification of existing timber licenses. Hence, the non-
impairment clause cannot as yet be invoked. Nevertheless, granting further that a law has actually been
passed mandating cancellations or modifications, the same cannot still be stigmatized as a violation of the
non-impairment clause. Such a law could have only been passed in the exercise of the police power of the
state for the purpose of advancing the right of the people to a balanced and healthful ecology, promoting their
health and enhancing the general welfare.
● "The freedom of contract, under our system of government, is not meant to be absolute. The same is
understood to be subject to reasonable legislative regulation aimed at the promotion of public health, moral,
safety and welfare. The constitutional guaranty of non-impairment of obligations of contract is limited by the
exercise of the police power of the State, in the interest of public health, safety, moral and general welfare.
3. JUDICIAL REVIEW; NO LONGER IMPAIRED BY THE POLITICAL QUESTION DOCTRINE- Political Question
doctrine is no longer the insurmountable obstacle to the exercise of judicial power or the impenetrable shield that
protects executive and legislative actions from judicial inquiry or review.
● The power to rule upon even the wisdom of the decisions of the executive and the legislature and to declare
their acts invalid for lack or excess of jurisdiction because tainted with grave abuse of discretion. The catch is
the meaning of grave abuse of discretion, which is a very elastic phrase that can expand or contract according
to the disposition of the judiciary."
4. CAUSE OF ACTION— the right of the petitioners to a balanced and healthful ecology is as clear as the DENR's
duty under its mandate and by virtue of its powers and functions under E.O. No. 192 and the Administrative Code of
1987 — to protect and advance the said right. A denial or violation of that right by the other who has the correlative
duty or obligation to respect or protect the same gives rise to a cause of action.
CONSTI I - CHAPTER 5
CASE DIGESTS
● Petitioners maintain that the granting of the TLAs, which they claim was done with grave abuse of discretion,
violated their right to a balanced and healthful ecology; hence, the full protection thereof requires that no
further TLAs should be renewed or granted.
5. MOTION TO DISMISS; LACK OF CAUSE OF ACTION, AS A GROUND- The question submitted to the court for
resolution involves the sufficiency of the facts alleged in the complaint itself. No other matter should be considered;
furthermore, the truth or falsity of the said allegations is beside the point for the truth thereof is deemed hypothetically
admitted. The only issue to be resolved in such a case is: May the court render a valid judgment in accordance
with the prayer in the complaint? After a careful examination of the petitioners' complaint, we find the statements
under the introductory affirmative allegations, as well as the specific averments under the subheading CAUSE OF
ACTION, to be adequate enough to show the claimed violation of their rights.

Case no. 093


LAGUNA LAKE DEVELOPMENT AUTHORITY (LLDA) vs. COURT OF APPEALS
G.R. No. 110120 March 16, 1994
Case Digest by: Atty. Tal

FACTS:
● The clash between the responsibility of the City Government of Caloocan to dispose the 350 tons of garbage
it collects daily and the growing concern and sensitivity to a pollution-free environment of the residents of
Barangay Camarin, Tala Estate, Caloocan City where these tons of garbage are dumped every day is the hub
of this controversy elevated by the protagonists to the LLDA for adjudication.
● LLDA was created through Republic Act No. 4850. It was granted, inter alia, exclusive jurisdiction to issue
permits for the use of all surface water for any project or activity in or affecting the said region including
navigation, construction, and operation of fish pens, fish enclosures, fish corrals and the like.
● RA 7160, the Local Government Code of 1991. The municipalities in the Laguna Lake region interpreted its
provisions to mean that the newly passed law gave municipal governments the exclusive jurisdiction to issue
fishing privileges within their municipal waters.
● On July, 19, 1993, the Court issued a temporary restraining order enjoining the City Mayor of Caloocan and/or
the City Government of Caloocan to cease and desist from dumping its garbage at the Tala Estate, Barangay
Camarin, Caloocan City, effective as of this date and containing until otherwise ordered by the Court.
● Both parties agree on the need to protect the environment and to maintain the ecological balance of the
surrounding areas of the Camarin open dumpsite, the question as to which agency can lawfully exercise
jurisdiction over the matter remains highly open to question.
● The City Government of Caloocan claims that it is within its power, as an LGU, pursuant to the general
welfare provision of the Local Government Code, to determine the effects of the operation of the dumpsite on
the ecological balance and to see that such balance is maintained.
HELD:
● The Court of Appeals sustained the position of the City of Caloocan on the theory that Section 7 of
Presidential Decree No. 984, otherwise known as the Pollution Control law, authorizing the defunct National
Pollution Control Commission to issue an ex-parte cease and desist order was not incorporated in PD No. 813
nor in EO No. 927, series of 1983.
● The Court of Appeals ruled that under Section 4, par. (d), of Republic Act No. 4850, as amended, the LLDA is
instead required "to institute the necessary legal proceeding against any person who shall commence to
implement or continue implementation of any project, plan or program within the Laguna de Bay region
without previous clearance from the Authority."
○ It must be recognized in this regard that the LLDA, as a specialized administrative agency,
is specifically mandated under Republic Act No. 4850 and its amendatory laws to carry out and
make effective the declared national policy of promoting and accelerating the development and
balanced growth of the Laguna Lake area and the surrounding provinces of Rizal and
Laguna and the cities of San Pablo, Manila, Pasay, Quezon and Caloocan with due regard and
adequate provisions for environmental management and control, preservation of the quality of
human life and ecological systems, and the prevention of undue ecological disturbances,
deterioration and pollution.
CONSTI I - CHAPTER 5
CASE DIGESTS
○ LLDA, by virtue of its special charter, obviously has the responsibility to protect the inhabitants of the
Laguna Lake region from the deleterious effects of pollutants emanating from the discharge of
wastes from the surrounding areas.
● The matter of determining whether there is such pollution of the environment that requires control, if not
prohibition, of the operation of a business establishment is essentially addressed to the Environmental
Management Bureau (EMB) of the DENR which, by virtue of Section 16 of Executive Order No. 192, series
of 1987,18 has assumed the powers and functions of the defunct National Pollution Control Commission
created under Republic Act No. 3931.
○ Under said Executive Order, a Pollution Adjudication Board (PAB) under the Office of the DENR
Secretary now assumes the powers and functions of the National Pollution Control Commission with
respect to adjudication of pollution cases.
● While it is a fundamental rule that an administrative agency has only such powers as are expressly granted
to it by law , it is likewise a settled rule that an administrative agency has also such powers as are
necessarily implied in the exercise of its express powers. In the exercise, therefore, of its express
powers under its charter as a regulatory and quasi-judicial body with respect to pollution cases in the
Laguna Lake region, the authority of the LLDA to issue a "cease and desist order" is, implied.

Section 17

SECTION 17. The State shall give priority to education, science and technology, arts, culture, and sports to
foster patriotism and nationalism, accelerate social progress, and promote total human liberation and development.

Case no. 094


TEOFISTO T. GUINGONA, JR. and AQUILINO Q. PIMENTEL, JR vs.
HON. GUILLERMO CARAGUE, in his capacity as Secretary, Budget & Management,
G.R. No. 94571 April 22, 1991
Case Digest by: Atty. Tal

FACTS:
● The 1990 budget consists of P98.4 Billion in automatic appropriation (with P86.8 Billion for debt service) and
P155.3 Billion appropriated under R.A. No. 6831,the General Appropriations Act, or a total of P233.5 Billion
● The appropriations for the Department of Education, Culture and Sports amount to P27,017,813,000.00. The
said automatic appropriation for debt service is authorized by P.D. No. 81, entitled "Amending Certain
Provisions of Republic Act Numbered Four Thousand Eight Hundred Sixty, as Amended (Re: Foreign
Borrowing Act), "by P.D. No. 1177, entitled "Revising the Budget Process in Order to Institutionalize the
Budgetary Innovations of the New Society," and by P.D. No. 1967, entitled "An Act Strengthening the
Guarantee and Payment Positions of the Republic of the Philippines on Its Contingent Liabilities Arising out of
Relent and Guaranteed Loans by Appropriating Funds For The Purpose."
● The petition seeks the declaration of the unconstitutionality of P.D. No. 81, Section 31 of P.D. No. 1177, and
P.D. No. 1967. The petition also seeks to restrain the disbursement for debt service under the 1990 budget
pursuant to said decrees. Respondents contend that the petition involves a pure political question which is the
repeal or amendment of said laws addressed to the judgment, wisdom and patriotism of the legislative body
and not this Court.
● R.A. 4860 entitled “An act authorizing the president of the Philippines to obtain such foreign loans and credits,
or to incur such foreign indebtedness, as may be necessary to finance approved economic development
purposes or projects, and to guarantee, in behalf of the R.P., foreign loans obtained or bonds issued by
corporations owned or controlled by the government of the Philippines for economic development purposes
including those incurred for purposes of relending to the private sector, appropriating the necessary funds
,and for other purposes.
ISSUES:
1. IS THE APPROPRIATION OF P86 BILLION IN THE P233 BILLION 1990 BUDGET VIOLATIVE OF SECTION 5,
ARTICLE XIV OF THE CONSTITUTION?
2. ARE PD NO. 81, PD NO. 1177 AND PD NO. 1967 STILL OPERATIVE UNDER THE CONSTITUTION?
3. ARE THEY VIOLATIVE OF SECTION 29(1), ARTICLE VI OF THE CONSTITUTION?"
CONSTI I - CHAPTER 5
CASE DIGESTS

HELD:
1. SENATORS MAY BRING SUIT QUESTIONING THE CONSTITUTIONALITY OF THE AUTOMATIC
APPROPRIATION FOR DEBT SERVICE IN THE 1990 BUDGET - Petitioners as Senators of the Republic of the
Philippines may bring this suit where a constitutional issue is raised.
2.APPROPRIATION ACT OF 1990; HIGHEST BUDGETARY PRIORITY TO EDUCATION; ALLOCATION OF P86
BILLION TO EDUCATION, NOT UNCONSTITUTIONAL EVEN IF CONGRESS APPROPRIATED AN AMOUNT FOR
DEBT SERVICE BIGGER THAN THE SHARE ALLOCATED TO EDUCATION. The amount of P29,740,611,000.00
set aside for the Department of Education, Culture and Sports under the General Appropriations Act (R.A. No. 6831),
is the highest budgetary allocation among all department budgets; a clear compliance with the constitutional mandate
according highest priority to education.
● Having faithfully complied therewith, Congress is certainly not without any power, guided only by its good
judgment, to provide an appropriation, that can reasonably service our enormous debt, the greater portion of
which was inherited from the previous administration.
● Thus, if in the process Congress appropriated an amount for debt service bigger than the share allocated to
education, the Court finds and so holds that said appropriation cannot be thereby assailed as unconstitutional.
3. P.D. NO. 81, P.D NO. 1177 AND P.D. NO. 1967 REGARDING AUTOMATIC APPROPRIATIONS REMAIN
OPERATIVE UNTIL AMENDED, REPEALED OR REVOKED. Section 3, Article XVIII of the Constitution recognizes
that "All existing laws, decrees, executive orders, proclamations, letters of instructions and other executive issuances
not inconsistent with the Constitution shall remain operative until amended, repealed or revoked"
● The Court, finds that R.A. No. 4860, as amended by P.D. No. 81, Section 31 of P.D. 1177 and P.D. No. 1967
constitute lawful authorizations or appropriations, unless they are repealed or otherwise amended by
Congress.
4. CONSTRUCTION OF THE CONSTITUTION AND LAW IS GENERALLY APPLIED PROSPECTIVELY.
Constitution and law is generally applied prospectively and not retrospectively unless it is so clearly stated. 5.
LEGISLATIVE DEPARTMENT; UNDUE DELEGATION OF POWER; CONSTRUED. "What cannot be delegated is
the authority under the Constitution to make laws and to alter and repeal them; the test is the completeness of the
statute in all its terms and provisions when it leaves the hands of the legislature. To determine whether or not there is
an undue delegation of legislative power, the inequity must be directed to the scope and definiteness of the measure
enacted.
● To avoid the taint of unlawful delegation there must be a standard, which implies that the legislature itself
determines matters of principle and lays down fundamental policy. The standard may be either express or
implied from the policy and purpose of the act considered as whole
6. POWER TO MAKE THE LAW DISTINGUISHED FROM POWER AND DISCRETION AS TO ITS EXECUTION.
The true distinction is between the delegation of power to make the law, which necessarily involves discretion as to
what the law shall be, and conferring authority or discretion as to its execution, to be exercised under and in
pursuance of the law. The first cannot be done; to the latter no valid objection can be made."
7. WHETHER THE COUNTRY SHOULD HONOR ITS INTERNATIONAL DEBT, A POLITICAL QUESTION. WON
the country should honor its international debt, more especially the enormous amount that had been incurred by the
past administration, which appears to be the ultimate objective of the petition, is not an issue that is presented or
proposed to be addressed by the Court. Indeed, it is more of a political decision for Congress and the Executive to
determine in the exercise of their wisdom and sound discretion.
● The legislative intention in R.A. No. 4860, as amended, Section 31 of P.D. No. 1177 and P.D. No. 1967 is that
the amount needed should be automatically set aside in order to enable the Republic of the Philippines to pay
the principal, interest, taxes and other normal banking charges on the loans, credits or indebtedness incurred
as guaranteed by it when they shall become due without the need to enact a separate law appropriating funds
therefor as the need arises. The purpose of these laws is to enable the government to make prompt payment
and/or advances for all loans to protect and maintain the credit standing of the country.

Section 18 and 20

SECTION 18.The State affirms labor as a primary social economic force. It shall protect the rights of workers and
promote their welfare.
SECTION 20. The State recognizes the indispensable role of the private sector, encourages private enterprise, and
CONSTI I - CHAPTER 5
CASE DIGESTS
provides incentives to needed investments.

In relation with:
Article XIII, Sec. 3. The State shall afford full protection to labor, local and overseas, organized and unorganized,
and promote full employment opportunities for all.

Case no. 095


PASEI vs. Drilon
G.R. No. 81958, June 30, 1988
Executive Secretary Kiezlle Manalili
FACTS:
Philippine Association of Service Exporters, Inc. (PASEI) challenges the Constitutional validity of Department
Order No. 1, Series of 1988, of the Department of Labor and Employment, in the character of "GUIDELINES
GOVERNING THE TEMPORARY SUSPENSION OF DEPLOYMENT OF FILIPINO DOMESTIC AND HOUSEHOLD
WORKERS," in this petition for certiorari and prohibition. Specifically, the measure is assailed for "discrimination
against males or females;" that it "does not apply to all Filipino workers but only to domestic helpers and females with
similar skills;" and that it is violative of the right to travel. It is held likewise to be an invalid exercise of the lawmaking
power, police power being legislative, and not executive, in character.

Section 3, of Article XIII, of the Constitution. This provision was invoked by PASEI, b providing for worker
participation "in policy and decision-making processes affecting their rights and benefits as may be provided by law."

Department Order No. 1, it is contended, was passed in the absence of prior consultations. It is claimed, finally, to be
in violation of the Charter's non-impairment clause, in addition to the "great and irreparable injury" that PASEI
members face should the Order be further enforced.

ISSUE:
Whether or not the Department Order No. 1 in nature of the police power is valid under the Constitution?

HELD:
In the light of the foregoing, the petition must be dismissed.

As a general rule, official acts enjoy a presumed validity. In the absence of clear and convincing evidence to
the contrary, the presumption logically stands.

The petitioner has shown no satisfactory reason why the contested measure should be nullified. There is no
question that Department Order No. 1 applies only to "female contract workers," but it does not thereby make
an undue discrimination between the sexes. It is well-settled that "equality before the law" under the Constitution
does not import a perfect Identity of rights among all men and women. It admits of classifications, provided that (1)
such classifications rest on substantial distinctions; (2) they are germane to the purposes of the law; (3) they are not
confined to existing conditions; and (4) they apply equally to all members of the same class.

The Court is well aware of the unhappy plight that has befallen our female labor force abroad, especially domestic
servants, amid exploitative working conditions marked by physical and personal abuse. As precisely the caretaker of
Constitutional rights, the Court is called upon to protect victims of exploitation. In fulfilling that duty, the Court sustains
the Government's efforts.

The same, however, cannot be said of our male workers. In the first place, there is no evidence that, except perhaps
for isolated instances, our men abroad have been afflicted with an identical predicament. Suffice it to state, then, that
insofar as classifications are concerned, this Court is content that distinctions are borne by the evidence.
Discrimination in this case is justified.

There is likewise no doubt that such a classification is germane to the purpose behind the measure. Unquestionably, it
is the avowed objective of Department Order No. 1 to "enhance the protection for Filipino female overseas workers"
CONSTI I - CHAPTER 5
CASE DIGESTS
this Court has no quarrel that in the midst of the terrible mistreatment Filipina workers have suffered abroad, a ban on
deployment will be for their own good and welfare.

The Order does not narrowly apply to existing conditions. Rather, it is intended to apply indefinitely so long as those
conditions exist. This is clear from the Order itself ("Pending review of the administrative and legal measures, in the
Philippines and in the host countries . . ."), meaning to say that should the authorities arrive at a means impressed
with a greater degree of permanency, the ban shall be lifted.

It is incorrect to say that Department Order No. 1 prescribes a total ban on overseas deployment. From scattered
provisions of the Order, it is evident that such a total ban has not been contemplated.

The consequence the deployment ban has on the right to travel does not impair the right. The right to travel is subject,
among other things, to the requirements of "public safety," "as may be provided by law. Neither is there merit in the
contention that Department Order No. 1 constitutes an invalid exercise of legislative power. It is true that police power
is the domain of the legislature, but it does not mean that such an authority may not be lawfully delegated. As we have
mentioned, the Labor Code itself vests the Department of Labor and Employment with rule-making powers in the
enforcement whereof.

The non-impairment clause of the Constitution, invoked by the petitioner, must yield to the loftier purposes
targeted by the Government. Freedom of contract and enterprise, like all other freedoms, is not free from
restrictions, more so in this jurisdiction, where laissez faire has never been fully accepted as a controlling
economic way of life.

This Court understands the grave implications the questioned Order has on the business of recruitment. The concern
of the Government, however, is not necessarily to maintain profits of business firms. In the ordinary sequence of
events, it is profits that suffer as a result of Government regulation. The interest of the State is to provide a decent
living to its citizens. The Government has convinced the Court in this case that this is its intent. We do not find the
impugned Order to be tainted with a grave abuse of discretion to warrant the extraordinary relief prayed for.

Case no. 096


De Vera vs. NLRC
G.R. No. 93212, Nov. 22, 1990

FACTS:
Diosdado de Vera was employed as a mechanic in the weaving department of the Central Textile Mills, the herein
private respondent. He claims he was active in union activities. In the course of his employment, he had been
administered disciplinary sanctions for various offenses, including not wearing the ID tag in the company premises,
leaving his workplace without permission, habitual tardiness, reporting for work under the influence of liquor, and
damaging property in the performance of his duties. He also had a row with Jesus Ocampo, a fellow worker, whom he
hit in the head with a stick.

Central Textile Mills. The president of the CMC Textile Workers Union filed on its behalf and of the petitioner a
complaint for illegal dismissal a

ISSUE:

Section 19
SECTION 19. The State shall develop a self-reliant and independent national economy effectively controlled by
Filipinos.

Case no. 097


Tanada vs. Angara
CONSTI I - CHAPTER 5
CASE DIGESTS
G.R. No. 118295, May 2, 1997

FACTS

Republic of the Philippines joined World Trade Organization as a founding member with the goal of improving
Philippine access to foreign markets, especially its major trading partners, through the reduction of tariffs on its
exports. The President also saw in the WTO the opening of new opportunities for the services sector, the reduction of
costs and uncertainty associated with exporting and the attraction of more investments into the country.
Final Act Embodying the Results of the Uruguay Round of Multilateral Negotiations. On April 15, 1994,
respondent Navarro, then DTI Secretary, signed in Marrakesh, Morocco, signed this. On December 14, 1994, the
Senate concurred in the ratification of the President of the Philippines of the Agreement Establishing the WTO which
includes various agreements and associated legal instruments. On December 16, 1994,the President signed the
Instrument of Ratification.

ISSUES:
1. Whether the WTO Agreement violated the mandated economic nationalism by the Constitution
2. Whether the provisions of the WTO Agreement restricts and impairs Philippine sovereignty, specifically the
legislative power vested in the Congress
3. Whether the Senate concurrence in the WTO Agreement and its annexes but not in the other documents referred to
in the Final Act is defective and insufficient and thus constitutes abuse of discretion

RULING:

1. No. The Constitution did not intend to pursue an isolationist policy. It did not shut out foreign investments,
goods and services in the development of the Philippine economy. In fact, it allows an exchange on the basis of
equality and reciprocity, frowning only on foreign competition that is unfair. The constitutional policy of a self-reliant
and independent national economy does not necessarily rule out the entry of foreign investments, goods and services.
It contemplates neither economic seclusion nor mendicancy in the international community.

2. No. While sovereignty has traditionally been deemed absolute and all-encompassing on the domestic level, it is
however subject to restrictions and limitations voluntarily agreed to by the Philippines, expressly or impliedly, as a
member of the family of nations. Unquestionably, the Constitution did not envision a hermit-type isolation of the
country from the rest of the world. By the doctrine of incorporation, the country is bound by generally accepted
principles of international law, which are considered to be automatically part of our laws. A treaty engagement is not a
mere moral obligation on the parties. By their inherent nature, treaties really limit or restrict the absoluteness of
sovereignty. The Philippines has effectively agreed to limit the exercise of its sovereign powers of taxation, eminent
domain and police power. The underlying consideration in this partial sovereignty is the reciprocal commitment of the
other contracting states in granting the same privilege and immunities to the Philippines, its officials and its citizens.
The same reciprocity characterizes the same commitments under WTO-GATT. The point is that a portion of
sovereignty may be waived without violating the Constitution, based on the rationale that the Philippines adopts the
generally accepted principles of international law as part of the law of the land and adheres to the policy of
cooperation and amity with all nations.

3. No. The petitioners submit that concurrence in the WTO Agreement alone is flawed because it is in effect a
rejection of the Final Act. The Court held that a final act is an instrument which records the winding up of the
proceedings of a diplomatic conference and not the treaty itself. On the other hand, the WTO Agreement itself
expresses what multilateral agreements are deemed included as its integral parts. It should be added that the Senate
was well-aware of what it was concurring in as shown by the member’s deliberation.

Case no. 098


Garcia vs. Bureau of Investments
FACTS:
CONSTI I - CHAPTER 5
CASE DIGESTS
Bataan Petrochemical Corporation (BPC) A Taiwanese private corporation, applied for registration with the Board
of Investments (BOI) in February 1988 as a new domestic producer of petrochemicals in the Philippines. It originally
specified the province of Bataan as the site for the proposed investment but later submitted an amended application to
change the site to Batangas.
Unhappy with the change of the site, Congressman Enrique Garcia of the Second District of Bataan requested a
copy of BPC’s original and amended application documents. The BOI denied the request on the basis that the
investors in BPC had declined to give their consent to the release of the documents requested, and that Article 81 of
the Omnibus Investments Code protects the confidentiality of these documents absent consent to disclose. The BOI
subsequently approved the amended application without holding a second hearing or publishing notice of the
amended application. Garcia filed a petition before the Supreme Court.

HELD:
The Court ruled that the BOI violated Garcia’s Constitutional right to have access to information on matters of
public concern under Article III, Section 7 of the Constitution. The Court found that the inhabitants of Bataan had
an “interest in the establishment of the petrochemical plant in their midst [that] is actual, real, and vital because it will
affect not only their economic life, but even the air they breathe” . The Court also ruled that BPC’s amended
application was in fact a second application that required a new public notice to be filed and a new hearing to be held.

Although Article 81 of the Omnibus Investments Code provides that “all applications and their supporting documents
filed under this code shall be confidential and shall not be disclosed to any person, except with the consent of the
applicant,” the Court emphasized that Article 81 provides for disclosure “on the orders of a court of competent
jurisdiction”. The Court ruled that it had jurisdiction to order disclosure of the application, amended application, and
supporting documents filed with the BOI under Article 81, with certain exceptions.

The Court went on to note that despite the right to access information, “the Constitution does not open every door to
any and all information” because “the law may exempt certain types of information from public scrutiny” (p.4). Thus it
excluded “the trade secrets and confidential, commercial, and financial information of the applicant BPC, and matters
affecting national security” from its order (p.4). The Court did not provide a test for what information is excluded from
the Constitutional privilege to access public information, nor did it specify the kinds of information that BPC could
withhold under its ruling

Case no. 099


Espina vs. Zamora, G.R. No. 143855
Sept. 21, 2010 Section 21
FACTS:

Retail Trade Liberalization Act of 2000 Republic Act 8762. Signed by President Joseph Estrada on March 7,2000.
The questioned law allows the said foreign trade placing them under 4 categories. It also allows natural-born Filipino
citizens, who had lost their citizenship and now reside in the Philippines, to engage in the retail trade business with the
same rights as Filipino citizens.

Gerardo S. Espina Filed a case assailing the constitutionality of RA 8762 as it is a clear violation of Sec. 9,19, and 20
of Article II of the Constitution. The petitioners stressed that the presence of foreign nationals would result in alien
control and monopoly of the retail trade.

ISSUE: Whether or not RA 8762 is unconstitutional.

HELD: In other words, while Section 19, Article II of the 1987 Constitution requires the development of a self-
reliant and independent national economy effectively controlled by Filipino entrepreneurs, it does not impose a
policy of Filipino monopoly of the economic environment. The objective is simply to prohibit foreign powers or
interests from maneuvering our economic policies and ensure that Filipinos are given preference in all areas of
development.

Indeed, the 1987 Constitution takes into account the realities of the outside world as it requires the pursuit of a trade
policy that serves the general welfare and utilizes all forms and arrangements of exchange on the basis of equality
and reciprocity; and speaks of industries which are competitive in both domestic and foreign markets as well as of the
protection of Filipino enterprises against unfair foreign competition and trade practices. Thus, while the Constitution
CONSTI I - CHAPTER 5
CASE DIGESTS
mandates a bias in favor of Filipino goods, services, labor and enterprises, it also recognizes the need for
business exchange with the rest of the world on the bases of equality and reciprocity and limits protection of
Filipino enterprises only against foreign competition and trade practices that are unfair.

In other words, the 1987 Constitution does not rule out the entry of foreign investments, goods, and services.
While it does not encourage their unlimited entry into the country, it does not prohibit them either. In fact, it allows an
exchange on the basis of equality and reciprocity, frowning only on foreign competition that is unfair. The key, as in all
economies in the world, is to strike a balance between protecting local businesses and allowing the entry of foreign
investments and services.

More importantly, Section 10, Article XII of the 1987 Constitution gives Congress the discretion to reserve to Filipinos
certain areas of investments upon the recommendation of the NEDA and when the national interest requires. Thus,
Congress can determine what policy to pass and when to pass it depending on the economic exigencies. It can enact
laws allowing the entry of foreigners into certain industries not reserved by the Constitution to Filipino citizens. In this
case, Congress has decided to open certain areas of the retail trade business to foreign investments instead of
reserving them exclusively to Filipino citizens. The NEDA has not opposed such policy.

The control and regulation of trade in the interest of the public welfare is of course an exercise of the police power of
the State. A persons right to property, whether he is a Filipino citizen or foreign national, cannot be taken from him
without due process of law. In 1954, Congress enacted the Retail Trade Nationalization Act or R.A. 1180 that restricts
the retail business to Filipino citizens. In denying the petition assailing the validity of such Act for violation of the
foreigners right to substantive due process of law, the Supreme Court held that the law constituted a valid exercise of
police power. The State had an interest in preventing alien control of the retail trade and R.A. 1180 was reasonably
related to that purpose. That law is not arbitrary.

Here, to the extent that R.A. 8762, the Retail Trade Liberalization Act, lessens the restraint on the foreigners right to
property or to engage in an ordinarily lawful business, it cannot be said that the law amounts to a denial of the
Filipinos right to property and to due process of law. Filipinos continue to have the right to engage in the kinds of retail
business to which the law in question has permitted the entry of foreign investors.

Certainly, it is not within the province of the Court to inquire into the wisdom of R.A. 8762 save when it blatantly
violates the Constitution. But as the Court has said, there is no showing that the law has contravened any
constitutional mandate. The Court is not convinced that the implementation of R.A. 8762 would eventually lead to
alien control of the retail trade business. Petitioners have not mustered any concrete and strong argument to support
its thesis. The law itself has provided strict safeguards on foreign participation in that business. Thus

First, aliens can only engage in retail trade business subject to the categories above-enumerated; Second,
only nationals from, or juridical entities formed or incorporated in countries which allow the entry of Filipino
retailers shall be allowed to engage in retail trade business; and Third, qualified foreign retailers shall not be
allowed to engage in certain retailing activities outside their accredited stores through the use of mobile or
rolling stores or carts, the use of sales representatives, door-to-door selling, restaurants and sari-sari stores
and such other similar retailing activities.

Section 21
SEC 21. The State shall promote comprehensive rural development and agrarian reform.

Case no. 100


Assoc. of Small Landowners vs. Secretary
G.R. No. 78742 July 14, 1989

Facts:

These are four consolidated cases questioning the constitutionality of the Comprehensive Agrarian Reform Act (R.A.
No. 6657 and related laws i.e., Agrarian Land Reform Code or R.A. No. 3844).

Article XIII of the Constitution on Social Justice and Human Rights includes a call for the adoption by the State of an
agrarian reform program. The State shall, by law, undertake an agrarian reform program founded on the right of
farmers and regular farmworkers, who are landless, to own directly or collectively the lands they till or, in the case of
other farmworkers, to receive a just share of the fruits thereof.
CONSTI I - CHAPTER 5
CASE DIGESTS
RA 3844 was enacted in 1963. P.D. No. 27 was promulgated in 1972 to provide for the compulsory acquisition of
private lands for distribution among tenant-farmers and to specify maximum retention limits for landowners.

In 1987, President Corazon Aquino issued E.O. No. 228, declaring full land ownership in favor of the beneficiaries of
PD 27 and providing for the valuation of still unvalued lands covered by the decree as well as the manner of their
payment. In 1987, P.P. No. 131, instituting a comprehensive agrarian reform program (CARP) was enacted; later,
E.O. No. 229, providing the mechanics for its (PP131’s) implementation, was also enacted. Afterwhich is the
enactment of R.A. No. 6657, Comprehensive Agrarian Reform Law in 1988. This law, while considerably changing the
earlier mentioned enactments, nevertheless gives them suppletory effect insofar as they are not inconsistent with its
provisions.

● A petition alleging the constitutionality of PD No. 27, EO 228 and 229 and RA 6657. Subjects of the petition
are a 9-hectare and 5 hectare Riceland worked by four tenants. Tenants were declared full owners by EO228
as qualified farmers under PD 27. The petitioners now contend that President Aquino usurped the
legislature’s power.
● A petition by landowners and sugarplanters in Victoria’s Mill Negros Occidental against Proclamation 131 and
EO 229. Proclamation 131 is the creation of Agrarian Reform Fund with initial fund of P50 Billion.
● A petition by owners of land which was placed by the DAR under the coverage of Operation Land Transfer.
● A petition invoking the right of retention under PD 27 to owners of rice and corn lands not exceeding seven
hectares.

Issue: Whether or Not the aforementioned EO’s, PD, and RA were constitutional.

Held: The promulgation of PD 27 by President Marcos was valid in exercise of Police power and eminent
domain.

The power of President Aquino to promulgate Proc. 131 and EO 228 and 229 was authorized under Sec. 6 of the
Transitory Provisions of the 1987 Constitution. Therefore it is a valid exercise of Police Power and Eminent Domain.

RA 6657 is likewise valid. The carrying out of the regulation under CARP becomes necessary to deprive owners of
whatever lands they may own in excess of the maximum area allowed, there is definitely a taking under the power of
eminent domain for which payment of just compensation is imperative. The taking contemplated is not a mere
limitation of the use of the land. What is required is the surrender of the title and the physical possession of said
excess and all beneficial rights accruing to the owner in favour of the farmer.

A statute may be sustained under the police power only if there is concurrence of the lawful subject and the method.

Subject and purpose of the Agrarian Reform Law is valid, however what is to be determined is the method employed
to achieve it.

Case no. 101


Luz Farms vs. Sec. of Agrarian Reform
G.R. No. 86889, Dec. 4, 1990
Facts:
RA 6657. On 10 June 1988, was approved by the President of the Philippines, which includes, among others, the
raising of livestock, poultry and swine in its coverage.
Petitioner Luz Farms, a corporation engaged in the livestock and poultry business, avers that it would be adversely
affected by the enforcement of sections 3(b), 11, 13, 16 (d), 17 and 32 of the said law. Hence, it prayed that the said
law be declared unconstitutional. The mentioned sections of the law provides, among others, the product-sharing
plan, including those engaged in livestock and poultry business.
Luz Farms further argued that livestock or poultry raising is not similar with crop or tree farming. That the land is not
the primary resource in this undertaking and represents no more than 5% of the total investments of commercial
livestock and poultry raisers. That the land is incidental but not the principal factor or consideration in their industry.
Hence, it argued that it should not be included in the coverage of RA 6657 which covers “agricultural lands”.
CONSTI I - CHAPTER 5
CASE DIGESTS
Issue: Whether or not certain provisions of RA 6657 is unconstitutional for including in its definition of “Agriculture” the
livestock and poultry industry?
Ruling:

The Court held YES.

Looking into the transcript of the Constitutional Commission on the meaning of the word “agriculture”, it showed that
the framers never intended to include livestock and poultry industry in the coverage of the constitutionally mandated
agrarian reform program of the government. Further, Commissioner Tadeo pointed out that the reason why they used
the term “farmworkers” rather than “agricultural workers” in the said law is because “agricultural workers” includes the
livestock and poultry industry, hence, since they do not intend to include the latter, they used “farmworkers” to have
distinction. Hence, there is merit on the petitioner’s argument that the product-sharing plan applied to “corporate
farms” in the contested provisions is unreasonable for being consficatory and violative of the due process of law.

Case no. 102


Hacienda Luisita vs. P.A.R.C
G.R. No. 171101, July 5, 2011
Facts:
Section 31 of RA 6657 provides two alternative modalities which are either land or stock transfer. The stock
distribution scheme appeared to Tadeco’s preferred option. Tadeco organized Hacienda Luisita Incorporation (HLI) as
vehicle to facilities stock acquisition by the farmer beneficiaries (FWB). It assigned and conveyed to HLI 4,915.75
hectares agricultural land portion and other farm-related properties of Hacienda Luisita in exchange for HLI shares of
stock.

Supreme Court en banc voted 11-0 dismissing the petition filed by HLI Affirm with modifications the resolutions of
the Presidential Agrarian Reform Council (PARC) revoking Hacienda Luisita Inc. Stock Distribution Plan (SDP) and
placing the subject land in HL under compulsory coverage of the CARP of the government.

Thereafter, the SC voting 6-5 averred that there are operative facts that occurred in the premises. The SC thereafter
declared that the revocation of the SDP shall, by application of the operative fact principle, give the 5,296 qualified
Farmworkers Beneficiaries (FWB) to choose whether they want to remain as HLI stockholders or choose actual land
distribution. Considering the premises, DAR immediately scheduled a meeting regarding the effects of their choice
and therefrom proceeded to secret voting of their choice.

The parties, thereafter, filed their respective Motion for Reconsideration regarding the SC’s decision.

Issue:

1. Whether or not Sec. 31 of R.A. 6657 unconstitutional.


2. Whether or not the 10-year period prohibition on the transfer of awarded lands under RA 6657 lapsed on May
10, 1999, since Hacienda Luisita were placed under CARP coverage through the SDOA scheme on May 11,
1989, and thus the qualified FWBs should now be allowed to sell their land interests in Hacienda Luisita to
third parties, whether they have fully paid for the lands or not?
3. Whether or not qualified FWBs shall be entitled to the option of remaining as stockholder be reconsidered.

Ruling:

1. The SC said that the constitutionality of Sec. 31 of R.A. 6657 is not the lis mota of the case and it was not
raised at the earliest opportunity and did not rule on the constitutionality of the law;
2. The SC ruled that it has not yet lapsed on May 10, 1999, and qualified FWBs are not allowed to sell their land
interest in HL to third parties; That the start of the counting of the prohibitive period shall be ten years from the
issuance and registration of the Emancipation Patent (EP for brevity) or Certificate of Land Ownership Award
(CLOA for brevity), and considering that the EPs and CLOAs have not yet been issued, the prohibitive period
has not started yet.
CONSTI I - CHAPTER 5
CASE DIGESTS
3. The SC ruled in the affirmative, giving qualified FWBs the option to remain as stockholder. YES, the ruling in
the July 5, 2011 Decision that the qualified FWBs be given an option to remain as stockholders of HLI should
be reconsidered.
[The Court reconsidered its earlier decision that the qualified FWBs should be given an option to remain as
stockholders of HLI, inasmuch as these qualified FWBs will never gain control [over the subject lands] given the
present proportion of shareholdings in HLI. The Court noted that the share of the FWBs in the HLI capital stock is
[just] 33.296%. Thus, even if all the holders of this 33.296% unanimously vote to remain as HLI stockholders, which is
unlikely, control will never be in the hands of the FWBs. Control means the majority of [sic] 50% plus at least one
share of the common shares and other voting shares. Applying the formula to the HLI stockholdings, the number of
shares that will constitute the majority is 295,112,101 shares (590,554,220 total HLI capital shares divided by 2 plus
one [1] HLI share). The 118,391,976.85 shares subject to the SDP approved by PARC substantially fall short of the
295,112,101 shares needed by the FWBs to acquire control over HLI.]

The SC PARTIALLY GRANTED the motions for reconsideration of respondents PARC, et al., The 6,296 original
FWBs shall forfeit and relinquish their rights over the HLI shares of stock issued to them in favor of HLI. The HLI
Corporate Secretary shall cancel the shares issued to the said FWBs and transfer them to HLI in the stocks and
transfer book. The 4,206 non-qualified FWBs shall remain as stockholders of HLI.

Section 22
Case no. 103
Cruz vs. Sec. of DENR
G.R. No. 135383, Dec. 6, 2000
William

SECTION 22. The State recognizes and promotes the rights of indigenous cultural communities within
the framework of national unity and development.

FACTS:
R.A No. 3871: Also known as Indigenous Peoples Rights Act (IPRA), enumerates the rights of indigenous
peoples over ancestral domains which may include natural resources.
Petitioners assail the constitutionality of the following provisions of the IPRA and its Implementing Rules on
the ground that they amount to an unlawful deprivation of the States ownership over lands of the public
domain as well as minerals and other natural resources therein, in violation of the regalian doctrine
embodied in Section 2, Article XII of the Constitution.
Regalian Doctrine dictates that all lands of the public domain belong to the State, that the State is the
source of any asserted right to ownership of land and charged with the conservation of such patrimony.

ISSUE:
Whether or not some of the IPRA provisions are unconstitutional?

HELD:
No, the provisions of IPRA are not in conflict with the constitution. There is nothing in the law that grants to
the ICC/IP ownership over the natural resources within their ancestral domain. Ownership over the natural
resources in the ancestral domains remains with the State and the rights grated by the IPRA to the ICC/IP
over the natural resources in their ancestral domains merely gives them, as owners and occupants of the
land on which the resources are found, the right to small scale utilization of these resources, and at the
same time, a priority in their large scare development and exploitation.

The SC deliberated upon the matter. After deliberation they voted and reached a 7-7 vote. They deliberated
again and the same result transpired. Since there was no majority vote, Cruz’s petition was dismissed and
the IPRA law was sustained.
CONSTI I - CHAPTER 5
CASE DIGESTS

Section 25
Case no. 104
Basco vs. PAGCOR
G. R. No. 91649, May 14, 1991
William

SECTION 25. The State shall ensure the autonomy of local governments.

FACTS:
Petitioners seek to annul the Philippine Amusement and Gaming Corporation (PAGCOR) Charter - PD
1869, because it is allegedly contrary to morals, public policy and order. Petitioners also claim that said PD
has a "gambling objective" and that Section 13 par 2 of the same PD which exempts PAGCOR from paying
any tax, any kind of term income or otherwise as well as fees, charges as levies of whatever nature
whether national or local is violative of the principles of local autonomy for it is a waiver of the right of the
City of Manila to impose taxes and legal fees.

ISSUE:
Whether or not the local autonomy clause is violated by PD 1869 – PAGCOR Charter.

HELD:
The petitioner‘s contentions are without merit.
The City of Manila, being a mere Municipal corporation has no inherent right to impose taxes. The Charter
of the City of Manila is subject to control by Congress. It should be stressed that:

"municipal corporations are mere creatures of Congress" which has the power to "create and abolish municipal
corporations" due to its "general legislative powers". Congress, therefore, has the power of control over Local
governments. And if Congress can grant the City of Manila the power to tax certain matters, it can also provide for
exemptions or even take back the power.”

The City of Manila's power to impose license fees on gambling, has long been revoked. Only the National
Government has the power to issue "licenses or permits" for the operation of gambling. Necessarily, the
power to demand or collect license fees which is a consequence of the issuance of "licensesor permits" is
no longer vested in the City of Manila.

“Local governments have no power to tax instrumentalities of the National Government. PAGCOR is a government
owned or controlled corporation with an original charter.”

The power of local government to "impose taxes and fees" is always subject to "limitations" which
Congress may provide by law. Since PD 1869 remains an "operative'' law, its "exemption clause" remains
as an exception to the exercise of the power of local governments to impose taxes and fees. It cannot
therefore be violative but rather is consistent with the principle of local autonomy. Besides, the principle of
local autonomy under the 1987 Constitution simply means "decentralization". It does not make local
governments sovereign within the slate or an - imperiurn in imperio.

Case no. 105


Dadole vs COA
G.R. No. 125350, Dec. 3, 2002
William
CONSTI I - CHAPTER 5
CASE DIGESTS

SECTION 25. The State shall ensure the autonomy of local governments.

FACTS:
Local Budget Circular No. 55: Issued by the Department of Budget and Management made effective on
March 15, 1994, limited the grant of the city or municipality to national employees (RTC and MTC judges)
to Php. 1000 and Php. 700 respectively from the previous Php. 1500 allowance granted. The judges were
also asked to reimburse the amount in excess of Php. 1000 from the last six months.
Petitioner argues that the circular is beyond the control of DBM and that the local government enjoys the
autonomy to disburse additional allowance for national employees.

ISSUE:
Whether or not Circular No. 55 is void.

HELD:
Yes. Although the Constitution guarantees autonomy to local government units, the exercise of local
autonomy remains subject to the power of control by Congress and the power of supervision by the
President. Sec 4 Art X of 1987 Constitution: "The President of the Philippines shall exercise general
supervision over local governments.” The said provision has been interpreted to exclude the power of
control.

The members of the Cabinet and other executive officials are merely alter egos of the President. As such,
they are subject to the power of control of the President; he will see to it that the local governments or their
officials were performing their duties as provided by the Constitution and by statutes, at whose will and
behest they can be removed from office; or their actions and decisions changed, suspended or reversed.
They are subject to the President's supervision only, not control, so long as their acts are exercised within
the sphere of their legitimate powers. The President can only interfere in the affairs and activities of a LGU
if he or she finds that the latter has acted contrary to law. This is the scope of the President's supervisory
powers over LGUs.
Case no. 106
Limbona vs Mangelin
G.R. No. 80391, Feb. 28, 1989
William

SECTION 25. The State shall ensure the autonomy of local governments.

FACTS:
Petitioner, Sultan Alimbusar Limbona, was elected Speaker of the Regional Legislative Assembly or
Batasang Pampook of Central Mindanao. Congressman Matalam, Chairman of the Committee on Muslim Affairs
of the House of Representative invited the petitioner in his capacity as speaker of the Assembly to participate in
consultation and dialogue regarding the charting of the autonomous government of Muslim Mindanao to be held in
Manila. Petitioner sent a telegram through the Secretary of the Assembly to all the members thereof informing that
―there will be no session this November‖ in view of the invitation of Cong. Matalam. However, on November 2, 1987,
the Assembly held session in defiance of the Limbona's advice, where he was unseated from his
position.Respondents allege that because the Sangguniang Pampook are "autonomous," the courts may
not rightfully intervene in their affairs, much less strike down their acts.

ISSUE:
CONSTI I - CHAPTER 5
CASE DIGESTS
Are the so-called autonomous governments of Mindanao, as they are now constituted, subject to the
jurisdiction of the national courts? In other words, what is the extent of self-government given to the two
autonomous governments of Region IX and XII?

HELD:
Yes, it may assume jurisdiction. In resolving this case the SC made a differentiation between
decentralization of administration and decentralization of power.

Decentralization of administration is when the central government delegates administrative powers to


political subdivisions in order to broaden the base of government power and in the process to make local
governments "more responsive and accountable," and "ensure their fullest development as self-reliant
communities and make them more effective partners in the pursuit of national development and social
progress." At the same time, it relieves the central government of the burden of managing local affairs and
enables it to concentrate on national concerns. The President exercises "general supervision" over them,
but only to "ensure that local affairs are administered according to law." He has no control over their acts in
the sense that he can substitute their judgments with his own.

Decentralization of power involves an abdication of political power in the favor of local governments units
declared to be autonomous. In that case, the autonomous government is free to chart its own destiny and
shape its future with minimum intervention from central authorities.

SC held: An examination of the very Presidential Decree creating the autonomous governments of
Mindanao persuades us that they were never meant to exercise autonomy in the second sense, that is, in
which the central government commits an act of self-immolation. Presidential Decree No. 1618, in the first
place, mandates that "the President shall have the power of general supervision and control over
Autonomous Regions." In the second place, the Sangguniang Pampook, their legislative arm, is made to
discharge chiefly administrative services.

Section 26
Case no. 107
Pamatong vs. COMELEC
G.R. No. 161872, April 13, 2004
William

SECTION 26. The State shall guarantee equal access to opportunities for public service, and prohibit political
dynasties as may be defined by law.

FACTS:
Petitioner Rev. Elly Velez Pamatong filed his Certificate of Candidacy for President. COMELEC refused to
give due course to petitioner‘s Certificate of Candidacy in its Resolution No. 6558 dated January 17, 2004.
The decision, however, was not unanimous since Commissioners Luzviminda G. Tancangco and Mehol K.
Sadain voted to include petitioner as they believed he had parties or movements to back up his candidacy.

On January 15, 2004, petitioner moved for reconsideration of Resolution No. 6558. The COMELEC, acting
on petitioner‘s Motion for Reconsideration and on similar motions filed by other aspirants for national
elective positions, denied the same under the aegis of Omnibus Resolution No. 6604 dated February 11,
2004. The COMELEC declared petitioner and thirty-five others nuisance candidates who could not wage a
nationwide campaign and/or are not nominated by a political party or are not supported by a registered
political party with a national constituency.
CONSTI I - CHAPTER 5
CASE DIGESTS

Commissioner Sadain maintained his vote for petitioner. By then, Commissioner Tancangco had retired.
Petitioner seeks to reverse the resolutions which were allegedly rendered in violation of his right to equal
access to opportunities for public service under Section 26, Article II of the 1987 Constitution, by limiting the
number of qualified candidates only to those who can afford to wage a nationwide campaign and/or are
nominated by political parties. In so doing, petitioner argues that the COMELEC indirectly amended the
constitutional provisions on the electoral process and limited the power of the sovereign people to choose
their leaders. The COMELEC supposedly erred in disqualifying him since he is the most qualified among all
the presidential candidates, i.e., he possesses all the constitutional and legal qualifications for the office of
the president, he is capable of waging a national campaign since he has numerous national organizations
under his leadership, he also has the capacity to wage an international campaign since he has practiced
law in other countries, and he has a platform of government. Petitioner likewise attacks the validity of the
form for the Certificate of Candidacy prepared by the COMELEC. Petitioner claims that the form does not
provide clear and reasonable guidelines for determining the qualifications of candidates since it does not
ask for the candidate‘s bio-data and his program of government.

ISSUE:
Whether the constitutional provision ensuring ―equal access to opportunities for public office‖ creates a
constitutional right to run for or hold public office and, particularly in his case, to seek the presidency?

HELD:
NO. Implicit in the petitioner‘s invocation of the constitutional provision ensuring ―equal access to
opportunities for public office‖ is the claim that there is a constitutional right to run for or hold public office
and, particularly in his case, to seek the presidency. There is none. What is recognized is merely a privilege
subject to limitations imposed by law. Section 26, Article II of the Constitution neither bestows such a right
nor elevates the privilege to the level of an enforceable right. There is nothing in the plain language of the
provision which suggests such a thrust or justifies an interpretation of the sort. The “equal access”
provision is a part of the Article II of the Constitution entitled “Declaration of State Policies”. The provisions
under the article are general considered not self-executing. The provision does not contain any judicially
enforceable constitutional right but merely specifies a guideline for legislative or executive action. The
disregard of the provision does not give rise to any cause of action before the courts.

Moreover, the provision as written leaves much to be desired if it is to be regarded as the source of positive
rights. It is difficult to interpret the clause as operative in the absence of legislation since its effective means
and reach are not properly defined. Broadly written, the myriad of claims that can be subsumed under this
rubric appear to be entirely open-ended. Words and phrases such as “equal access,” “opportunities,” and
“public service” are susceptible to countless interpretations owing to their inherent impreciseness. Certainly,
it was not the intention of the framers to inflict on the people an operative but amorphous foundation from
which innately unenforceable rights may be sourced.

As earlier noted, the privilege of equal access to opportunities to public office may be subjected to
limitations. Some valid limitations specifically on the privilege to seek elective office are found in the
provisions of the Omnibus Election Code on Nuisance Candidate and COMELEC Resolution No. 6452
dated December 10, 2002 outlining the instances wherein the COMELEC may motu proprio refuse to give
due course to or cancel a Certificate of Candidacy. everybody equally without discrimination, however, the
equal access clause is not violated. Equality is not sacrificed as long as the burdens engendered by the
limitations are meant to be borne by any one who is minded to file a certificate of candidacy. In the case at
bar, there is no showing that any person is exempt from the limitations or the burdens which they create.

Section 27
CONSTI I - CHAPTER 5
CASE DIGESTS
IN THE SYLLABUS BUT NOT IN THE MIDTERMS CASE LIST.

Case No. X
Libanan vs. Sandiganbayan
G.R. No. 112386, June 14, 1994
William

SECTION 27. The State shall maintain honesty and integrity in the public service and take positive and effective
measures against graft and corruption.

Article 11, SECTION 1. Public office is a public trust. Public officers and employees must at all times be accountable
to the people, serve them with utmost responsibility, integrity, loyalty, and efficiency, act with patriotism and justice,
and lead modest lives.

FACTS:
Petitioner Marcelino C. Libanan, the incumbent vice-governor of Eastern Samar was a member of the
Sangguniang Panlalawigan of that province. He was among those charged before the Sandiganbayan on
May 25, 1992 violating Section 3(e) of R.A. No. 3019. Libanan et al. wilfully and unlawfully, through evident
bad faith and manifest partiality, prevent and exclude Agustin B. Docena, a duly appointed and Qualified
replacement of deceased Sangguniang Panlalawigan member Luis A. Capito, from exercising his rights
and prerogatives as a member of the said body, by promulgating in their official capacities Sangguniang
Panlalawigan Resolution No. 01, Series of 1991, wherein accused expressed their recognition of Atty.
Socrates B. Alar as the official replacement of aforesaid deceased member, notwithstanding the recall of
his appointment by the Department of Local Government, to the damage and prejudice of Agustin B.
Docena.

ISSUE:
Whether the order of suspension once implemented would amount to an assault of the sacred covenant
reposed on Libanan by the people of Eastern Samar.

HELD:
The petition is without merit. Petitioner contends that the order of suspension, being predicated on his acts
supposedly committed while still a member of the Sangguniang Bayan, can no longer attach to him now
that he is the duly elected and incumbent Vice-Governor of Eastern Samar. In Deloso vs. Sandiganbayan,
this Court rejected a similar argument advanced by Governor Deloso who, at the time of issuance of the
suspension order, was already occupying the office of governor and not the position of municipal mayor
that he held previously when charged with having violated the Anti-Graft Law. Prior to Deloso, in Bayot vs.
Sandiganbayan, the suspension of then Cavite mayor Bayot was also sustained even as he was charged
for acts committed as a government auditor of the Commission on Audit. In both instances, this Court ruled
that the term "office" used in the law could apply to any office which the officer charged might currently be
holding and not necessarily the particular office under which he was charged. Libanan’s so-called
"covenant" with the people of Eastern Samar is far from being synonymous to, or the equivalent of, license,
and it is not one that can cut athwart the long arm of the law.
CONSTI I - CHAPTER 5
CASE DIGESTS

Section 28
Case no. 108
Aquino-Sarmiento vs. Morato
G.R. No. 92541, Nov. 13, 1991
William

SECTION 28. Subject to reasonable conditions prescribed by law, the State adopts and implements a policy of full
public disclosure of all its transactions involving public interest.

FACTS:
FEBRUARY 1989: Petitioner, a member of respondent MTRCB, wrote its records officer requesting that
she be allowed to examine the board's records pertaining to the voting slips accomplished by the individual
board members after a review of the movies and television productions. It is on the basis of said slips that
films are either banned, cut or classified accordingly. Petitioner's request was denied by respondent Morato
on the ground that whenever the members of the board sit in judgment over a film, their decisions as
reflected in the individual voting slips partake the nature of conscience votes and as such, are purely and
completely private and personal. Petitioner counters that the records she wishes to examine are public in
character and other than providing for reasonable conditions regulating the manner and hours of
examination, respondents have no authority to deny any citizen seeking examination of the board's
records.

ISSUE:
Was the denial made by the respondent on the request of the petitioner proper?

HELD:
We find respondents' refusal to allow petitioner to examine the records of respondent MTRCB, pertaining to
the decisions of the review committee as well as the individual voting slips of its members, as violative of
petitioner's constitutional right of access to public records. May the decisions of respondent Board and the
individual members concerned, arrived at in an official capacity, be considered private? Certainly not. As
may be gleaned from the decree (PD 1986) creating the respondent classification board, there is no doubt
that its very existence is public in character; it is an office created to serve public interest. it being the case,
respondents claim to privacy is not valid. The right to privacy belongs to the individual acting in his private
capacity and not to a governmental agency or officers tasked with, and acting in, the discharge of public
duties.

Case no. 109


Legaspi vs CSC
G.R. No. L- 72119, May 29, 1987
William

SECTION 28. Subject to reasonable conditions prescribed by law, the State adopts and implements a policy of full
public disclosure of all its transactions involving public interest.

FACTS:
The fundamental right of the people to information on matters of public concern is invoked in this special
civil action for mandamus instituted by petitioner Valentin L. Legaspi against the Civil Service Commission.
The respondent had earlier denied Legaspi's request for information on the civil service eligibilities of
certain persons employed as sanitarians in the Health Department of Cebu City. These government
CONSTI I - CHAPTER 5
CASE DIGESTS
employees, Julian Sibonghanoy and Mariano Agas, had allegedly represented themselves as civil service
eligibles who passed the civil service examinations for sanitarians. Claiming that his right to be informed of
the eligibilities of Julian Sibonghanoy and Mariano Agas, is guaranteed by the Constitution, and that he has
no other plain, speedy and adequate remedy to acquire the information, petitioner prays for the issuance of
the extraordinary writ of mandamus to compel the respondent Commission to disclose said information.

ISSUE:
Whether or not Legaspi should be allowed such right.

HELD:
Yes, Legaspi should be allowed such right. The constitutional right to information on matters of public
concern is recognized in the Bill of Rights. These constitutional provisions are self-executing. They supply
the rules by means of which the right to information may be enjoyed by guaranteeing the right and
mandating the duty to afford access to sources of information. Hence, the fundamental right therein
recognized may be asserted by the people upon the ratification of the constitution without need for any
ancillary act of the Legislature. What may be provided for by the Legislature are reasonable conditions and
limitations upon the access to be afforded which must, of necessity, be consistent with the declared State
policy of full public disclosure of all transactions involving public interest.

For every right of the people recognized as fundamental, there lies a corresponding duty on the part of
those who govern, to respect and protect that right. That is the very essence of the Bill of Rights in a
constitutional regime. Only governments operating under fundamental rules defining the limits of their
power so as to shield individual rights against its arbitrary exercise can properly claim to be constitutional.
Without a government's acceptance of the limitations imposed upon it by the Constitution in order to uphold
individual liberties, without an acknowledgment on its part of those duties exacted by the rights pertaining
to the citizens, the Bill of Rights becomes a sophistry, and liberty, the ultimate illusion.

In recognizing the people's right to be informed, both the 1973 Constitution and the New Charter expressly
mandate the duty of the State and its agents to afford access to official records, documents, papers and in
addition, government research data used as basis for policy development, subject to such limitations as
may be provided by law. The guarantee has been further enhanced in the New Constitution with the
adoption of a policy of full public disclosure, this time "subject to reasonable conditions prescribed by law,"

Article 11, Section 28: Subject to reasonable conditions prescribed by law, the State adopts and implements a policy
of full public disclosure of all its transactions involving public interest

The absence of discretion on the part of government agencies in allowing the examination of public
records, specifically, the records in the Office of the Register of Deeds, is emphasized in Subido vs.
Ozaeta, supra: Except, perhaps when it is clear that the purpose of the examination is unlawful, or sheer,
idle curiosity, we do not believe it is the duty under the law of registration officers to concern themselves
with the motives, reasons, and objects of the person seeking access to the records. It is not their
prerogative to see that the information which the records contain is not flaunted before public gaze, or that
scandal is not made of it. If it be wrong to publish the contents of the records, it is the legislature and not
the officials having custody thereof which is called upon to devise a remedy.

Thus, while the manner of examining public records may be subject to reasonable regulation by the
government agency in custody thereof, the duty to disclose the information of public concern, and to afford
access to public records cannot be discretionary on the part of said agencies. Certainly, its performance
cannot be made contingent upon the discretion of such agencies. Otherwise, the enjoyment of the
constitutional right may be rendered nugatory by any whimsical exercise of agency discretion. The
CONSTI I - CHAPTER 5
CASE DIGESTS
constitutional duty, not being discretionary, its performance may be compelled by a writ of mandamus in a
proper case.

Case no. 110


Valmonte vs. Belmonte
G.R. No. 74930, February 13, 1989
William

SECTION 28. Subject to reasonable conditions prescribed by law, the State adopts and implements a policy of full
public disclosure of all its transactions involving public interest.

FACTS:
Petitioner Valmonte wrote a letter to respondent Belmonte, General Manager of GSIS, requesting the latter
to furnish him the list of the names of the Batasang Pambansa members belonging to the UNIDO and PDP-
Laban who were able to secure clean loans immediately before the February 7 election thru the
intercession/marginal note of the then First Lady Imelda Marcos. The Deputy General counsel of the GSIS
wrote back the petitioner turning down his request on the ground that there exists a confidential relationship
between the GSIS and all those who borrow from it, which confidence it is the GSIS is duty bound to
preserve.

ISSUE:
Whether or not the writ of mandamus lies to compel respondents to perform the acts sought by the
petitioner to be done in pursuant with his right to information.

HELD:
Yes. The people‘s right to information is limited to matters of public concern and is further subject to such
limitations as may be provided by law. The GSIS is a trustee of contributions from the government and its
employees and administration of various insurance programs for the benefit of the latter. Undeniably, its
funds assume a public character. It is the legitimate concern of the public to ensure that these funds are
managed properly with the end in view of maximizing the benefits to insured government employees. The
public nature of the loanable funds of the GSIS and the public office held by the alleged borrowers make
the information sought clearly a matter of public interest and concern. Furthermore, the "constituent-
ministrant" dichotomy characterizing government function has long been repudiated. That the GSIS, in
granting the loans, was exercising a proprietary function would not justify the exclusion of the transactions
from the coverage and scope of the right to information. The Court said that the government, whether
carrying out its sovereign attributes or running some business, discharges the same function of service to
the people.

Respondent next asserts that the documents evidencing the loan transactions of the GSIS are private in
nature and hence, are not covered by the Constitutional right to information on matters of public concern
which guarantees access to official records, and to documents, and papers pertaining to official acts,
transactions, or decisions" only.

Moreover, the intent of the members of the Constitutional Commission of 1986, to include government-
owned and controlled corporations and transactions entered into by them within the coverage of the State
policy of full public disclosure is manifest from the records of the proceedings Considering the intent of the
framers of the Constitution which, though not binding upon the Court, are nevertheless persuasive, and
considering further that government-owned and controlled corporations, whether performing proprietary or
governmental functions are accountable to the people, the Court is convinced that transactions entered into
by the GSIS, a government-controlled corporation created by special legislation are within the ambit of the
CONSTI I - CHAPTER 5
CASE DIGESTS
people's right to be informed pursuant to the constitutional policy of transparency in government dealings.
In fine, petitioners are entitled to access to the documents evidencing loans granted by the GSIS, subject to
reasonable regulations that the latter may promulgate relating to the manner and hours of examination, to
the end that damage to or loss of the records may be avoided, that undue interference with the duties of
the custodian of the records may be prevented and that the right of other persons entitled to inspect the
records may be insured.

However, although citizens are afforded the right to information and, pursuant thereto, are entitled to
"access to official records," the Constitution does not accord them a right to compel custodians of official
records to prepare lists, abstracts, summaries and the like in their desire to acquire information on matters
of public concern.

You might also like